Tổng hợp đề và lời giải đề chọn đội tuyển TST Việt Nam

184 346 1
Tổng hợp đề và lời giải đề chọn đội tuyển TST Việt Nam

Đang tải... (xem toàn văn)

Tài liệu hạn chế xem trước, để xem đầy đủ mời bạn chọn Tải xuống

Thông tin tài liệu

Chú ý là số điểm là đầu mút của các đoạn thẳng là 4 4 4 5 D 1 9 8 0 < 2 0 1 7 nên bài toán sẽ được giải quyết hoàn toàn nếu ta chứng minh được mệnh đề sau: Nếu có n điểm trên mặt phẳ[r]

(1)(2)

1 LỜI GIẢI ĐỀ THI CHỌN ĐỘI TUYỂN QUỐC GIA

DỰ THI IMO 2011

*********** Bài

Trên mặt phẳng tọa độ có cào cào điểm (1;1) Nó nhảy từ điểm A sang điểm B tam giác OAB có diện tích 1

2 tọa độ A, B nguyên dương

1 Tìm điểm ( , )m n cho cào cào có thể nhảy đến sau hữu hạn bước 2 Chứng minh cào cào nhảy đến ( , )m n kể sau mn bước. Lời giải

Trước hết, ta cần chứng minh nhận xét sau:

Với ( , )m n cặp số nguyên tố tồn a b nguyên tố cho

mbnaa b m n Thật vậy, khơng tính tổng qt, giả sử mn

Xét số có dạng mb1,1 b n Dễ thấy ta có n số bb'

1 ' 1(mod )

(3)

2

Đặt a mb a mn m

n n

    mban1

Ta có n b( a)  (mn b)  1 (mn n) (mn b)(   n) (mn n)   b a mn Do b a m n Nhận xét chứng minh

Trở lại toán, ta thấy rằng, cào cào đỉnh A a b( , ) muốn nhảy sang đỉnh B c d( , )

thì phải có 1 1

2 2

OAB

S   adbc   adbc

Từ suy xuất phát từ điểm ( ; )1 , cào cào nhảy đến điểm ( ; )m n mà ( , )m n 1 Hơn nữa, với ( , )m n 1 theo nhận xét cào cào nhảy đến điểm có tọa độ tương ứng ( ; )m n Vậy điểm cần tìm ( ; )m n với m n, nguyên tố Xét điểm nguyên dương ( ; )m n mà ( , )m n 1,m n 1 cào cào nhảy đến điểm sau bước nhảy

Ta lại xét điểm ( ; )m n mà ( , )m n 1, m n 1 theo nhận xét trên, tồn số m n', ' cho mm'nn' 1 m'n'  mn 1

Lặp lại trình này, ta thấy cào cào nhảy đến điểm ( ; )m n sau không m n bước

(4)

3 Bài

Cho đường (O) điểm A nằm ngồi đường trịn Kẻ hai tiếp tuyến AB AC t, ới (O) với A, B các tiếp điểm Gọi P điểm thuộc tia đối tia BA, Q điểm thuộc tia đối tia CA cho PQ tiếp xúc với (O) Qua P kẻ đường thẳng song song với AC cắt đường thẳng BC E Qua Q kẻ đường thẳng song song với AB cắt BC F

1 Chứng minh đường thẳng QE PF, luôn qua điểm cố định M, N 2 Chứng minh tích PM QNkhông đổi

Lời giải

Trước hết, ta chứng minh nhận xét sau: Cho tam giác ABC ngoại tiếp (I) có tiếp điểm (I) lên AB, AC E, F Đường thẳng qua B, song song với AC cắt EF K; CK cắt AB G Khi đó, tam giác AGI vuông I.

Thật vậy:

Đặt ABc BC, a CA, b chu vi tam giác ABC 2p

Do BK // AC nên tam giác BKF cân B, suy ra: AEAF p a Theo định lí Thales thì: BG BK p b AB p AG bc

AG AC b AG b p

     

AF p a nên AF p p( a)

AG bc

Ta có: , sin sin2 cos ( )

2 2

sin

AF A AH A A p p a

AI AH AF

A AI bc

 

     

Từ suy ra: AF AH p p( a)

AG AI bc

(5)

4 Bổ đề chứng minh Trở lại toán cho

1/ Gọi M, N giao điểm QE với AB PF với AC Theo bổ đề trên, ta thấy tam giác OMA ONA vuông O nên điểm M, N cố định

2/ Trước hết, ta đặt

, ,

ABACa BPx CQy Chu vi tam giác APQ

2( )

pa x y

Theo bổ đề trên, ta tính được:

( ) ( )

AP PQ

PM AP

AP AQ PQ

AP AP AQ PQ a x x

AP AQ PQ a x y

  

 

   

   

Tương tự QN (a y y)

a x y

 

 

Cần chứng minh tích ( )( 2 )

( )

xy a x a y PM QN

a x y

 

 

  không đổi Thật vậy: Nếu gọi R bán kính (O) diện tích tam giác APQ bằng:

sin ( )( )

2 sin

pR BAC AP AQ a x a y R

a x y BAC

 

  

 

Suy tỉ số (a x a)( y) k

a x y

  

  không đổi, với sin

R k

BAC

 Từ đó, ta có

(a x a)( y) k a( x y) a a( x y) xy k a( x y) a xy k

a x y

              

  , tức tỉ số xy

a x y không đổi

Vậy ( )( 2 )

( )

xy a x a y PM QN

a x y

 

 

(6)

5 * Cách khác:

1 Ta chứng minh giao điểm M EQ AB, giao điểm N FP AC điểm cố định

Giả sử (O) tiếp xúc với BC D Gọi K, L giao điểm QO BC, PK OD theo thứ tự

Dễ thấy PKO900,suy ra: cặp tam giác LKO, LDP LOP, QOA đồng dạng với Suy ra,

OK LO OQ OK PB

PDLPAQOQAQ Mặt khác dễ thấy PEPB

nên OK PE EM

OQAQMQ Áp dụng định lí thales đảo ta được: MO song song với BC đoạn cố định mà M thuộc AB cố định nên M cố định Tương tự, N điểm cố định Ta có đpcm

2.Từ câu 1), ta có M O N, , thẳng hàng Từ dễ có PMOONQ g g( ) Suy PM QN OM ON OM2là số

Ta chứng minh M, N tiếp điểm đường tròn A-Mixitilinear đoạn AP, AQ tam giác APQ

L

N K

M

F

E

Q P

C B

O A

(7)

6 Bài 3

Cho cho n nguyên dương thỏa n3và n số thực x x x1, 2, 3, ,x thn ỏa mãn đồng thời (i) x1   x2 x3 xn0

(ii) x12x22x23  xn2n n( 1) (iii) x1x2x3  xn

Tìm giá trị lớn nhỏ của f  x1 x2 Lời giải

*Tìm giá trị lớn

Theo BĐT Bunhiacopski cho n2 số thực x x3, 4, ,xn, ta có:

2

2 2 2 2

3

( )

(1 1 )( ) ( )

2 n

n n n

x x x

x x x x x x x

n

  

           

 Suy

2 2

2 2 2

1 2

2

1 2

( ) ( ) ( )

( 1)

2 2

2 ( 1)( 2)

( ) 2( 1)( 2) 2( 1)( 2)

( 2)

n n

x x x x x x x

n n x x x x x x

n n

n n n

x x n n x x n n

n                                 

Do đó, giá trị lớn f  x1 x2 2(n1)(n2), đạt

1 2, n

xx xx  x hay

( 1)( 2) 2( 1)

,

2 n

n n n

x x x x x x

n

  

       

*Tìm giá trị nhỏ nhất.

Ta xét trường hợp:

- Nếu n3 x12x2 x1 x2x30 2x1x2 x1 x2x30

Thay x3 (x1x2) vào đẳng thức (ii), ta 2 2

1 ( 2) 2 xxxx  xxx x  Từ đánh giá (2x1x2)(x12x2) 0 x12x22x x1 2(2x1x2)(x12x2) 3 (x1x2)21 Dễ thấy x1x2 1 x3   1 x1 x2, mâu thuẫn nên x1x21

Đẳng thức xảy x12,x2x3 1

(8)

7 Đánh giá

2 2 2

( ) 2 ( ) ( )( )

bca   b c aaa b c bc  ab a c Áp dụng đánh giá với x2x3x4, ta được: 2 2

3 ( 2) xxxx  x x Dễ thấy x2  x3 x4 x x2, 2x5 nên áp dụng tiếp đánh giá (*), ta có:

2 2 2 2

3 ( 2) 2 ( 2) xxxxx  x xxxx  x x Tiếp tục áp dụng đánh giá (*) với số a a6, 7, ,an, ta được:

 2  2

2 2

1 2 n ( 3) n ( 3) ( 3) ( 2) xxx  x  n xx     x x n xnxx  n x Suy x12 (n 2)x22x1 (n 2)x22n n( 1)

Ta chứng minh

 2

2 2 2

1 2 2

( 1)( )

( 2) ( 2) ( 2) 2( 2)

4

n n x x

x  n xx  n xxnnxnx x   

Bất đẳng thức tương đương với: (x1x2) ( n2 n 8)x1 (n 1)(3n8)x20, mà bất đẳng thức x1x2

2

1 1

(n  n 8)x  (n 1)(3n8)x (n  n 8)x (3n8)xn n( 4)x 0 Từ điều này, ta có:

2

2

1

( 1)( )

( 1) ( )

4

n n x x

n n     xx  , dễ thấy x1x22 Trong trường hợp này, giá trị nhỏ biểu thức cho 2, đạt

1 n 1, n xxx   x  x  n

Vậy giá trị lớn f  x1 x2 2(n1)(n2), đạt

1 2, n

xx xx  x hay 1 2 ( 1)( 2), 3 4 5 2( 1)

2 n

n n n

x x x x x x

n

  

       

 Giá trị nhỏ f  x1 x2 với n3, đạt x12,x2 x3 1 với

4

n , đạt 1 2 ( 1)( 2), 3 4 5 2( 1)

2 n

n n n

x x x x x x

n

  

       

(9)

8 Bài

Cho dãy số(an)thỏa mãn a01,a13

2 n n n a a a           Chứng minh với số tự nhiên n a an n2an212n Lời giải

Dễ thấy số hạng dãy số nguyên dương

Trước hết, quy nạp, ta chứng minh cho thỏa mãn hệ thức truy hồi ,

n n n

a 2 4a 12a  n

Thật vậy, tính tốn trực tiếp, ta có a2 10,a3 34,a4 116 thỏa mãn hai hệ thức truy hồi, tức khẳng định với n0 2, ,

Giả sử khẳng định đến nk k, 2, tức ak2 4ak12ak Ta chứng minh với nk1

Ta có k ( k k) k

k k k

k k k

a a a a

a a a

a a a

                            

2 2

2

3

1 1

4

1 1 16 16

Ta lại cần chứng minh k ,  * k

k a

a k

a 

         1

4

Thật vậy,

(*) k k

k k k k k k

k k

a a

a a a a a a

a   a    

        

2

2

1 1

1

4

1

4

Hơn nữa, theo giả thiết k k k

k k k k k k k

k k k

a a a

a a a a a a a

a a a

    

  

 

           

2 2

2

1 1 1

1 1

1

Theo giả thiết quy nạp, ta có

( ) ( )

k k k k k k k k k k k k

a 1  aa 1  a 1 a 2  a 1 a 1 a 1a 2  a 1a 1 a 1

4 4 2

4 Do đó, ta có ak2 ak1ak1 ak21ak1

4 hay (*) Từ ,

k

k k

a

a k

a  

         1

(10)

9

( ) ( )

k k k k k k k k k k

a 3  1 16a 116a 4a 1 1 4a 12a 2 4a 2a 1 4a 2 2a 1

Suy khẳng định với nk1 theo giả thiết quy nạp khẳng định chứng minh Ta chứng minh dãy số thỏa mãn a0 1,a1 3,an 4an12an2,n2 thỏa mãn đẳng

thức n

n n n

a 2aa21 2 với n (**)

(11)

10 Bài

Tìm tất số nguyên dương n cho biểu thức sau

2n (2n 1) 8.3n

A    

là số phương Lời giải

Giả sử tồn số n nguyên dương thỏa mãn đề Khi đó, tồn m cho:

2 2 1

2n (2n 1) 3n n 2n 3n (2n 1) 8.3n

m                  

hay ( )( )

n n n m m         1

2

2

4

Đặt ( ) , ( )

n n t t a b         1

2

2 ,

n n

a bab

 2 11  2

Dễ thấy t số lẻ nên a b, số nguyên dương Do đó, a3u,b 2 3v ngược lại Trong hai trường hợp, ta có 3u  2 3v 2n11 u v n

Ta xét trường hợp sau:

- Nếu n1 3u  2 3v 3,u v 1, hệ khơng có nghiệm nguyên dương - Nếu n2 u v ,

u v

    

3 2, hệ khơng có nghiệm nguyên dương - Nếu n3 3u 2 3v15,u v 3, hệ có nghiệm u2,v1

- Nếu n4 3u  2 3v 31,u v 4, hệ khơng có nghiệm ngun dương dương - Nếu n5 3u  2 3v 63,u v 5, hệ có nghiệm u2,v3

- Nếu n5, ta chứng minh hệ tương ứng trường hợp vơ nghiệm Ta có

( )

( ) ( )

n

u n u n n n

u n v v

   

          

2

1 2

3

3 3

Tương tự, ta có un21

3 Đặt min{ , }

n

wu v  2 1 w2

3 Suy

(mod ) n

 

2 Bằng cách thử trực tiếp, ta có n 1 0(mod )6 hay * ,

(12)

11 Từ đó, ta 3u  2 3v (2k1 2)( k 1 4)( 2k4k 1)

Dễ dàng thấy 42k 4k    1 1 0(mod )3 42k 4k 1 2k4k  1 0(mod )9 nên 3w1 ước (2k1 2)( k1) Suy 3w12k 1 3kn2 1 w 1 n1n11

3

Dễ thấy không tồn số nguyên dương nn 1 0(mod )6 5n11 nên trường hợp này, hệ khơng có nghiệm

(13)

12 Bài

Có n học sinh ngồi quanh bàn tròn, tay học sinh có số kẹo cho tổng số kẹo tất học sinh bội số n Ta thực quy tắc chuyển kẹo sau: có học sinh có số kẹo lớn số kẹo người bạn bên tay phải ta lấy người chuyển sang cho người bạn bên tay phải

Chứng minh với quy tắc trên, sau số hữu hạn bước với cách chuyển thích hợp, số kẹo học sinh nhau.

Lời giải

Xuất phát từ học sinh đó, theo chiều ngược chiều kim đồng hồ, ta đánh số em 3, , , ,n Khi đó, với i1,n, ngồi bên phải em i em i1

(với quy ước em n1 em 1)

Với *

t , ta định nghĩa thời điểm t thời điểm nằm lần chuyển kẹo thứ t thứ t1 Với i1,n, kí hiệu x số kẹo em i thời điểm gọi x ti( ) số kẹo em i thời điểm t Ta gọi ( ,x x x1 2, 3, ,xn) trạng thái

Xét đại lượng n

i i

F x

 

1

xét thời điểm t tùy ý Giả sử, lần chuyển kẹo thứ t1, người thực việc chuyển kẹo em i Khi đó:

 

( ) ( ), ( ) ( ), ;

( ) ( ) , ( ) ( )

i i j j

i i i i

x t x t x t x t j i i

x t x t x t x t

 

     

     

1

1

1

1 1

Do F t( 1)F t( )x ti( )12xi1( )t 12x ti( )2xi1( )t 2 2xi1( )t  1 x ti( )0 Và F t( 1)F t( )0x ti( )xi1( )t 1 (*) (F t( ) kí hiệu giá trị F thời điểm t) Như vậy, giá trị F khơng tăng q trình chuyển kẹo F nhận giá trị thời điểm liên tiếp t t, 1 lần chuyển kẹo thứ t + 1, em chuyển kẹo có nhiều em nhận kẹo kẹo Hơn nữa, trạng thái cho ta nghiệm tự nhiên phương trình

n x1x2 x3 xn

(14)

13 Xảy hai trường hợp sau:

* Trường hợp 1: F nhận giá trị số hữu hạn thời điểm liên tiếp

Trong trường hợp này, sau số hữu hạn lần chuyển kẹo, F nhận giá trị nhỏ kể từ thời điểm (đầu tiên) F nhận giá trị nhỏ đó, việc chuyển kẹo thực thêm số hữu hạn lần Khi việc chuyển kẹo khơng thể thực được, ta phải có

n

x1x2 x3  xx1 hay x1 x2 x3  xn Khẳng định toán chứng minh

* Trường hợp : Tồn giá trị mà F nhận giá trị vơ hạn thời điểm liên tiếp Giả sử F t( )F t( 1), t t0 Khi đó, theo (*), kể từ thời điểm t0, lần chuyển kẹo chỉđược thực em i mà thời điểm xixi1 1 (**)

Do kể từ thời điểm t0 ta nhận vô số trạng thái mà có hữu hạn trạng thái đơi khác (chứng minh trên) nên phải tồn trạng thái xuất tối thiểu lần Xét trạng thái số trạng thái vậy, gọi A

Giả sử A xuất thời điểm t1t0; gọi k số nguyên dương nhỏ cho thời điểm t1k, A lại xuất lần

Gọi (C) trình chuyển kẹo kể từ thời điểm t1 đến thời điểm t1k

Do (C) khởi đầu kết thúc trạng thái nên q trình em i nhận kẹo lần chuyển kẹo sau i phải thực việc chuyển kẹo cho bạn ngồi bên phải Từ đó, em ngồi quanh bàn tròn, suy (C), em phải thực việc chuyển kẹo lần

Hơn nữa, tổng số kẹo em bội n nên với ( ,x x1 2, ,xn) trạng thái tùy ý, ta phải có x1 x2 x3  xn, tồn i j 1 3, , , ,n cho xixj 2 Suy ra, trạng thái (C) có tính chất: tồn i j 1 3, , , ,n cho xixj 2 Gọi m số nguyên dương nhỏ cho (C) có trạng thái mà trạng thái tồn i1 3, , , ,n cho xixi m 2 Gọi B trạng thái có tính chất gần A (có thể BA) Giả sử B xuất (trong (C)) thời điểm t2 t1

(15)

14 - Khả 1: m1, ta lại xét tiếp trường hợp:

+ Trường hợp 1.1: BA Khi đó, lần (trong (C)) chuyển kẹo cho em i1, em i có nhiều i + kẹo; mâu thuẫn với (2)

+ Trường hợp 1.2 : BA Khi đó, trạng thái A, ta có xixi1 1 Do đó, để trở lại trạng thái A, sau thời điểm t2, i phải chuyển kẹo cho em i1 lần lần lần i có nhiều i1ít kẹo; mâu thuẫn với (2)

- Khả 2: m1 Khi đó, từ định nghĩa m suy số kẹo em

, , , ,

i i1 im1im(trong trạng thái B) phải thỏa mãn

,

i i i i i m i m

xx11 x1 x1  x 1 x 1

+ Trường hợp 2.1: BA Trong trường hợp này, lần có em , , , ,

i1i i1 im thực việc chuyển kẹo; em chuyển kẹo i1hoặc i hoặcim1 im

 Nếu i1 thực việc chuyển kẹo sau bước chuyển đó, ta có trạng thái mà xixi12

 Nếu i thực việc chuyển kẹo sau bước chuyển đó, ta có trạng thái mà i i m

x1x 2

 Nếu im1thực việc chuyển kẹo sau bước chuyển đó, ta có trạng thái mà i i m

xx 1 2

 Nếu im thực việc chuyển kẹo sau bước chuyển đó, ta có trạng thái mà i m i m

x 1x 2

Trong tình trên, ta nhận điều mâu thuẫn với tính nhỏ m + Trường hợp 2.2 : BA Khi đó, trạng thái A, số ,x xi i1, ,xi m 1,xi m không thỏa mãn quan hệ (∗) Vì thế, để trở lại trạng thái A, sau thời điểm t2 phải có thời điểm mà sốđó thay đổi Điều có có em

, , , ,

i1i i1 im thực việc chuyển kẹo Lập luận hoàn toàn tương tự trường hợp 2.1, ta nhận điều mâu thuẫn với tính nhỏ m

(16)

Vietnam TST 2012 – Li gii bình lun

Trần Nam Dũng & K0

Kỳ thi chọn đội tuyển Việt Nam tham dự IMO 2012 diễn ngày 16 17/04/2012 Hà Nội Mỗi ngày thí sinh phải giải tốn vịng 30

phút Theo đánh giá chung, đề thi năm thuộc loại khó Vềphân mơn, tốn phân bốnhư sau:

Bài 1. Hình học phẳng (Quỹtích điểm cốđịnh)

Bài 2. Tổ hợp (Phủ)

Bài 3. Số học (Hệ thặng dư) Bài 4. Số học (Dãy số)

Bài 5.Đại số (Bất đẳng thức)

Bài 6. Tổ hợp (Lý thuyết đồ thị)

Nếu sâu vào lời giải thấy toán túy đại số Bài số học mang đậm chất tổ hợp Như thế, thấy đề thi năm nặng Đại số Tổ hợp, phần Số học Hình yếu, dù hình tốn tốt

Vềđộ khó, có dễ chịu cả, lại tốn khó, thách thức đáng kểđối với thí sinh

Một đặc điểm đềthi năm có nhiều tốn sử dụng ý tưởng định lý mạnh định lý Cauchy-Davenport (bài 3), định lý Dirac, định lý Tutte (bài 6) Điều mặt tích cực hướng học sinh đến việc làm quen với vấn đềcơ sở toán cao cấp, mặt khác tạo bất lợi cho học sinh chưa có điều kiện làm quen với kiến thức Đây điều mà người dẫn dắt phong trào HSG Việt Nam phải thảo luận kỹđể có định hướng

Dưới chúng tơi trình bày lời giải chi tiết tốn Vietnam TST 2012 bình luận

Bài viết hoàn thành với tham gia trực tiếp bạn: Võ Quốc Bá Cẩn

(ĐH Y Cần Thơ) và Lê Phúc Lữ(ĐH FPT), Lê Hồng Quý cũng tham gia gián tiếp thầy Nguyễn Chu Gia Vượng (Viện Toán học), thành viên mathscope.org như

(17)

Bài

Trên mt phẳng, cho đường tròn ( )O và hai điểm c định B C , trên đường trịn sao cho BC khơng đường kính ca ( )O Gi A một điểm di động đường tròn ( )O A không trùng với hai điểm B C G, i D K J l, , ần lượt trung điểm ca

, ,

BC CA AB E M N l, , ần lượt hình chiếu vng góc ca A B C , ,

, ,

BC DJ DK

Chng minh rng tiếp tuyến ti M N c, ủa đường tròn ngoi tiếp tam giác EMN ct tại điểm T cđịnh điểm A thay đổi ( )O

Lời giải

Đây toán thú vị với phát biểu nhẹ nhàng, cấu hình khơng q phức tạp gợi nhiều ý tưởng việc xử lí khơng dễ, quan trọng phải đốn điểm cố định nêu Dưới xem xét sốhướng tiếp cận xử lí mở rộng toán

Cách (s dụng hàng điểm điều hòa tgiác điều hòa)

Gọi H trực tâm tam giác ABC Ta xét trường hợp H nằm tam giác, trường hợp lại chứng minh tương tự

Trước hết, ta chứng minh T nằm đường thẳng OD

(18)

Đường thẳng qua H, song song với BC cắt đường thẳng OD điểm S Do HSD900 nên S thuộc đường tròn đường kính HD Gọi X hình chiếu E lên AD X thuộc đường trịn

Ta chứng minh tứ giácDMSN XMEN, tứgiác điều hòa

Thật vậy, HS BC D trung điểm BC nên theo tính chất vềchùm điều hịa, ta có (HS HD HC HB, , , ) 1 hay tứ giác DMSNtương ứng tứ giác điều hịa Theo tính chất tứgiác điều hịa, ta có T nằm đường thẳng DO

Dễ thấy tứ giácDEJKlà hình thang cân nên nên ENK EMJ g g( )

Suy EM EJ AB

ENEKAC Hơn nữa,

sin sin sin

sin sin sin

XM XNM XDM DAC AB

XN XMN XDN DAB AC

  

   

  

Do đó, EM AB

ENAC hay tứ giác XMEN điều hịa Ta có T nằm EX hay T

là giao điểm EX AO

Ta chứng minh khoảng cách từ T đến D không đổi Gọi B hình chiếu B AC Do AHX ADE nên

(19)

hay tứ giác CDXB nội tiếp Suy DXC DB C  DCADX DA DC2

Theo định lí Thales

2

AE DX AD DX DC

DT

AX AH AH

 

  

Dễ thấy DC AH, không đổi nên độdài đoạn DT không đổi hay T điểm cốđịnh

Ta có đpcm

Cách (dùng phương tích, trục đẳng phương)

Gọi R S, trung điểm DB DC, R, S tâm đường tròn ngoại

tiếp tam giác BMD CND, Ta có TM TN, 1

2

MRDB BCDCNS biến đổi góc, ta thu TMR TNS hay TMR TNS c g c( )

Suy TRTS hay T nằm đường trung trực BC

Gọi X tâm đường tròn ngoại tiếp tam giác HBC X cốđịnh Ta chứng minh T nằm trục đẳng phương đường tròn (S) (X) Gọi U trung điểm OD Ta thấy

 

2 2

/( ) /( )

2 2 2 2 2 2

2 2 2 2 2

2

T X T S TX XC TS SC

TX TS XD CD SC TD SD XD CD SC TC XD

TD XD TC XD CD TD XD DS DU DT

   

           

(20)

Điều tương đương với tam giác TSU vuông S Hơn nữa, ta thấy

0 0

0

90 90 180

180

TSU STU SUT RTS BXC

MTN MIN

         

   

Đẳng thức cuối nên suy T nằm trục đẳng phương (S) (X) Do hai

đường tròn cốđịnh nên trục đẳng phương chúng cốđịnh T giao điểm hai đường thẳng cốđịnh nên T điểm cốđịnh Ta có đpcm

Bình luận

So sánh với tốn hình vị trí nhiều năm trở lại khó hẳn

Hướng giải theo đường hình học túy bắt buộc phải kẻ thêm nhiều đường phụvà điều khiến nhiều bạn phải bỏ Có cách giải trường hợp dùng phương pháp tọa độ giả thiết tương đối thuận lợi Đôi cách

tiếp cận đại số đem lại hiệu cao Chúng ta tìm hiểu cách làm biến đổi vector sau:

Ta thấy điểm M, N trung điểm đường cao tương ứng tam giác ABC Các điểm M N E H D, , , , thuộc đường trịn đường kính HD Gọi R điểm đối xứng với O qua đường thẳng BC S giao điểm đường tròn ngoại tiếp tam giác BCR với đường thẳng OD Gọi F chân đường cao kẻ từ C đến AB T trung điểm DS Dễ thấy T điểm cốđịnh Ta tính

cos cos cos cos

(21)

 

0 0 0

90 90 90 90 90

FCS FCR A B A B

           

 

0 0

90 90 90

DCS RCD A A

        nên

2cos BC CS

A

Do T I N, , trung điểm đoạn DS HD CF, , nên ta có:

2NTCSFD, 2NICDFH      

Suy ra: 4NT NI CSFD CD FHCS CD CS FH FD CD FD FH             

Ta tính

2

2 2

sin

BC

CS CD CD  RA  

2 2

cos cos sin cos

FD CD DF DC  CDFCD BR A B  

2

cos cos cos sin 2sin cos sin cos

2 BC

FD FH FD FH  DFH   R A B BRA B B A

 

2

cos cos cos sin( )

2 cos sin cos sin( )

BC

CS FH CS FH FCS R A B A B

A

R A B A B

              Do      

2 2

2 2

4 sin sin cos 2sin cos sin cos sin cos sin( )

sin cos sin cos sin cos sin( )

NT NI R A A B A B B A A B A B

R A B A B B A A B

    

    



Từđó suy NTNI hay TN tiếp tuyến đường tròn ngoại tiếp tam giác MNE Chứng minh tương tự, ta có TM tiếp tuyến đường trịn ngoại tiếp tam giác

Do đó, hai tiếp tuyến kẻ từ M N đường tròn ngoại tiếp tam giác MNE cắt T là điểm cốđịnh, ta có đpcm

Bài tốn có nội dung tương tự với mở rộng 2, IMO 2009:

(22)

Một kết quen thuộc khác có từ tốn là: tiếp tuyến H đường tròn ngoại tiếp tam giác EMN cắt AB, AC hai điểm đối xứng qua H

Cách giải thứ túy đẹp mắt, thay việc chứng minh tam giác phép quay Trong trường hợp tam giác tù (tại B C), hình vẽ

và vịtrí điểm có nhiều thay đổi, sử dụng góc định hướng để có lời giải tốt hơn!

Lời giải bình luận thực Lê Phúc Lữ, dựa cách giải

Hoàng Đỗ Kiên, Phan Đức Minh, Lê Thanh Tú thân người bình luận

Bài 2.

Trên một cánh đồng hình ch nhật kích thước m n vng gm m hàng n ct, người ta đặt mt s máy bơm nước vào ô vuông Biết rng mỗi máy bơm nước có th tưới nước khơng nhng cho ô vuông cha ô vuông có chung cnh vi đó mà cịn có thtưới cho vng ct với cách một vng Tìm s nh nhất máy bơm nước cần đặt đểcác máy bơm có thểtưới hết c cánh đồng hai trường hp:

1) m4 2) m3

Lời giải

1) Với m4, ta chứng minh số máy bơm nước nhỏ thỏa mãn yêu cầu đề

bài n

Điều kiện đủ hiển nhiên với cách đặt cột máy bơm hàng thứ hai sau:

X X X … X X X

… …

Chú ý máy bơm tưới tối đa ô nên điều kiện cần rõ ràng với n1 n2

Ta chứng minh điều kiện cần phản chứng Giả sử tồn n cho cánh đồng kích

(23)

Xét cánh đồng kích thước 4n0 Theo định nghĩa n0, tồn cách xếp kn0

máy bơm để tưới hết cánh đồng Vì số máy bơm nhỏ số cột nên phải tồn cột khơng chứa máy bơm (ta gọi cột trống)

Bước 1. Ta thấy cột trống cột biên cột trống cột biên, chẳng hạn cột thứ để tưới cột trống, cột thứ hai phải chứa máy bơm Khi đó, cách thêm máy bơm vào cột hàng (nếu chưa có máy bơm), ta thấy

0

n  cột lại (bỏđi cột 2) sẽđược tưới k    4 k n02 máy bơm, mâu thuẫn với cách chọn n0

Bước 2. Vì cột trống khơng nằm biên, ta xét cột trống từ bên trái sang Ta giả

sử cột cột j Để tưới ô cột trống này, tổng cộng hai cột hai bên cột trống phải có máy bơm (*)

Xét trường hợp sau:

i) Cột j1 chứa máy bơm Khi cột từ1 đến j2 khơng trống nên j cột đầu chứa j máy bơm Suy n0 j cột sau chứa nhiều kj máy bơm Vì ngăn cách cột trống nên rõ ràng máy

bơm bơm cho tất cách đồng kích thước 4n0 j Vì

k j nj nên điều mâu thuẫn với cách chọn n0

ii) Cột j1 chứa máy bơm, đó, (*), cột j1 phải chứa

máy bơm Khi đó, j1 cột đầu chứa j    1 j máy bơm

nên n0  k j 2, tức bên cạnh cột j1 cịn cột Bây giờ, cách thêm vào cột hàng j2 máy bơm cần, ta thấy cánh đồng gồm n0 j 1 cột lại sau bỏ j1 cột đầu tưới

 2 1

k j    k j máy bơm Vì k  j n0 j 1 nên ta nhận

được mâu thuẫn với cách chọn n0

Như điều kiện cần chứng minh Ta có kết luận: Với cánh đồng 4n, cần n máy bơm để tưới nước thỏa mãn yêu cầu toán

2) Ta chứng minh số máy bơm để tưới cánh đồng 3n

4 n n  

 

  Trước hết ta chứng minh điều kiện đủ Với n5 điều kiện đủ hiển nhiên,

(24)

X X X X

Từđây dễ dàng cách đặt máy bơm cho n bất kỳ Chẳng hạn với n20, ta đặt 16

máy bơm sau

X X X X X X

X X X X X

X X X X X

Bây ta chứng minh điều kiện cần Chú ý máy bơm nước tưới tối đa ô nên với n1, 2, ta thấy cần phải có n máy bơm nước tưới tất cánh đồng 3n

Tương tự phần 1), ta chứng minh phương pháp phản chứng

Đặt ( ) , 1, 2,3,

n

f n  n    n

 

 

Giả sử tồn số nguyên dương n sao cho cánh đồng kích thước 3n tưới kf n  máy bơm nước Gọi n0 số nhỏ Theo ý n03 Do f n 0 n0 nên từđây ta suy kn0 Như phải có cột trống

Lý luận tương tựnhư phần 1, ta thấy cột trống không thểở biên Xét cột trống từ bên trái sang Giả sửđó cột j Khi đó, để tưới ô cột j, hai cột kề bên cột j phải chứa máy bơm nước

Xét trường hợp sau:

i) Cột j1 chứa máy bơm nước Khi j cột đầu chứa j máy bơm nước (do cột từ đến j2 chứa 1, cột j1 chứa 2) Suy n0 j cột cịn lại chứa khơng q kj máy bơm nước máy bơm tưới hết ô

cánh đồng kích thước 3n0 j

Ta có 0

0 0

1

( ) ( )

4

n n j

k j f n  j n  j   n  j     f nj

   

    nên từđây

(25)

ii) Cột j1 chứa máy bơm nước Khi cột j1 phải chứa máy

bơm nước Như j1 cột đầu chứa j1 máy bơm nước Suy n0 j 1 cột chứa nhiều k j 1 máy bơm nước Tiếp tục xét hai trường hợp:

Trường hợp 1 Cột j2 cột trống Khi n0 j 2 cột cịn lại sau bỏ j2 cột đầu tưới đủ nhiều k j 1 máy bơm nước Ta có

0

0 0

0

0

1

( 1) ( ) ( 1) ( 1) ( 2)

4

( 2)

( 2) ( ( 2))

4

n n

k j f n j n j n j

n j

n j f n j

                                                 

(do j2 nên j 2 4)

Điều mâu thuẫn với cách chọn n0

Trường hợp 2 Cột j2 có máy bơm Khi máy bơm từ cột j2 đến cột n0 tưới đủ ô cột (n0 j 1 cột) Theo tính tốn trên, sốmáy bơm

các cột không k j 1 Ta lại có đánh giá

 

0

0 0

0

1 ( 1)

( 1) ( ) ( 1) ( 1) ( 1)

4

( 1)

n n j

k j f n j n j n j

f n j

                                   

mâu thuẫn với cách chọn n0

Bài toán giải hồn tồn Vậy sốmáy bơm nhỏ để có thểtưới tất ô cánh đồng 3n

4 n n  

 

 

Bình luận

 Đây toán hay thú vị theo nghĩa để giải khơng cần kiến thức cao siêu đòi hỏi suy luận tinh tế Những toán mang

đậm chất IMO

(26)

Nhiều thí sinh TST số bạn ngồi có dựđốn sai sốmáy bơm

cần thiết n, từđó đưa lời giải sai

 Phương pháp chứng minh trình bày hai lời giải gọi phương

pháp phản ví dụ nhỏ nhất, nằm chủ đề Phương pháp chứng minh phản chứng chủđề Nguyên lý cực hạn

 Một cách khác để trình bày lời giải tốn dùng phép quy nạp tốn học  Bài có nét giống với VietnamTST 2010 có phần dễhơn

Bài

Cho s nguyên t p17 Chng minh rng t3 s nguyên dương lớn nht tha mãn điều kin: Vi s nguyên bt kì a b c d cho cho , , , abc không chia hết cho p a b c chia hết cho p tn ti s nguyên x y z thu, , c tp

0,1, , p t

   

 

    

 

    

 

  cho axbycz chia hết cho p Lời giải

Ta xửlí tốn theo bước sau:

1 Trước hết ta chứng minh với t3 ln tồn , ,x y z thỏa mãn toán

Đặt: 1, { | , , }

3 p

L   Saxbyczx y zL

 

  Yêu cầu toán tương đương với

việc chứng minh S chứa hệ thặng dư đầy đủ mod p

Kí hiệu ab ab chia hết cho p, ab a không đồng dư với b mod p, a1 số nghịch đảo a theo mod p, S số phần tử khác S theo mod p

2 Bởi a  b c nên S{axby (a b z) | 0x y z, , L} Việc nhân a b, với b10 không làm thay đổi số phần tử tập S theo mod p Do ta xem

1

b nên ta có S{ax  y (a 1) | 0zx y z, , L}

(27)

{ ( 1) | , , } {( 1) ( ) | , , }

Sax  y a zx y zL  S p a z y pa xx y zL

Do , ,x y z có thểđổi chỗ cho nên ta có vai trị a p a 1là nhau nên ta giả sử

2 p

a  Với p

a  kminL, 2La0

4 Với 0 l L, đặt Xl a z    lya 1z| 0 y L, 0  z L l

  

 | , 

l

Yax  y a xl   x L l  y L

0

L L

l l

l l

S X Y

 

   

 

   

     Ta có

   

 

 

 

– | , –

– | , –

– , – , ,

l

X a z l y a z y L z L l

y z al y L z L l

l L al l L al L al

       

     

     

là tập gồm 2L – l số nguyên liên tiếp Và tương tự,

                

– | ,

– – | ,

– – , – – 1, , –

l

Y ax y a x l x L l y L

y x a l x L l y L

l L a l l L a l L a l

        

      

     

là tập gồm 2L –l số nguyên liên tiếp Dễ thấy với l k minL, 2La

   

1 1

Lal     L l a lL a 1l     1 L (a1)l

nên L l l X

 chứa tất số nguyên từ L đến Lak L l l Y

 chứa tất số từ

 1

L k a k

    đến L

Do S chứa tất số từ  L ka1k đến Lak Dễ thấy a k, 1 nên

 ( 1)  2 2 2 2

Lak   L k ak   Lak  Lak   Lak

Mặt khác kminL, 2La nên:

Nếu kL 2L2a2k 1 2L2a2L 1 4L 1 p (Đây chỗ sử dụng

điều kiện p17)

(28)

Vậy S chứa khơng p số ngun liên tiếp nên ta có S chứa hệ thặng dư đầy đủ

theo mod p

5 Với t4 lấy a b 1,c 2 p

d  

1 1

2 2

4 2

p p p p p

x y z

        

   

           

   

     

Nên

2 2

p

x y zp

      đồng dư mod p, tức không tồn

, ,

x y z thỏa mãn

6 Kết hợp lý luận ta có đpcm

Bình luận

 Bài toán xứng đáng toán số kì thi Việc chứng minh cho trường

hợp t4 dễ dàng; nhiên, với trường hợp t3 vấn đề phức tạp

nhiều Với cách giải ta mở rộng toán sau: Bài toán 3.1

Cho số nguyên tố p Tìm số nguyên dương L nhỏ cho với ba số nguyên

không đồng dư với đôi theo mod p a b c chia hết cho p với d đều tồn 0x y z, , L cho axbycz chia hết cho p

 Quay trở lại với lời giải toán ban đầu

Mấu chốt vấn đề tận dụng tính chất: nhân ba số a b c, , với số

0

m theo mod p vai trị a b c, , ma mb mc, , tương đương

toán tương đương tốn Lại có a  b c 0(mod )p nên ta

thay c  a b giảm độ phức tạp Một suy nghĩ tự nhiên tìm m cho ma mb Điều đơn giản

Sau bước ta thu S ax y (a1) | 0zx y z, , L với

1 p L  

 

  Khi đến ta thử với a1

 | , ,   , 1, , 1, 

(29)

gồm số nguyên liên tiếp

Lại thử với a2 S2x y | 0zx y z, , L  3 , 3LL1, ,3L1,3L gồm số nguyên liên tiếp

Như qua hai trường hợp a1 a2 ta có ý tưởng chứngminh S chứa

một tập có dạng M, M 1, ,M 1,M

Chú ý tập yz| 0x y z, , L    L, L 1, ,L1,L nên ta nghĩ đến việc đặt x z đặt

   

 – | , –   – | , – 

l

Xa z l y az  y L  z L ly zal  y L  z L l tập gồm số nguyên liên tiếp Tương tự ta có Y1 đặt z x

Đểcó S chứa số ngun liên tiếp ta cần có X1 Xl1 phải giao nhau, ta thu điều kiện cần đủ a l 2L Vậy ta cần phải có a2L Kiểm tra với Y1 ta thu

đượcđiều kiệntương tự

Vấn đề lại phải có a2L Quay trở lại ta ý đến vai trò a (a 1) p (a 1)

     hai phảikhông vượt

2 p

L

 Các cách giải khác :

Hai cách giải sử dụng đến định lý thuộc lĩnh vực lý thuyết số cộng tính tổ hợp sau

Định lý Cauchy-Davenport

Cho hai tập số nguyên A, B số nguyên tố p Khơng có hai phần tử A đồng dư với theo modun p Tương tự cho B Khi tập Cab a| A b, B

chứa minAB 1,p phần tử đơi không đồng dư với mod p

Đặt A0,1, 2, ,L ta có SaAbAcA Theo định lý Cauchy-Davenport ta có

3

3 p SA    

 

 

Tuy nhiên, nói chung chưa phải điều ta cần, ta có thểvượt qua điều

(30)

Cách 1. (dựa theo bạn chemthan)

Đặt Baxbycz| 1x y z, ,  L 1 Caxbycz| 1 x y z, , 1 Áp dụngđịnh lý Cauchy-Davenport, ta có:

 

min 1,

SBCp B 3L 1

Do S min 3L – 5  C –1, pmin 3L – 6  C , p

Nếu ta chứng minh 3L 6 Cp ta chứng minh tốn cho t3

Ta có 11

3 p

L      p

 

  nên C phải 11 (ta cần chứng minh) Đặt Da b c, , ,  a, b, c

 , , , , ,   , , , , , 

Eab bc ca a b c b c a c a bab bc ca a b c dễ thấy DE 6 Nếu D E rời ta có CDE 12 thỏa mãn Nếu D E không rời tồn

- Trường hợp : a a b (không xảy ra)

- Trường hợp : a b c hay a    c b b b (không xảy ra)

- Trường hợp : a2b dễ dàng C 13 Vậy ta có C 11 ta có đpcm

Cách (Dựa cách với cách đặt tập C) a  b c nên với

2 L

K  ta có SX , với

 | , , 

Xaxbycz  K x y zK     C C C C (K phiên C) Áp dụng định lý Cauchy-Davenport K1 lần X minK C  K 1,p

Ta cần có K C   K p Ta đếm C Dễ thấy

0, , , , , , , , , 

Da b c  a b c ab bc ca chứa phần tửđôi không đồngdư

(31)

Ta có K C  K 9K1 Ta cần chứng minh 9K 1 p với  / 3

2 p

K    

  (*)

Đếnđây thử với p6l1 p 6l với l3 ta thấy (*)

 Lời giải bình luận thực bạn Lê Hồng Quý (Traum), huy

chương đồng Olympic Toán quốc tếnăm 2006

 Định lý Cauchy – Davenport có nhiều cách chứng minh khác nhau, có

cách chứng minh dựa vào định lý không điểm tổ hợp (Combinatorial Nullstenlenzat, viết Đa thức toán t hp đăng Tạp chí Tốn học Tuổi trẻ (6/2009)) Một cách chứng minh khác dùng đa thức nhiều biến : http://planetmath.org/encyclopedia/ProofofCauchyDavenportTheorem.html

Dưới chúng tơi trình bày cách chứng minh sơ cấp cho định lý Cảm ơn bạn Nguyễn Ngọc Trung (chemthan), huy chương vàng Olympic Toán quốc tếnăm 2010, cung cấp tư liệu tiếng Anh

Đặt Aa a1, 2, ,am, Bb b1, 2, ,bn Ta chứng minh kAB   m n (theo mod p) Ta giả sử mn chứng minh quy nạp theo m

Với m1, mệnh đề từ a1  bi a1 bj(mod )p ij nên

1 1

aBB    n n

Khi bất đẳng thức 0 F   m n G F, Gp giả thiết quy nạp cho ta mệnh đềđúng tập hợp F G

Giả sử mệnh đề với hai tập hợp X Y cho Xm X, Y XYp Giả sử A  m Bn, mn m n p Khi

np tồn cB.Ta chọn phần tử a a1, 2 khác thuộc A Vì dãy  1(mod )

ct aa p với t1, 2,3, ,p1 chứa tất số dư, trừ c nên  1

b c t aaB với t Gọi t số nhỏ có tính chất

Tập hợp A ba2A chứa phần tử ba2a b1, a2a2b Chú ý

  

2 1

ba    a c t aaBA  Bba2 A B nên ta cần

(32)

Đặt FAB FABbF b, a2 a1 Fba2 a1 A nên F tập thực khác rỗng A Như B tập thực G

Từđây suy 0 F   m n G

Mặt khác m n A  BA B ABFG

Ta ý F G AB (Với fF g, G, ta giả sử gA

khi f  F B suy f  g AB) Suy A  BFG

Khi bất đẳng thức 0 F   m n G F, Gp giả thiết quy nạp cho ta mệnh đềđúng tập hợp F G Do

1 1

ABA B FGFG   A  B   m n

và phép quy nạp hoàn tất

 Theo lời giải tốn cho p13

 Đây toán khó gần với tốn tổ hợp toán số

học (vềphương pháp chứng minh lập luận)

 Các bạn tham khảo tốn có ý tưởng giải tương tựnhư sau:

Bài toán 3.2. Cho p3 số nguyên tố a a a1, 2, 3, ,ap2 dãy số tự nhiên cho p không chia hết a k akk 1 Chứng minh ta chọn số số hạng của dãy sốđể tích chúng có sốdư chia cho p

Hướng dẫn.Dùng ngun thủy đưa tốn cộng tính

Bài

Cho dãy snguyên dương  xn được xác định bi *

2

1, 2011,

4022 ,

n n n

x x

xxx n

 

  

   

 

Chng minh rng 2012 2012

x

schính phương.

(33)

Đây tốn nhẹ nhàng quen thuộc đề thi lần Đểđơn giản

trong việc dùng kí hiệu hiểu rõ chất vấn đề, ta phát biểu chứng minh toán tổng quát sau:

Cho plà sốnguyên dương lẻ lớn

Xét dãy sốnguyên dương  xn được xác định

1

*

2

1, ,

2 ,

n n n

x x p

xpxx n

           

Chứng minh 1 p x

p  

sốchính phương. Bài tốn có số lời giải sau

Cách (dùng công thc tng quát ca dãy biến đổi trc tiếp)

Phương trình đặc trưng dãy sốđã cho 2

2

tpt tpt  có

1

p

   

nên phương trình có hai nghiệm 2

1 1,

tpptpp

Công thức tổng quát dãy cho xnAt1nBt n2n, 1, 2, 3,

Thay n1, tương ứng với hai số hạng cho trước dãy, ta hệphương trình sau:

2 2

1 At Bt

At Bt p

 

 

 

Giải hệnày, ta thu ,

2

t t

AB hay

1 1

, 1, 2,3, n n n t t x n     

Suy  

2 / /2 1 2

1 2( 1) 2( 1)

p p

p p

p t t

x t t

p p p

    

 

  

Chú ý t1t2 2 , p t t1 21 nên t1  t2  t1t22 t t1 2  2(p1)

Hơn nữa, ta có ,

n n n

Stt   n S S1, 2 Sn2 2pSn1Sn

Đặt t1 a, t2 b a b  2(p1),ab1 / / 2

p p p p

ttab

Ta có

1

1

0

( ) ( 1) 2( 1) ( 1)

p p

p p i i p i i i p i

i i

a b a b a b p a b

 

   

 

(34)

3

1

1

1 2 2

1

0

2

3

1 2

1

2

1 2 2 2 2

2

1

0

2

( 1) ( 1) ( 1) ( ) ( 1)

( 1) ( ) ( 1) ( ) ( 1) ( 1) ( 1) ( 1)

p

p p

p p

i i p i i i p i i i p i

p

i i i

p p

p p i p i p

p p

i i p i i i p i i i

p p

i i i i

a b a b ab a b

ab b ab a t t

                                                            3

1 2 1

2

2 2

1 2

0

( 1) ( 1) ( 1) ( 1)

p p

p i p i p p

i i

p i

i i

t t S N

                               

Do / /  / / 22 2 

1 2( 1) 2( 1)

p p p p

ttN p  ttN p

Vậy  

2

1 2( 1)

1 2( 1)

p

x N p

N

p p

  

 

  sốchính phương Ta có đpcm

Cách (Xét dãy s ph dùng quy np) Ta thấy

2

3

2

3

4 2

5

1

1,

1

1

2 1, (2 1) ,

1

1

8 1, 16 20 (4 1)

1

x p

x p

p p

x p p

x p x p p p

p p

x

x p p x p p p p p

p                                 

Từ cơng thức truy hồi xn2 2pxn1xn, ta có 2pxn1 xnxn2 Suy

  2

2 2 (4 2) 2,

n n n n n n n n n

x  p pxx xp xpx  xpxx   n

Ta xây dựng công thức dãy

, 1, 2, 3, n n x y n p   

 chứng minh số hạng

của dãy nguyên

Xét dãy  yn thỏa mãn

1

2

1,

,

n n n

y y p

yayby n

  

 

  

với a b, chọn sau Do y3 4p 2p1 nên

(2 1)

(35)

Dãy sốtương ứng

2

1,

2 ,

n n n

y y p

ypyy n

        

Ta chứng minh quy nạp 2

, 1 n n x y n p      (*)

Với n1, 2, khẳng định (*)

Giả sử ta có 2 2 1 , 1 n n n n x x y y p p        

Ta có 2

2 (2 ) 1(2 1) 1 ,

n n n n n n n n n n n n

yyy   py  y yypyy   yy  yn

Hơn

3 2

y yyp nên

2 2,

n n n

yyy   p n

Từ cơng thức xác định dãy

 

2 2 2 2 2

2 2 2 2 2

n n n n n n n n n n n n

yy   py   yy   y y   p y   yy   p y   p y  hay

2 2 2 2

2

2

2 2

1

(4 2) 4( 1) (4 2) 4( 1)

1

(4 2) 1

1

n n

n n n

n n n

x x

y p y y p p p

p p

p x x x

p p                           

Khẳng định (*) với n2 Theo nguyên lí quy nạp, (*) chứng minh

Do đó, ta chứng minh với n chẵn 1 n x

p

 sốchính phương; nói riêng, ta có 1

1 p x

p  

 sốchính phương Ta có đpcm

Bình luận

Cách thứ kết hợp thêm quy nạp để chứng minh / /2

1 2( 1)

p p

ttN p cho

đơn giản thay biến đổi trực tiếp

Cách thứ hai tốn dựa định lí dãy số gồm tồn sốchính phương sau:

(36)

Xét dãy số  vn với  

2 2

1 , , n ( 2) n n , va vb v   cv  vu uu n

Khi

n n

vu với sốnguyên dương n

Ý tưởng để chứng minh định lí hồn tồn áp dụng vào lời giải thứ hai Cũng cách cho ta kết mạnh toán cho khẳng định

đúng thay xp1 xn với n chẵn

Ngồi ra, ta dựa tính chất dãy số (xn) 2

2 1

n n n

xxx  p  để suy

2

2 1 1

1 1

n n n

x x x

p p p

                          

dùng quy nạp đểcó điều phải chứng minh

Ta có hướng tiếp cận khác là:

Xét phương trình Pell có dạng x2(p21)y21 với p sốnguyên dương lẻ

Ta xét đồng thời hai dãy sốđược xác định sau:

1 2

1 2

1, ,

, 1, 2,3,

0, 1,

n n n

n n n

x x p x px x

n

y y y py y

                

Khi đó, quy nạp, ta chứng minh xn2p21yn21 x yn, n

chính tất nghiệm phương trình Pell nêu

Bằng quy nạp, ta chứng minh x2k1  p1 , x2k 1  p1 với

k nên viết đẳng thức quan hệ x yn, n thành 2

1 k k x x y p p                       

Ta chứng minh hai số 2 , 1 k k x x p p  

  nguyên tố nên

sốđều phải số phương Bài tốn giải nhanh chóng có lẽđây sởđể xây dựng số này!

Từđây, ta có thểthêm vào tốn ban đầu kết thú vị 2012 2010

x

(37)

Ngồi ra, cơng thức xác định dãy cho có liên quan đến đa thức Chebyshev loại I

như sau:

0( ) 1, ( )1 , n 2( ) n 1( ) n( ), T xT xx TxxTxT x n

Do đó, dựa vào tính chất biết loại đa thức này, tốn cho phát triển theo nhiều hướng thú vịhơn

Lời giải bình luận thực Lê Phúc Lữ, tham khảo lời giải các bạn Lê Việt Hải, Võ Anh Đức Hoàng Đỗ Kiên, Nguyễn Huy Tùng

Bài

Chng minh rng C10 24 hng s ln nht cho nếu có 17 s thực dương 1, 2, , 17

a a a tha mãn điều kin a12a22  a172 24

17

3

17

1

aa aaa aC

thì vi mi 1   i j k 17, ta có a ai, j,ak là độ dài ba cnh ca mt tam giác

Lời giải

Đặt ai  24xi,  i 1, 2, , 17, u cầu tốn tương đương với:

Chứng minh C10 số lớn cho có 17 số thực dương

1, 2, , 17

x xx thỏa mãn x12x22  x172 1

3 3

1 17 17

24(xx   x )(xx   x )C

thì với i j k, , thỏa mãn 1   i j k 17, ta có x xi, j, xk là độ dài ba cạnh tam giác

Chứng minh gồm hai phần:

(a) Chứng minh số C10 thỏa mãn u cầu đề Khơng tính tổng quát, ta cần chứng minh x x1, 2, x3 độ dài ba cạnh tam giác Để ý với

0 t 1, ta có

3 2

24t  t (16t 9 )tt(1t)(4t1) 0

Do đó, từ giả thiết ta suy

4 4 2

1 17 17

(38)

hay

4 4 2 2

1 17 17

16(xx   x ) 1 (xx   x ) Bây giờ, sử dụng bất đẳng thức Cauchy-Schwarz, ta có

14 so

4 4 4 4 4

1 17 17

2

4 4 2

1 17

16( ) 1 ( ) ( )

2( )

x x x x x x x x x

x x x x x x

                                        Từđây kết hợp với trên, ta thu

4 4 2

1 3

2(xxx ) xxx , hay

4 4 2 2

1 3

2(xxx )(xxx )

Đến đây, cách sử dụng đồng thức

2 2 4

1 3 3 3

(xxx ) 2(xxx )2(xxx )(xxx )(x  x x)(x  x x ), ta dễdàng suy x x1, 2, x3 độ dài ba cạnh tam giác

(b) Chứng minh C10 số lớn thỏa mãn yêu cầu đề Giả sử tồn số C 10 thỏa mãn yêu cầu đềbài Khi đó, ta xét 17 sốdương x x1, 2,, x17 với

2 17

1

1 1

, , ,

4

1

, 16

6 x 14 16

xx   x x      

Lúc này, dễ thấy x12x22x172 1 Ngồi ra, ta có

2

2

1

1 1 1

4 16 16 16

x    x           x

nên x x1, 2, x3 độ dài ba cạnh tam giác

Bây giờ, ta chứng minh rằng, cách chọn thích hợp, số x x1, 2,, x17 thỏa mãn bất đẳng thức

3 3

1 17 17

24(xx   x )(xx   x )C

Rõ ràng chứng minh điều có nghĩa ta chứng minh C10 số lớn thỏa mãn yêu cầu toán

(39)

3 3 1

16 14

1 1

24 14 14

4 16 16 14 C

                                             

Dễ thấy vế trái hàm liên tục 0, 16

 

 

 

  Ngoài ra,

3

3

1 1 1

24 14 14 10

4 16 16 16 16

VT C                                  

Do đó, theo tính chất hàm liên tục, ta thấy tồn giá trị 0 0, 16   cho 0 VTC Lúc này, ta có số

2

0 0

1 17 0

1

( , , , ,

16

1 1

, ) , , ,

4 16 14 16 14

x

x x  

                 

thỏa mãn đồng thời điều kiện x12x22  x127 1

3 3

1 17 17

24(xx   x )(xx   x )C

Nhưng chúng có ba số x x1, 2, x3 khơng lập thành độ dài ba cạnh tam giác

Điều mâu thuẫn với giả thiết C 10 số thỏa mãn yêu cầu đề Vậy max 10

C

Bình luận

Lời giải ởtrên sử dụng đánh giá đểđưa xét bất đẳng thức

4 4 2 2

1 17 17 ,

16(xx   x )(xx   x )

rồi từđó suy x xi, j, xk (1   i j k 17) độ dài ba cạnh tam giác

Như vậy, từ việc giải toán cho, ta thu toán khác là:

Bài toán 5.1 Cho sốdương x x1, 2,, x17 thỏa mãn

2 2 4

1 17 17

(xx   x ) 16(xx   x )

Chứng minh rằng, với 1   i j k 16, ta có x xi, j, xk là độ dài ba cạnh tam giác

(40)

Bài toán 5.2 Cho số tự nhiên n3 a1, a2,,an số thực dương thỏa mãn 2

2

2

1

4

1

) ( 1)(

(aa annaa   an)

Chứng minh rằng, với 1   i j k n, ta có a ai, j,ak là độ dài ba cạnh tam giác

Cách giải tốn tổng qt hồn tốn giống với tốn 5.1, từ giả thiết, ta

tìm cách đánh giá đưa ba biến sau sử dụng khai triển quen thuộc

2 2 4

(abc ) 2(abc )(a b c b)(  c a c)(  a b a)(  b c)

đểsuy điều phải chứng minh

Qua lời giải trình bày trên, thấy đánh giá

3

24t  t 16t 9 ,t  t (0, 1)

chính mấu chốt, chìa khóa quan trọng để giải tốn Và giải tốn này, chúng tơi có cảm giác tác giả toán phát đánh giá

3

24t  t 16t 9t ,  t (0, 1) trước sau ngược từ tốn 5.2 để đến

tốn cho Rất có thểđây cách mà tác giảđã tạo toán Việc nhận toán gốc bất đẳng thức trung gian

3

24t  t 16t 9 ,t  t (0, 1)

là điều khơng dễ dàng gì, điều gây khó khăn cho khơng thí sinh

trong kỳ thi vừa qua Tuy nhiên, theo quan điểm cá nhân mình, chúng tơi nghĩ việc sử dụng “mẹo” che giấu đề làm khó tốn khơng nên Nên có tốn với ý tưởng tựnhiên

Lời giải bình luận thực Võ Quốc Bá Cẩn

Bài

Có 42 hc sinh tham d kì thi chọn đội tuyn Olympic tốn quc tế Biết rng mt hc sinh bt kì quen đúng 20 học sinh khác

Chng minh rng ta có th chia 42 hc sinh thành nhóm hoc 21 nhóm cho s hc sinh nhóm bng hc sinh bt kì nhóm quen

Lời giải

(41)

Bài tốn phát biểu dạng đồ thị cách hiển nhiên: đồ thị 2n2

đỉnh, bậc đỉnh n hợp rời đồ thị đầy đủ n1 đỉnh tìm n1 cạnh với đầu mút đơi rời

Ta xét bổ đề sau: Một đồ thị liên thông N đỉnh, bậc nhỏ d (bậc đỉnh

khơng d) ln có đường có độdài khơng min , d N1

Quay trở lại tốn, G khơng liên thơng hiển nhiên điều kiện bậc cho thấy G

được tạo thành từhai đồ thịđầy đủ n1đỉnh

Giả sử G liên thơng, theo bổđề trên, G có đường đicó độ dài 2n 2n1 Nói G có đường độ dài 2n1 nghĩa nói G có đường Hamilton (đi qua

2n2 đỉnh, đỉnh lần), trường hợp hiển nhiên cho ta lời giải toán

Giả sửG có đường độ dài 2n: A B A B1 1 2 2 A B An n n1 Gọi đỉnh lại An2 Nếu An2 kề Ai với i nào cạnh An2A A Bi, j j, ji B Aj j1, ji thỏa mãn toán Giả sử An2 khơng kề với Ai, bậc An2 n nên ta suy An2 kề

i

B với i

Lập luận với đường An2B A1 2 A B An n n1 (thay đổi vai trò A1 với An2) ta suy A1 kề Bi với i

Tương tự, suy luận với đường A B A1 1 n2B A B2 3 3 A B An n n1 (thay đổi vai trò A2 với

n

A ) ta suy A2 kề Bi với i

Bằng cách này, ta dễ dàng suy Aj kề Bi với i, j Thế đỉnh Bi có bậc n2, vơ lý

Ta có điều phải chứng minh

Để kết thúc phép chứng minh, ta chứng minh bổđề

Gọi G đồ thị có liên thơng có N đỉnh bậc đỉnh không nhỏhơn d Với X đồ thị

bất kỳ ta ký hiệu V X E X( ), ( ) tương ứng tập đỉnh, cạnh X

Trước hết ta có nhận xét sau:

Nếu P một đường G Nếu tồn phần tử xV G( ) \ ( )V P tồn ( ) \ ( )

(42)

Thật vậy, giả sử xV G( ) \ ( )V P vV P( ) Do tính liên thơng, tồn đường Q từ x

đến v Lấy đỉnh y đỉnh kềtrước v Q ta điều phải chứng minh

Bây gọi Pa b đường dài G Gọi k là độ dài P giả sử

 

min ,

kd N

Nếu abE G( ) yzPabPz* đường G dài P (đường từ y đến z, từ z theo P đến a, đến b, từ b theo P đến z* đỉnh kề với z P)

Nếu abE G( ), gọi KV P( ) tập đỉnh trước đỉnh kề a P (với P

được thứ tự từ a đến b) Vì tất cảcác đỉnh kề a phải nằm P, ta có Kdk2d, ta có V P( ) \K\bd Như tồn qK kề với b Bây ta xét chu trình CaPqbPq a* , yzCz* (theo chiều C) đường G dài

P, mâu thuẫn

Từ mâu thuẫn trên, ta suy kmin , d N1 Bổđềđược chứng minh

Cách 2. (Ca Traum@ da ý tưởng ca mnnn@ )

Nhắc lại cặp ghép (matching) hay tập cạnh độc lập của đồ thị G tập cạnh khơng có đỉnh chung Cặp ghép cực đại (maximal matching) của G cặp ghép mà mở rộng

1 Gọi S một cặp ghép cực đại G Nếu S 21 ta có điều phải chứng minh Nếu S 20, gọi V(S) tập đỉnh S

3 Gọi A B hai đỉnh khơng thuộc V(S) Vì S cặp ghép cực đại nên A B không k

4 Gọi N A N B( ), ( ) tập đỉnh kề với A, B V(S) tương ứng ta có

( ) ( ) 20

N AN B  , bậc đỉnh 20

5 Với C thuộc N(A), ta gọi đỉnh lại cạnh chứa C S C(S) Đặt M(A) tập đỉnh C(S) ta có M A( )  N A( ) 20

(43)

cặp ghép lớn S, điều mâu thuẫn với giả thiết lớn S Vậy M A( )N B( ) 0

7 Bởi M A( ) 20 N B( ) 20 suy V S( )  M A( )  N B( ) 40, mà ta có 40V S( ) nên V S( ) 40 hay S 20

8 Từ 7, ta thấy C kề với A mà C(S) khơng kề với A ta có C kề với B S(C) không kề với B

9 Gọi X(A) tập đỉnh C V(S) mà C C(S) kề với A Tương tựta có định nghĩa cho X(B) Gọi Y tập đỉnh kề với A B Z đỉnh không kề với A B Theo nhận xét 6, 7, 8, ta có YZ X A( )  X B( ) X A( ) Y 20 10 Ta có điều sau:

a Các đỉnh thuộc X(A) kề với đỉnh thuộc X(A) Y

b Các đỉnh thuộc X(B) kề với đỉnh thuộc X(B) Y c Các đỉnh thuộc Z kề với đỉnh thuộc Y

11 Các đỉnh thuộc Z kề với đỉnh thuộc Y Vì đỉnh thuộc Z kề với đỉnh thuộc Y nên ta có 20Z 18Y , mà YZ nên YZ 0

Vậy ta có X A( )  X B( ) 20 Mà theo 10 đỉnh thuộc X(A) kề với A

đỉnh thuộc X(A) Nên ta có 21 đỉnh gồm A và đỉnh X(A) tạo thành K21

Tương tự ta có K21 tương ứng với B

12 Bài tốn chứng minh xong

Bình luận

 Đây toán tồn cặp ghép hoàn hảo (perfect matching)

đồ thị có bậc chẵn Vì xu hướng sử dụng suy luận liên quan đến cặp ghép rõ ràng

 Lời giải định hướng điều kiện “Dirac” đề Chú ý theo phiên sở định lý Dirac đồ thị n đỉnh có bậc đỉnh không nhỏ

2 n

(44)

nhiên, sử dụng định lý Dirac khơng giúp ích Ở đây, ta cần đến biến thể định lý Dirac, bổđề lời giải Bổđể thực chất tập tài liệu lý thuyết Graph Chứng minh nêu bổđềđược tham khảo đây:

http://www.mpi-inf.mpg.de/departments/d1/teaching/ss11/graph_theory /exercises/Graph_Theory_sol_02.pdf

 Ngoài cách phát biểu định lý Dirac ngun thủy, ta cịn có cách phát biểu khác cho định lý Dirac, tổng quát gần với bổđề:

Định lý (Dirac). Cho G đồ thị 2-liên thông với bậc đỉnh khơng nhỏhơn d Khi G chứa chu trình độ dài min , d V G( )

Có thể tham khảo chứng minh định lý tại: http://math.fau.edu/locke/Dirac.htm

 Lời giải tự nhiên, theo hướng chứng minh khơng tồn cặp ghép hồn chỉnh G phải hợp phiên K21 rời Cũng lời giải 1, việc sử dụng nguyên lý cực hạn (cặp ghép cực đại) đem lại thông tin bổ sung quan trọng (A, B thuộc V G( ) \S không kề nhau) dẫn đến lời giải tốn  Ngồi hai cách giải cịn cách giải khác sử dụng đến kết cặp

ghép hoàn hảo định lý Tutte sau:

Định lý Tutte. Đồ thị G = (V, E) có cặp ghép hồn hảo với tập con U V, số thành phần liên thơng có sốđỉnh lẻtrong đồ thị cảm sinh

\

V U nhỏhơn hay U

Chứng minh định lý Tutte tham khảo tại: http://en.wikipedia.org/wiki/Tutte_theorem

Lời giải sử dụng định lý Tutte tham khảo tại:

http://forum.mathscope.org/showthread.php?t=30511&page=3

 Đây tốn khó thí sinh khơng quen thuộc với lý thuyết đồ thị,

nhưng bạn biết cặp ghép hoàn hảo định lý Dirac (đặc biệt

phương pháp suy luận đường dài – longest path argument)

khơng q khó Đây điểm khiến chúng tơi đánh giá tính phân loại khơng cao 2, thế, tạo nhiều lợi cho bạn “biết

(45)

1 LỜI GIẢI VÀ BÌNH LUẬN

ĐỀ CHỌN ĐỘI TUYỂN QUỐC GIA DỰ THI IMO NĂM 2013

Những người thực hiện: Trần Nam Dũng

Trần Quang Hùng

Võ Quốc Bá Cẩn Lê Phúc Lữ

Xin chân thành cảm ơn thầy Nguyễn Tăng Vũ, thầy Trần Quốc Luật, bạn Nguyễn Văn Quý, Võ Anh Đức, Hoàng Đỗ Kiên, Phạm Tuấn Huy, Nguyễn Huy Tùng nhiều thành viên diễn đàn đóng góp ý kiến để chúng tơi hồn tất tài liệu này!

(46)

2 Phần

ĐỀ THI CHÍNH THỨC Ngày thi thứ

Bài

Cho tứ giác ABCD có cạnh đối khơng song song nội tiếp đường trịn ( , )O R Gọi E giao điểm hai đường chéo đường phân giác góc AEB cắt đường thẳng

, , ,

AB BC CD DA điểm M N P Q , , ,

1 Chứng minh đường tròn (AQM),(BMN),(CNP),(DPQ qua ) điểm Gọi điểm K

2 Đặt minAC BD, m Chứng minh

2

2

2

R OK

R m

Bài

1 Chứng minh tồn vô số số nguyên dương t cho 2012t1 2013t1 số phương

2 Xét ,m n số nguyên dương cho mn1 (m1)n1 số phương Chứng minh n chia hết cho 8(2m1)

Bài

Với số n nguyên dương, đặt Sn0,1, 2, , 2n1 Xét hàm số f : (Sn)[0;1]

thỏa mãn đồng thời điều kiện sau:

i/ f x( ,0) f x n( , 1) 0 với số nguyên x

ii/ f x( 1, )yf x( 1, )yf x y( , 1) f x y( , 1) 1 với x y,  1y2n Gọi F tập hợp tất hàm số f thỏa mãn

1 Chứng minh F vô hạn

(47)

3

Ngày thi thứ hai

Bài

Tìm số k nguyên dương lớn thỏa mãn: Với a b c dương mà , , abc1 ta có bất đẳng thức sau

1 1

3

k k

abca b c    

Bài

Cho tam giác ABC nhọn khơng cân có A 45 Các đường cao AD BE CF đồng , , quy trực tâm H Đường thẳng EF cắt đường thẳng BC P Gọi I trung điểm BC; đường thẳng IF cắt PH Q

1 Chứng minh IQH  AIE

2 Gọi K trực tâm tam giác AEF ( )J đường tròn ngoại tiếp tam giác KPD Đường thẳng CK cắt ( )J G, đường thẳng IG cắt ( )J M, đường thẳng JC cắt đường trịn đường kính BCtại N Chứng minh điểm G M N C thuộc , , , đường tròn

Bài

(48)

4 Phần

LỜI GIẢI CHI TIẾT VÀ BÌNH LUẬN

Bài

Cho tứ giác ABCD lồi có cạnh đối khơng song song nội tiếp đường tròn ( , )O R Gọi E giao điểm hai đường chéo đường phân giác góc AEB cắt đường thẳng

, , ,

AB BC CD DA điểm M N P Q , , ,

1 Chứng minh đường tròn (AQM),(BMN),(CNP),(DPQ qua điểm ) duy Gọi điểm K

2 Đặt minAC BD, m Chứng minh

2

2

2

R OK

R m

Lời giải

R

S K

P M

Q

N

E O

A

B

C D

(49)

5 A S nằm ,, C R hình vẽ Gọi K giao điểm đường tròn ngoại tiếp tam giác RAB SBC ,

180

BKR BKS BAD BCD

         hay R K S thẳng hàng , ,

Suy RK RS RB RC RA RDSK SR SB SA SC SD     nên tứ giác ADSK

CDRK nội tiếp hay K thuộc đường tròn (RCD () SDA )

Do đó, ta có

AKD ASD BSC BKC

       ADK ASK BSK BCK

nên tam giác KAD KBC đồng dạng Suy KA AD AE AM

KBBCBEBM , theo tính

chất đường phân giác KM phân giác góc AKB

Mặt khác, ta có RNQ BNE CBD BEN CAD AEQ RQN nên ta

2

ARB BNM

  

Từ suy 1

2

BKM AKB ARB BNM

       hay tứ giác BMNK nội tiếp, tức

K thuộc đường tròn (BMN )

Chứng minh tương tự, ta có K thuộc đường tròn (AQM),(CNP),(DPQ )

Tiếp theo, ta chứng minh K điểm chung đường tròn Thật vậy, đường trịn (AMP),(BMQ có hai điểm chung ) K M cịn đường trịn , (DNP CNP có hai điểm chung ),( ) K N Do đó, bốn đường trịn có hai điểm , chung M N trùng nhau, vơ lí ,

Ta có đpcm

2 Theo tính chất phương tích

2 2

,

RK RS RB RC   ROR SK SR SB SA SO    R nên

2 2

ROSORK RS SK SR RK    SK Từ suy OKRS

Hơn nữa, theo định lí Brocard tứ giác nội tiếp ABCD E trực tâm tam giác ORS, suy OERS

(50)

6 Ta lại có RKA SKC RBA SBC 2 ADC AOC nên AKC AOC180

hay tứ giác AOCK nội tiếp Suy

2 ( )

EO EK EA EC ROEEO EO EK R hay

2 R OK

EO  Mặt khác, theo bất đẳng thức đường xiên đường vng góc

  2 2 2

max ( , ), ( , ) max , 4

2 2

EOd O AC d O BC   RAC RBD  Rm

 

Vậy ta

2

2

2

R OK

R m

Đây đpcm

Nhận xét

Lời giải toán dựa hai định lý hình học điểm Miquel tứ giác tồn phần định lý Brocard

Nói đến định lý Miquel nhiều người quen thuộc dạng phát biểu sau

Định lý Miquel Cho tam giác ABCvà điểm D E F thuộc đường thẳng , , , ,

BC CA AB Khi đường trịn ngoại tiếp tam giác (AEF),(BFD CDE có ),( ) điểm chung M gọi điểm Miquel

Hệ Các điểm D E F thẳng hàng điểm Miquel , , M thuộc đường tròn (ABC )

Hệ có ý nghĩa ta coi tam giác đường thẳng tứ giác tồn phần ta phát biểu lại hệ sau

Hệ Cho tứ giác ABCDAB giao CD E, AD giao BC F Khi đường trịn (EAD),(EBC),(FAB),(FCD có điểm chung M Điểm ) M gọi điểm Miquel tứ giác ABCD

Điểm Miquel M đặc biệt có nhiều tính chất thú vị tứ giác ABCD nội tiếp Một tính chất quan trọng sau

Hệ Cho tứ giác ABCD nội tiếp đường tròn ( )O Giả sử AB giao CD E, AD

giao BC F, AC giao BD G Khi đường trịn (EAD),(EBC),(FAB),(FCD ) có điểm chung M O G M thẳng hàng , ,

(51)

7 Định lý Brocard Cho tứ giác ABCD nội tiếp đường tròn ( )O Giả sử AB giao CD E,

AD giao BC F, AC giao BD G Khi O trực tâm tam giác EFG

Một toán ứng dụng hay định lý thi quốc gia Việt Nam năm 2012 vừa qua

Chứng minh định lý Miquel hệ coi thể quan trọng việc phải sử dụng góc định hướng hình học Ở này, khơng dùng độ dài đại số góc định hướng, ta phải quy ước vị trí điểm trình bày để lời giải chặt chẽ

Bài thi ý tưởng hay cho việc kết hợp định lý Miquel định lý Brocard đặc biệt câu 2) Ý tưởng câu có lẽ bắt nguồn từ đẳng thức

2 OK OE OR OS   R

 

; nhiên, để giấu diện điểm E thi đưa bất đẳng thức liên hệ đường xiên hình chiếu,

   

2

2 ,

max ( , ), ( , )

4

AC BD OEd O AC d O BDR

Thực ý tưởng muốn giấu điểm E hay xong việc phải dùng đến bất đẳng thức hình học khiến cho tốn nhiều vẻ đẹp

Bài tốn giải cách sử dụng bổ đề sau liên quan đến phép biến sau:

Cho hai đoạn thẳng AB CD cho , ABCD hình thang Khi đó, tồn phép vị tự quay tâm O biến AB thành CD Nếu P giao điểm AB CD ,

Q giao điểm AD BC tứ giác ADPK BCPK ABQK CDQK nội tiếp , , ,

Trong toán cho, điểm K tâm phép vị tự quay

Ý tưởng sử dụng bất đẳng thức giống với hình số đề thi TST 2006, so sánh đường xiên đường vng góc Tuy nhiên, ta khơng cần với trường hợp đẳng thức xảy

Nếu tổng quát lên, thay đường phân giác góc E thành đường thẳng qua E, ta có tốn sau:

(52)

8 thẳng AB BC CD DA điểm , , , M N P Q Chứng minh giao điểm , , , khác M N P Q cặp đường tròn (, , , AQM),(BMN ; () BMN),(CNP ; () CNP),(DPQ ) và (DPQ),(AQM thuộc đường tròn qua điểm Miquel ) K tứ giác ABCD

K

N P

M Q

R

S E

O A

B

C

D

Trong trường hợp đường thẳng trở thành phân giác bốn giao điểm trùng trùng với K

Bài

1 Chứng minh tồn vô số số nguyên dương t cho 2012t1 2013t1 là số phương

2 Xét ,m n số nguyên dương cho mn1 (m1)n1 số phương Chứng minh n chia hết cho 8(2m1)

(53)

9 Đặt d(2012t1, 2013t1) dễ thấy d1 Do đó, 2012t1 2013t1 số phương (2012t1)(2013t1)y2 với y số nguyên dương Ta biến đổi đẳng thức

2

2 2

2

(2012 1)(2013 1)

4 2012 2013 2012 2013 4025 2012 2013 2012 2013 (2 2012 2013 4025) 2012 2013

t t y

t t y

t y

  

             

        

Đặt x 2 2012 2013 t4025 ta có phương trình x2  4 2012 2013 y2 1

Dễ thấy 2012 2013  khơng phải số phương nên phương trình Pell loại có vơ số nghiệm Nghiệm nhỏ phương trình ( , ) (4025,1)x y  nên nghiệm cho cơng thức

0

0

1, 4025, 8050

, 1, 1, 8050

n n n

n n n

x x x x x

n

y y y y y

               

Bằng quy nạp, ta chứng minh x2 1i chia 2012 2013  dư 4025 với i

giá trị nguyên dương 4025 2012 2013

i

x  

  cho ta giá trị t thỏa mãn đề

Vậy tồn vô số giá trị nguyên dương tsao cho 2012t1 2013t1 số phương Ta có đpcm

2 Đặt d(mn1,mn n 1)

 1

d mn n  mn hay d n , suy (d mn 1 mn) hayd 1 Do d1 hay số mn1,(m1)n1 nguyên tố

Khi đó, mn1 (m1)n1 số phương

 

(mn1) (m1)n1 số phương Giả sử (mn1) ( m1)n1y2 với y

 Biến đổi biểu thức này, ta thu

 

2

2 2

2 2

( 1) (2 1)

4 ( 1) ( 1)(2 1) ( 1) ( 1) ( 1) (2 1) ( 1)

m m n m n y

m m n m m m n m m m m y

m m n m m m y

    

        

(54)

10 Đặt x2 (m m1)n(2m1) ta có phương trình sau

2

4 ( 1)

xm my  (*)

Đây phương trình Pell loại (m m1) khơng số phương với m ngun dương nên (*) có vơ số nghiệm

Phương trình (*) có nghiệm nhỏ ( , ) (2x ym1,1) nên công thức nghiệm ( , )x yi i

của viết dạng

0

0

1, 1, 2(2 1)

, 0, 1, 2(2 1)

i i i

i i i

x x m x m x x

i

y y y m y y

                  

Bằng quy nạp, ta chứng minh x2i chia (m m1) dư x2 1i chia (m m1) dư

2m1 với i0,1, 2, (**)

Thật vậy,

- Với i0, theo cơng thức truy hồi dãy ( )xi ta thấy khẳng định (**)

- Giả sử (**) đến i, tức x2i chia (m m1) dư x2 1i chia (m m1) dư 2m1 Ta có

2i 2(2 1) 2i 2i 2(2 1)(2 1) ( 1) 1 (mod ( 1))

x   mx  xmm   m m   m m

2i 2(2 1) 2i 2i 2(2 1) (2 1) (mod ( 1))

x   mx  x   m  m  mm m

Do đó, (**) với i1

Theo nguyên lí quy nạp, (**) với số tự nhiên i

Tiếp theo, ta xây dựng công thức truy hồi cho dãy rix2i1 với i0,1, 2,

Ta có

 

   

   

2 3 2

2

2 2 3

2

2 1

2(2 1) 2(2 1) 2(2 1)

4(2 1) 2(2 1) 4(2 1) ( ) 4(2 1) 4(2 1)

i i i i i i i

i i i i i

i i i i

r x m x x m m x x x

m x m x m x x x

m x x m r r

                                         

Đặt ri 2 (m m1)si (2m1) dãy ( ),s ii 0 nguyên dương xác định

(55)

11            2 2 2

2 ( 1) (2 1) 4(2 1) 2 ( 1) (2 1) ( 1) (2 1) ( 1) ( 1) 4(2 1) 2 ( 1) 4(2 1) (2 1)

4(2 1) 8(2 1)

i i i

i i i

i i i

m m s m m m m s m m m s m

m m s m m m s m m s m m

s m s s m

                                     

Ta tính r0 x12m1 nên s0 0

 

1 2(2 1) 2(2 1) (2 1) 16 ( 1)(2 1)

rxmm   m  m mm  m nên s18(2m1)

Ta có công thức truy hồi dãy số ( )si

 

0

2

2

0, 8(2 1),

4(2 1) 8(2 1),

i i i

s s m

sm ss m i

              

Từ suy dãy số ( )si có số hạng chia hết cho 8(2m1) với i0,1, 2,

Hơn nữa, với cách đặt x2 (m m1)n(2m1) dễ dàng thấy n thỏa mãn đề n s ii, 1, 2, 3, (do s0 0 số nguyên dương)

Vậy tất giá trị n chia hết cho 8(2m1) Đây đpcm

Nhận xét

Ý thứ toán thực trường hợp đặc biệt dẫn dắt cho ý thứ Ta giải ý theo cách tính toán nhận xét:

Nếu x y, 1 nghiệm phương trình 2013x2 2012y2 1 ty2x2* số 2012t1, 2013t1 phương

Hơn nữa, nghiệm ( , )x y phương trình x2 2012 2013 y2 1 cho ta nghiệm (x2012 ,y x2013 )y phương trình 2013x22012y2 1 nên dễ thấy đpcm

Cách tiếp cận phương trình Pell hồn tồn tự nhiên xây dựng thành cơng ta cịn cần thao tác dãy số nguyên Trong năm gần đây, toán dãy số nguyên ưa chuộng, đặc biệt dãy số nguyên liên quan đến cơng thức nghiệm phương trình nghiệm ngun Chẳng hạn

(Việt Nam TST 2012) Cho dãy số nguyên dương ( )xn xác định

1

2

1, 2011,

4022 , 1, 2, 3,

n n n

x x

xxx n

  

  

(56)

12 Chứng minh 2012

2012

x

số phương

Trong việc giải ý thứ 2, ta lập luận sau:

Do mn1 (m1)n1 số phương nên ta đặt

2

1 ,( 1)

mn a mn b

Từ ta có phương trình (m1)a2mb2 1 Đổi biến a x my b x  ,  (m1)y để đưa phương trình Pell liên kết 2

( 1)

xm my

Cách giải hoàn toàn phù hợp ý phương trình Pell tổng qt dạng Ax2By2 n nói chung có nhiều nghiệm sở Ta cần chứng minh trường hợp đặc biệt (AB1 n1) có nghiệm sở Nếu khơng lời giải thực cịn thiếu sót

Ngồi cách sử dụng phương trình Pell trên, ta lập luận sau:

Do (mm1,mn n 1) 1 nên ta cần có (mn1)(mn n 1) số phương hay tồn a cho (m m1)n2 4(2m1)n4a2, đưa

 2 2

(2m1)n2 na

Ta chứng minh n phải số chẵn phương trình Pythagore nên phải tồn số p q k cho , ,

2

2 ,

(2 1) ( ) n kpq

m n k p q

   

   

 

trong k n( ,(2m1)n2) 2 nên k1 k2

Nếu k1 dễ thấy p2q2 chẵn, suy p q, tính chẵn lẻ Nếu p q, lẻ n chia dư (2m1)n2 chia hết cho p2q2 chia dư 2, mâu thuẫn Nếu

,

p q chẵn n chia hết cho dẫn đến (2m1)n2 chia dư p2q2 chia hết cho 4, mâu thuẫn

Nếu k2 n4pq (2m1)n2 2( p2q2), suy

2

2(2 1)

(57)

13 Đến ta chứng minh (sử dụng phương pháp Viète Jumping)p q, hai số hạng liên tiếp dãy số cho công thức

1

2

1, 2(2 1),

2(2 1) ,

i i i

x x m

xm xx i

   

   

Từ suy pq chia hết cho 2(2m1) hay n chia hết cho 8(2m1)

Phương pháp xây dựng nghiệm có nét giống với đề VMO 2012 Xét số tự nhiên lẻ a b mà a ước số ,

2

b b ước số 2

a Chứng minh a b số hạng dãy số tự nhiên , ( )vn xác định

1

vv vn4vn1vn2 với n2

Cách giải có phần ngắn gọn xử lí nhẹ nhàng cách nêu lời giải ban đầu để trình bày chặt chẽ điều đơn giản

Bài

Với số n nguyên dương, đặt Sn 0,1, 2, , 2n1 Xét hàm số f : (Sn)[0;1]

thỏa mãn đồng thời điều kiện sau:

i/ f x( ,0) f x n( , 1) 0 với số nguyên x

ii/ f x( 1, )yf x( 1, )yf x y( , 1) f x y( , 1) 1 với x y,  1y2n Gọi F tập hợp tất hàm số f thỏa mãn

1 Chứng minh F vô hạn

2 Với hàm số fF, đặt v tập hợp ảnh f Chứng minh f v hữu hạn f 3 Tìm giá trị lớn v với ffF

Lời giải

1 Trong đẳng thức ii/ cho, ta thấy

(58)

14 Điều có nghĩa giá trị hàm f x y với ( , ) x y, tính chẵn lẻ x y, khác tính chẵn lẻ khơng có liên hệ với

Ta tìm xác định hàm f hai trường hợp

Trong mặt phẳng tọa độ Oxy ta xét lưới nguyên nằm ngang có chiều cao , 2n1, chiều rộng vơ hạn điểm có tọa độ ( , )i j gán giá trị ( , )f i j

Điều kiện i/ hiểu tất số thuộc hai biên (trên dưới) lưới gán số 0; điểm nguyên bên lưới gán giá trị thuộc [0; 1]

Điều kiện ii/ với hình vng nằm nghiêng (các đường chéo song song với trục tọa độ) có đỉnh nguyên cạnh có tổng số gán cho đỉnh

Với điểm có tọa độ A nguyên thuộc lưới nguyên xét, ta đặt f A f A1( ), ( )2 giá trị gán cho điểm có tung độ với A, có hồnh độ lớn nhỏ hoành độ A đơn vị

Tiếp tục đặt akf k k( , ) với k1, 2, 3, , 2n giá trị gán cho điểm A k kk( , ) Theo điều kiện xác định hàm số

1 1 1

2 1

2 2 2 1 1

2 1 2

( ) ( ) , ( ) ( ) ( ) ,

( ) ( ) ( ) ,

0 ( ) ( ) ( )

n n n n n n

n n n n

a a f A f A a a

a a f A f A f A a a

a a f A f A f A a a

a f A f A f A a

                                

Tương tự, ta có

2 2

2 2 2

2 1

2 2

( ) ( )

( ) ,

( )

n n n

n n n

n n

n n

f A a a

f A a a

f A a a

f A a

            

Từ ta thấy dãy ak xác định giá trị f A1( k), (f A2 k) hoàn toàn xác định Ta chọn giá trị ak cho f A1( k), (f A2 k) thuộc [0; 1]

(59)

15 Hơn nữa, ta thấy

1 1 1 1

2 2 2 2 2

( ) ( ) ( ) ( ), ( ) ( ) ( ) ( ), ( ) ( ) 1, ( ) ( ) ( ) ( ), ( ) ( ) ( ) ( ), ( ) ( )

n n n

n n n

f A f A f A f A f A f A f A f A f A f A f A f A f A f A f A f A f A f A f A f A

         

         

Do đó, tính đơn điệu hai dãy có số chẵn lẻ bảo toàn

Tương tự, từ dãy f A1( k), ta xác định f f A1( (1 k)) thỏa mãn, từ dãy f A2( k), ta xác định f f A2( (2 k)) thế, tức xây dựng tất giá trị cho hàm số

( , )

f x y trường hợp x y chẵn

Tiếp tục đặt bkf k( 1, )k với k1, 2, 3, , 2n giá trị gán cho điểm B kk( 1, )k hồn toàn tương tự trên, ta xây dựng tất giá trị f x y với ( , ) x y lẻ

Dễ thấy có vơ số cách chọn dãy ( ),( )ak bk thỏa mãn điều kiện nên có vơ số hàm f thỏa mãn đề bài, tức F vô hạn Ta có đpcm

2 Trong đẳng thức f x( 1, )yf x( 1, )yf x y( , 1) f x y( , 1) 1 , ta thay x y, x1,y1, ta f x y( , 1) f x( 2,y1) f x( 1, )yf x( 1,y2) 1

Từ suy f x( 1, )yf x y( , 1) f x( 1,y2) f x( 2,y1) (*) hay (1,1) (2,0) (3, 3) (4, 2) (2 1, 1) (2 2, ) ffff   f nn  f nn ,

(60)

16 Tương tự, ta có f(2, 2) f(2n2, 2n1), (4, 2)ff(2n5, 2n1)

Tiếp tục áp dụng nhiều lần đẳng thức (*), suy

( , ) (2 ,2 ), ( 2, ) (2 , ) f k kf n k n k f kkf n k n k Chứng minh tương tự, ta có

(2 , ) (4 , ), (2 , ) (4 , ) f n k n  kf n k k f n k n kf n k k

Do đó, f k k( , ) f(2n 1 k n,  1 k) f(4n 2 k k, ) với k1, 2, 3, , 2n thế, quy nạp, ta có

( , ) ((2 1) , ) ((4 2) , )

f k kf ni k n  kf ni k k với i k1, 2, 3, , 2n Hơn nữa, xác định giá trị đường chéo (nằm phía phải) hồn tồn giống dải điểm ( , )k k ((2n1)i k n ,  1 k) nên giá trị tương ứng Suy giá trị f có tuần hồn minh họa hình bên dưới, tức giá trị f x y với ( , ) x y chẵn lặp lại giá trị gán cho điểm nguyên tam giác  có tọa độ đỉnh (1,1),(2n1, ),(4n n1,1)

1 2n+1 2n

O 1 2n+1 2n+2 4n+1 4n+2 4n+3 4n+3 6n+3

Tuy nhiên, giá trị hữu hạn ta tính tổng cộng có khơng q 2    n n n (2 1) giá trị

Tương tự với giá trị f x y mà ( , ) x y lẻ, ta có thêm khơng q n n(2 1) giá trị Kết hợp với 0, ta có vf 2 (2n n1) 1

(61)

17 Ta xây dựng hàm số f có vf 2 (2n n1) 1 giá trị lớn v f

Bằng quy nạp, ta chứng minh

1 ( 1) 1 ( 1) 

( , ) ( 1) ( 1)

4

i k

k k i

k i k

f i k i aa

   

     

với i1, 2, , 2n k mà 0  k i 2n1 (**) Thật vậy, với k0 (**) hiển nhiên

Giả sử (**) với ( , )i k mà km km1,ij Từ điều kiện ii/, ta có

( 2( 1), 1) ( , 1) ( 2( 1), ) ( 2 , 2) f j  mj  f j  m j  f jmjf j  m j  Sử dụng giả thiết quy nạp, ta tính

 1 1

2

2

1 ( 1) ( 1)

( 2( 1), 1) ( 1) ( 1)

4

j m

m j m

m j m

f j m j a a

                    

Suy (**) với i j 1,km1

Như thế, giá trị gán cho điểm tam giác  có dạng ijaiaj với  0,1

ij

 , giá trị ij dấu a a xác định theo ,i, j i j Tiếp theo, ta chọn 2 1 21 1 , 2 2(11 )

3

k k k n k

a    a    với k1, 2, ,n hệ số 3,

số ngun có cách biểu diễn dạng tổng r i i i

 với r 0  1,0,1

i

e   ) nên suy tất giá trị f x y dùng để đánh số cho điểm nằm ( , ) tam giác  đơi phân biệt khác 0, tức có n n(2 1) giá trị thế, tức ta xây dựng giá trị cho hàm số f x y với ( , ) x y chẵn

Tương tự, để xây dựng cho f x y với ( , ) x y lẻ, ta chọn dãy số

2 2 1 2( 1 )

1

,

3 3

k k k n k

b    b    với k1, 2, , 2n

(62)

18 Từ đó, ta xây dựng toàn giá trị cho hàm số f x y có tất ( , ) (2n n1) 1 giá trị đôi khác

Vậy giá trị lớn cần tìm v miền f F (2n n1) 1

Nhận xét

Đây tốn địi hỏi kiến thức tổng hợp phong cách làm việc từ tốn, Mơ hình lưới bảng mơ hình tự nhiên mà ta nghĩ đến, hàm số xác định theo hàng, cột đường chéo

Trong toán có chứa tham số n , ta nên giá trị n nhỏ để hình dung toán cách tốt nhất, cụ thể nhất, từ tìm cách tiếp cận tổng qt Nếu chọn cách có lẽ câu 1) 2), chí phần chặn câu 3) khó Điểm khó câu 1) cách xây dựng hàm phải thỏa mãn điều kiện

( , ) [0; 1]

f x y  Để có điều này, ta phải tìm điều kiện có tính bất biến dãy giá trị đường chéo (khơng đề dãy kề thuộc [0; 1] , mà để dãy thế)

Để làm câu 3), ta phải có nhận xét tinh tế hơn, địi hỏi nhiều thời gian Có lẽ câu dành cho bạn giải tốt hai 1, vòng 1,5 - giờ, nhiều thời gian để tập trung toàn lực cho Ý tưởng dự đoán chứng minh công thức tổng quát hàm dựa theo phần tử thuộc đường chéo Từ cơng thức tổng qt ta tìm cách chọn “giá trị khởi tạo” để tất giá trị nằm “tam giác tuần hồn” đơi khác

Để chọn giá trị khác nhau, ta có số định hướng sau: + Dùng bất đẳng thức

+ Dùng tính “độc lập tuyến tính” số vơ tỷ, cụ thể là: Nếu p p1, 2, ,pn số nguyên tố phân biệt khơng tồn số hữu tỉ c c1, , ,2 cn không đồng thời cho c1 p1 c2 p2  cn pn 0

Tuy nhiên, việc chứng minh bổ đề không đơn giản Ta thay số

i

(63)

19 Một cách tiếp cận thú vị sơ cấp cho vấn đề trình bày sử dụng tính chất hệ đếm số 3: Mọi số nguyên N0 biểu diễn dưới dạng

1

1

3i 3i

i i

N    i  1,j  1; 0;1 ,  j 0,1, ,i1 Mệnh đề chứng minh dễ dàng quy nạp toán học

Việc f x y( , ) [0; 1] không thành vấn đề ta chia giá trị khởi tạo cho số đủ lớn để đạt điều

Ngoài cách xây dựng theo đường chéo với 4n giá trị khởi tạo (2n cho trường hợp f x y mà ( , ) x y chẵn 2n cho trường hợp f x y mà ( , ) x y lẻ), ta xây dựng theo hai cạnh dọc theo hai cạnh ngang có độ dài 2n

Chẳng hạn, trường hợp xây dựng theo cạnh dọc, song song với trục Oy , ta làm sau:

Bằng quy nạp, ta chứng minh

 

 

( , ) ( 1,1) ( 3,1) ( 1,1)

( , 1) ( ,1) ( 2,1) ( ,1) ( 2,1) ( ,1)

f x i i f x i f x i f x i

f x i f x i f x i f x f x i f x i i

          

              

Ta cần chọn f x( ,1),x [ 2n1; ]n cho:

i/ i 1 f x( 2i1,1) f x( 2i3,1)   f x( 2i1,1)i

ii/ if x( 2 ,1)if x( 2i2,1)   f x( ,1)   f x( 2i2,1) f x( 2 ,1)i  i iii/ Các giá trị f x( ,1) phân biệt với x [ 2n1; ]n

iv/ f x i( , 1) f x( 2 ,1)if x( 2i2,1)   f x( ,1)   f x( 2i2,1) f x( 2 ,1)ii với i n ln n

Từ đưa lên đường tròn kết hợp với bất đẳng thức để chọn, tức xét số giá trị đủ nhỏ thích hợp chúng đơi phân biệt Cách xây dựng cho trường hợp cạnh ngang thực tương tự

(64)

20 (Việt Nam TST 2003) Cho hàm số f :   thỏa mãn đồng thời điều kiện sau: i) f(0,0)52003, (0, )f n 0 với n số nguyên khác

ii) ( , ) ( 1, ) ( 1, ) ( 1, 1) ( 1, 1)

2 2

f m n f m n f m n

f m nf mn                

     

với số nguyên dương m số nguyên n

Chứng minh tồn số nguyên dương M cho f M n( , )1 với số nguyên n thỏa

2003

2

n  f M n( , )0 với số nguyên n thỏa

2003

2 n  Tuy nhiên toán lại câu chuyện khác !

Bài

Tìm số k nguyên dương lớn thỏa mãn: Với a b c dương mà , , abc1 ta có bất đẳng thức sau

1 1

3

k k

abca b c     (*)

Lời giải

Giả sử k số nguyên dương cho bất đẳng thức cho với a b c mà , ,

1

abc Thay 2,

3

b c  a vào (*), ta

3 880

2 14

2

2 63

1 3

k k

k

       

  

Hơn nữa, k số nguyên dương nên từ đánh giá trên, ta có k13 Ta chứng minh với k13 bất đẳng thức (*)

Thật vậy, với k13 ta có bất đẳng thức 1 13 25

abca b c    (**)

Đặt ( , , ) 1 1

f a b c

a b c a b c

   

   Khơng tính tổng quát, ta giả sử

 

max , ,

(65)

21

 

 

 

2

1 1

( , , ) , , 13

1 2 1

1 13

( 1) f a b c f a bc bc

b c bc a b c a bc

b c

bc a b c a bc

                                      

Do amaxa b c, ,  giả thiết abc1 nên ta có bc1, suy 1

bc

Mặt khác, sử dụng bất đẳng thức AM-GM biến đổi cho biểu thức ngoặc đẳng thức trên, ta có

    

13 13 13

1, 16 (a b c 1) a bc abc abc

  

      

nên hiển nhiên f a b c( , , ) f a , bc, bc Ta đưa toán chứng minh

2

1 25 , ,

4

f x x

x

 

 

  với x bc, x

  

Nếu x1 bất đẳng thức trở thành đẳng thức Trong trường hợp 0x1, cách sử dụng lại biến đổi thực trình tìm điều kiện cần cho k ta thấy , bất đẳng thức tương đương với

3

3

4 2

( 2)(2 1) 13

4( 2)(2 1) 13 (2 1) (2 1)

4(2 2) 26 13 20 18

x x x

x x x x x

x x

x x x x x x x x x x

                        Ta có

4 2

2 2

8 20 18 (8 2) (20 20 ) (10 14 6) 2(2 1) (2 1) 2(5 3)

x x x x x x x x x x x

x x x x x

            

       

do 2(2x21)2 0, (2x x1)2 0 5x27x 3 (tam thức bậc hai có hệ số cao dương biệt thức   11 0 ) Như vậy, bất đẳng thức cuối hiển nhiên Ta đến kết luận cuối k13 giá trị cần tìm

Nhận xét

(66)

22 Do bất đẳng thức có dạng đối xứng nên ta nghĩ đến việc chọn số mà có hai biến nhận giá trị Điều lý giải cho việc chọn a b x c 12

x

 lời giải Một cách khác để chứng minh bất đẳng thức (**) dùng phương pháp dồn biến sau:

Biến đổi (**) dạng đa thức ( )( 1) 13 25( )

ab bc ca a b c       a b c 

Giả sử amaxa b c, ,  a1 đặt x b c  bất đẳng thức viết lại thành ( )( 1) 13 25( )

4

ax bc a x     a x Bất đẳng thức tương đương với

2 2 2 25 25

( ) 14

4

25 25

4 25

4 14 0

2

a

ax bc a a x

bc a a

bc a a

a x a a                                    

Do x b c bc a     nên

2 2

2 25 2 25 4 25

4 4

4 0

2 2

a bc a a a a

bc a

a a

x

a a a

      

 

   

Ta đưa chứng minh

2

2

2 25

1 25

4 4

25

4 14 0

2

a a

a a a a

a a a a a                             

Đặt a  t ta đưa

2 2 2 2 2 25 25 4 25

4 14 0

4

2

t t

t t t

t t t t t t                             

hay (t1) 82 t49t318t220t80 Đặt t

u

 ta đưa bất đẳng thức đề cập cách ban đầu

Ta chứng minh bất đẳng thức 1 13 25

1

abca b c    không dùng đến

(67)

23 Do abc1 nên tồn số thực dương x y z, , cho

2

2

, y ,

x z

a b c

yz zx xy

   , ta cần chứng minh

2 2 3

13 13

3

yz zx xy xyz

xyzxyzxyz  

Do x y z, , bình đẳng nên ta giả sử xyz Viết bất đẳng thức thành dạng tương đương

3 3 3 2 3

2 2 3

3 13( )

4( )

x y y z z x x y z x y z xyz

x y z x y z xyz

     

  

Ta có đánh giá x3y3z33xyz(x y z  ) ( x y )2(z x z y )(  )

 

3 3 3 2 2

2

3 ( ) ( ) ( )( ) ( ) ( ) ( )( )

x y y z z x x y z xy yz zx z x y xy z x z y xy xy yz zx x y z x z y

 

           

 

        

Ta cần chứng minh

2

2 3

( ) ( ) ( )( ) 13( ) ( ) ( )( )

4( )

xy xy yz zx x y z x z y x y z x y z x z y

xyz x y z xyz

   

               

   hay

3 3

4(xy yz zx x  )( yzxyz) 13 xyz x y z(   )

Do tính nên ta giả sử x y 2 dẫn đến

3

3 2 2

2 x y xy     

 

Suy

3 3 3

(xy yz zx x  )( yzxyz) ( xy2 )(2zzxyz)xy(2z ) (2 zzxyz) Ta đưa

3 3

4 3

4 (2 ) (2 ) 13 ( 2) (2 ) (9 18 8) 18 16 (1 )(9 18 8)

xy z z z xyz xyz z z z xy z z

z z z z xy z z

          

         

Chú ý xy1 z1 nên (1xy)(9z318z28) 0 ; tiếp tục đặt z 1 t với t0

và thay vào 8z49z3 18z216z 8 8t423t33t215t5 với t0

Dễ dàng chứng minh biểu thức không âm nên bất đẳng thức cần chứng minh Từ ta có đpcm

Ta thấy việc xử lí bất đẳng thức (**) khơng q khó vấn đề lại nghĩ cách chọn 2,

3

b c  a để có k14 Điểm tinh tế khó

(68)

24 Trong bất đẳng thức cho, đẳng thức xảy a b c  1 nên ta dự đoán giảm số

biến cách đặt a 1,b x c,

x

   thu k x x

 

    

 

, tiếp tục cho

1

x có k16 Tuy nhiều điều kiện chưa chặt!

Ta tiếp tục cho b c 1,a x2

x

   thay vào bất đẳng thức ban đầu, ta

4 3

2

4(2 2) 4( 2)(2 1)

( )

2

x x x x x x x

k f x

x x x x

      

  

  với x0 (***) Khi đó, ta phải có

0 ( )

x

k f x

Dưới ta phân tích số cách để từ (***) suy k14

(1) Ta có f x( ) 0 tương đương với phương trình bậc 5:

5

( )

g xxxxx 

Ta không giải phương trình này, khơng thể tìm chặn xác cho k Tuy nhiên, tốn u cầu tìm k ngun dương lớn nhất, nên ta khơng cần tìm giá trị min, mà đánh giá nhằm tìm chặn cho k

Chú ý phương trình g x( ) 0 viết dạng

2

4 4x 8x

x x

   

Trên miền (0;) vế trái hàm tăng, vế phải hàm giảm nên g x( ) 0 có nhiều nghiệm dương

Chú ý 0, (1) 12

g    g  

  nên nghiệm x0 phương trình nằm

1

2 Ta khơng có ý định tìm x0 mà thơng tin dùng để có định hướng để

chọn giá trị x để thay vào bất đẳng thức

3

2

4( 2)(2 1)

x x x

k x x

  

Ta thay giá trị đặc biệt nằm 1;1        Thay x1, ta k16

 Thay

2

(69)

25  Thay

3

x , ta k13,98

 Thay

4

x , ta k14,

Đến đây, ta k14

(2) Ở cách tiếp cận này, ta chứng minh k14 không (do đánh giá đơn giản dễ đưa trường hợp k14 nhiều hơn)

Với k14, ta có bất đẳng thức 4x410x310x25x 4 Ta chứng minh bất đẳng thức không với x0 Đặt h x vế trái nó, ta có ( )

3 2

( ) 16 30 20 5, ( ) 48 60 20

h x  xxxh x  xx

Ta thấy h x( ) 0 có nghiệm f(0) 0  f(1) nên khảo sát hàm số h x ta ( ), phương trình h x( ) 0 có nghiệm x0 thỏa x0(0;1) 0

0

min ( ) ( )

xh xh x Chú ý 0 0 3;1

4 4

f  a  a  

    nên

3

0 0

16x 30x 20x  5 ta có

4 3

0 0 0 0 0

3

0 0

7 ( ) 7(4 10 10 4) (16 30 20 5) ( 1)(12 52 28)

f x x x x x x x x x

x x x x

        

    

Ta có

3

3

0 0

3

12 52 28 12 52 30

4

xxx          

   

x0 1 nên f x( ) 00  , mâu

thuẫn (chú ý

12x 52x 2x28 đồng biến 3;

 

 

 

) Do k14 khơng thỏa mãn

(3) Do f x có dạng cồng kềnh nên để nguyên mà khảo sát ( ) phức tạp Do đó, ta nghĩ đến việc biến đổi f x dạng đơn giản để dễ tính đạo ( ) hàm f x có dạng phân thức nên ta nghĩ đến việc tách chia đa thức: ( )

2

( )

2

f x x x

x x

   

(70)

26

2

1

( )

(2 1)

f x x

x x            

Ta cần giải phương trình f x( ) 0. Quan sát chút, ta phát :

3 2

2 2 2

1 ( 1)( 1) 4( 1)

,

(2 1) (2 1)

x x x x x x

x

x x x x x

     

    

 

Như vậy, ta phân tích nhân tử:

2

1

( ) ( 1) ,

(2 1)

x f x x x

x x             

từ ta đưa việc xét f x( ) 0 xét phương trình đơn giản hơn:

3

2

1

0 4 (2 1)

x

x x x

x x

      

Đến đây, ta gặp phải khó khăn lớn, phương trình 4x34x23x 1 0 khơng có “nghiệm đẹp” Khi khảo sát phương trình, ta quan tâm đến hai việc: phương trình có nghiệm giá trị Việc thứ dễ dàng thực cách viết lại phương trình dạng:

2

4x 4x

x

   

Vế trái phương trình hàm liên tục đồng biến với x0, đồng thời kiểm tra trực tiếp ta dễ thấy phương trình có nghiệm thuộc (0, 1) nên cách kết hợp điều lại, ta khẳng định phương trình f x( ) 0 có nghiệm thuộc (0, 1)

Việc thứ hai thực khó khăn trường hợp tốn Rõ ràng với việc tính tay, ta khó tính giá trị xác nghiệm phương trình

3

(71)

27 trình 4x34x23x 1 Gọi x0(0, 1) nghiệm phương trình Ý tưởng ta khử dạng bậc ba để có phương trình bậc thấp giải nhanh chóng tính tay Đầu tiên ta có để ý với x0(0,1) 4x03 4x02,

2

0 0 0

8x 3x  1 4x 4x 3x  1

Từ đây, ta tìm chặn cho x0 0 41 16

x    Tiếp theo, ta tìm chặn cho x0 Bằng ý nhỏ với x0(0,1) lũy thừa nhỏ “xấp xỉ” với nên để khử bậc ba, ta mạnh dạn sử dụng bất đẳng thức AM-GM sau:

3 2 2

0 0 0 0 0 0

04x 4x 3x  1 (4xx ) 4 x 4x  1 4x 4x 4x  1 8x 4x 1

Từ đây, ta tìm chặn cho x0 0

x   Như 41 0 16 x

 

  Vì

các đánh giá sát nên ta mạnh dạn chọn

x  để thay vào f x Để ý ( ) x nghiệm phương trình 8x2 4x 1 0, 1, 8 4, 4

8

x

x x x

x x

    

Suy

4 45 95 45 95 45 50

( ) 2(8 4) (4 )

8 8

x

f x    xx   xx      

Với kết này, ta thu 45 50 45 25

2 45 0,866 25 13,97

k        

Tất nhiên, lựa chọn giá trị đẹp cho x để thuận tiện cho việc tính tốn ta nên thận trọng cần chọn số x lệch xa so với x0 thơi đưa đến việc k bị lệch đơn vị sang 14, 15 chí

16 Tốt nhất, ta nên sử dụng giá trị mà ta biết “sát” với

x dù lẻ chút khơng sao, bù lại ta yên tâm kết

Sau tìm k13, ta thử chứng minh bất đẳng thức với k13

(72)

28 Cuối cùng, ta cách để tìm số thực k gần với số tốt để bất đẳng thức (*)

Đặt F x( ) 4(2x4 5x3 2x2 x 2) k x(2 2) 0,x

         Vì với số tốt điểm cực tiểu x0 đa thức gần với

3 nên ta giả sử với k cần tìm điểm cực

tiểu x0

3, tức đạo hàm

3

( ) 32 60 16 (4 1)

F x  xxx k x

2

x , ta tính 1372

99

k Với giá trị k F x đạt cực tiểu ( )

3 nên

2 152

( )

3 891 F xF  

 

Do đó, bất đẳng thức (*) với 1372 13, 85859 99

k 

Chú ý với 1 880 13,96825 63

kk   với k2 13,85859 (*) nên

số thực tốt k thỏa mãn k2 kk1 (ta tính k13,96764) Một điều thú vị dù k2 dù lớn 13 nhiều bất đẳng thức trường hợp tương ứng lại dễ chứng minh

Nhận xét chung, tốn có hình thức đơn giản lại địi hỏi nhiều xử lí trung gian tinh tế, phần tính tốn điều kiện thời gian có giới hạn khơng có máy tính hỗ trợ Nhiều bạn chủ quan gặp này, đánh giá k vội vàng để

8

kk16 từ “kết luận” để điểm đáng tiếc Một suy nghĩ thường thấy việc chứng minh bất đẳng thức biến chuyện đơn giản (các dồn biến thường đưa bất đẳng thức biến hiển nhiên đúng) thực vậy; phương trình đại số, bất đẳng thức biến khó chí khó q trình xử lí, khơng thu nghiệm đặc biệt

Có tốn có cách giải giống với (phần chứng minh), nhiên đơn giản Có lẽ tốn phát triển từ toán

(73)

29 Cho a b c số thực dương cho , , abc1 Chứng minh

(a b b c c a )(  )(  ) 7 5(a b c  )

2 Cho a b c số thực dương cho , , abc1 Chứng minh

2

3

a b c   ab bc ca 

Bài

Cho tam giác ABC nhọn khơng cân có góc A 45 Các đường cao AD BE CF , , đồng quy trực tâm H Đường thẳng EF cắt đường thẳng BC P Gọi I trung điểm BC; đường thẳng IF cắt PH Q

1 Chứng minh IQH AIE

2 Gọi K trực tâm tam giác AEF ( )J đường tròn ngoại tiếp tam giác KPD Đường thẳng CK cắt ( )J G, đường thẳng IG cắt ( )J M, đường thẳng JC cắt đường trịn đường kính BCtại N Chứng minh điểm G M N C thuộc , , , một đường tròn

Lời giải

1 Giả sử ABAC, B nằm P C Trường hợp , ABAC chứng minh hoàn toàn tương tự

Trước hết, ta chứng minh PH vuông góc với AI

Thật vậy, gọi U V trung điểm , AH IH ta có , UV AI

Dễ thấy ( , , , )P D B C  1 nên theo tính chất hàng điểm điều hịa PB PC PD PI

Ta có PE PF PB PC nên PE PF PD PI hay P nằm trục đẳng phương đường trịn đường kính AH (tâm U) IH (tâm V)

Hơn H nằm trục đẳng phương hai đường tròn nên PHUV Do PHAI

Vì BAC45 nên EIF90, suy raIQH90  AIF EIF AIF AIE

(74)

30 T

N

M G

J K

Q

I P

E

D H F

A

B C

2 Ta thấy EKF ECF EKF EAF180 nên K thuộc đường trịn đường kính BC Do hàng điểm D P B C điều hịa nên ta có , , , ID IP IC2, mà IM IG ID IP (cùng phương tích I đến ( )J ) nên

IM IG IC hay ( )

IMC ICG c g c

  nên IMC ICG ICK45  (1)

Gọi T trung điểm PD theo hệ thức Maclaurin, ta có CB CT CD CP CK CG  

hay tứ giác GTBK nội tiếp BKG90 nên có GTD90 hay GTPD

Tam giác JPD cân J có T trung điểm PD nên JT vng góc với PD

Do đó, G J T thẳng hàng , , KGJ45

Mặt khác CN CJ CB CT CK CG nên tứ giác KNJG nội tiếp dẫn đến 45

JNG JKG JGK

       (2)

(75)

31 Nhận xét

Ở ý thứ 1, việc chứng minh PH vuông góc với AI thực nhiều cách Chẳng hạn, xét tứ giác BCEF nội tiếp đường trịn tâm I Theo định lí Brocard

H trực tâm tam giác AIP nên PH vng góc với AI

Cũng dùng cách tính tốn chứng minh AD PD

IDHD để chứng minh kết

Ý thứ ứng dụng đẹp hang điểm điều hịa có kết hợp hai hệ thức hệ thức Newton hệ thức Maclaurin Số tốn hàng điểm điều hịa vận dụng đến hai hệ thức toán không nhiều, nên câu 2) ý chứng minh hay thú vị Ngoài cách giải trên, ta sử dụng hướng tiếp cận khác sau:

Gọi L giao điểm thứ hai khác K hai đường tròn ( ),( )I J Khi đó, để chứng minh , , ,

G M N C thuộc đường tròn, ta đưa chứng minh GI CJ LK đồng quy , , Trước hết, ta theo hướng hoàn toàn túy sau:

L

N

M G

J K

I P

E

D H F

A

(76)

32 Chú ý ( , , , )P D B C  1 BKC90 nên KB KC phân giác ngồi , góc PKD Từ dẫn đến DKI DPK hay IK tiếp xúc với đường tròn ( )J , điều có nghĩa đường trịn ( ),( )I J trực giao dẫn đến ILJ GLC90

Hơn nữa, GI CJ đường đối trung tam giác GKL CKL Gọi ,

U trung điểm KL GI CJ đối xứng với , GU CU qua phân giác góc , ,

LGC LCG

  tam giác LGC

Do đường thẳng LK GU CU đồng quy , , U LK phân giác góc GLC nên đường thẳng LK GI CJ đồng quy , ,

Từ lời giải tốn hồn tất

Tuy nhiên, từ đầu, ta phát biểu lại tốn thành:

Cho tam giác vuông KIJ K dựng phía ngồi tam giác KJG vng cân đỉnh J KIC vuông cân đỉnh I Chứng minh GI CJ đồng quy điểm nằm đường , cao đỉnh K tam giác KIJ

(Chú ý tam giác GJK CIK vuông cân chứng minh hồn tồn dễ dàng) , Để giải toán này, ta cần biến đổi đại số thơng qua định lí Ceva, Thales xử lí nhanh chóng

Tuy điều chưa hồn hảo tốn câu 1) câu 2) khơng liên quan tới

Thực chất vai trò giả thiết gócA 45 khơng thật cần thiết tốn Các bạn tìm hiểu mở rộng đơn giản sau

Cho tam giác ABC đường cao AD BE CF đồng quy , , H, EFcắt BC G Gọi ( )K là đường trịn đường kính BC Trung trực BC cắt ( )K điểm L cho A L nằm , cùng phía BC Gọi ( )N đường tròn ngoại tiếp tam giác GDL Đường thẳng

CL cắt ( )N M khác L Đường thẳng MK cắt ( )N P khác M Đường thẳng

CN cắt ( )K Q khác C Chứng minh M P Q C thuộc đường tròn , , ,

Cách chứng minh hoàn toàn tương tự lời giải toán gốc

(77)

33 Bài

Cho khối lập phương 10 10 10  gồm 1000 ô vuông đơn vị màu trắng An Bình chơi trị chơi An chọn số dải 1 10  cho hai dải khơng có chung đỉnh cạnh đổi tất ô sang màu đen Bình chọn số của hình lập phương hỏi An màu Hỏi Bình phải chọn ô để với câu trả lời An Bình ln xác định màu đen?

Lời giải

Trước hết, ta chứng minh nhận xét tổng quát: Cho khối lập phương 2n2n2n gồm

8n ô vuông đơn vị màu trắng An Bình chơi một trò chơi An chọn số dải 1 n  cho với hai dải chúng khơng có chung đỉnh cạnh đổi tất sang màu đen Bình chọn số hình lập phương hỏi An có màu Khi đó, Bình cần chọn 6n ô xác định 2 được ô có màu đen

(78)

34 Do điều kiện hai dãy chọn khơng chung cạnh đỉnh nên với câu trả lời màu cho u mà Bình chọn Ta thấy gọi Ru hợp ô chọn màu đen, có ba dải ngang, dọc, chéo qua ô tơ màu đen Trong trường hợp này, ta cần chọn thêm số ô thuộc Ru để biết xác dải Nếu chọn thêm thơi An trả lời màu trắng, Bình khơng xác định dải hai dải cịn lại tơ đen Do đó, Bình phải chọn thêm hai Ru có khả trả lời Thêm vào đó, hai phải thuộc hai dải khác chúng thuộc dải tương tự trường hợp chọn ô nêu

Với nhận xét này, ta gán cho u hình lập phương số ( , , )a b c với định nghĩa sau:

a2 dải hình hộp theo chiều ngang qua u khơng có thêm điểm thuộc Sn a1 ngược lại

b2 dải hình hộp theo chiều dọc qua u khơng có thêm điểm thuộc

n

(79)

35  c2 dải hình hộp theo chiều chéo qua u khơng có thêm điểm thuộc

n

S c1 ngược lại

Khi đó, có hai ba số a b c có giá trị số cịn lại khơng vượt nên , ,

4

a b c  

Đặt T tổng số dùng để gán cho hình lập phương Khi đó, ta có

( )

n

n u S

T a b c S

   

Mặt khác, với dãy 1 n  khối lập phương (theo ba chiều) có khối thuộc tập hợp Sn khơng, Bình khơng có thơng tin dải trường hợp An trả lời tất Bình chọn tơ màu trắng Bình khơng biết dải cịn lại có tơ màu đen hay khơng

Dễ thấy có tất (2 )n 2 dải

1 2n  nằm ngang tất dải đóng góp 2(2 )n 2 đơn vị vào T (đóng góp vào số a theo định nghĩa trên)

Tương tự với (2 )n 2 dải

1 2n  dọc (2 )n 2 dải

1 2n  chéo nên ta suy

2

3 2(2 ) 24

T  nn

Từ đó, ta 4Sn 24n2 hay Sn 6 ,n n2  Đến đây, ta suy hai điều sau:

- Trong hình lập phương 2 2  , Bình cần chọn - Trong hình lập phương 10 10 10  , Bình cần chọn 150

Ta cách tô màu ô thỏa mãn đề từ cách xây dựng cho hình lập phương 10 10 10  cho (Trên thực tế, ta hồn tồn xây dựng cho trường hợp tổng quát cho hình lập phương 2n2n2n)

Thật vậy, hình lập phương 2 2  , trừ hai đối xứng qua tâm, Bình chọn cịn lại hình bên

(80)

36 Dễ thấy hình lập phương 2 2  , có khơng q dải tơ màu nên khơng có có hình lập phương tơ màu phải thuộc dải Do đó, ô chọn, ta có trường hợp:

- Nếu khơng có tơ đen hình lập phương khơng có

- Nếu có tơ đen đen cịn lại thuộc khơng chọn có dải với ô đen biết

- Nếu có tơ đen tất đen hình lập phương Do đó, cách chọn với hình lập phương 2 2  thỏa mãn điều kiện đề Tiếp theo, ta xây dựng cho hình lập phương 10 10 10  sau:

Ta chia hình lập phương thành lớp 10 10 2  ta chia thành 25 phần, phần hình lập phương 2 2  đánh số hình bên dưới:

5 5

5

5 4 4

4

4

3 3

3 3

2

2 2

2

1

1 1

1

5 4 3

(81)

37 Ở lớp thứ i với i1, 2,3, 4, 5, ta chọn khối đánh số i với khối đó, ta bỏ ô đối xứng qua tâm cách nêu chọn lại

Dễ thấy với cách tô vậy, ta chiếu hình lập phương chọn xuống mặt hình chiếu phủ kín mặt Điều có nghĩa với dải mà An chọn qua hình lập phương 2 2  mà Bình chọn Khi đó, chứng minh trên, ta xác định dải có tơ màu hay khơng, tức xác định màu tất ô tơ màu đen hình lập phương ban đầu Do đó, cách chọn thỏa mãn đề

Vậy số mà Bình cần chọn 150

Nhận xét

Có thể nói tốn khó kỳ thi, nhiên, khó khơng nặng tính kỹ thuật mà khó mặt phương pháp Thực tế nhiều thí sinh nói “Em thực phải xoay sở nào!”

Để làm toán này, trước hết phải thật tập trung để hiểu rõ yêu cầu toán phải dùng trường hợp n2 xét tốn chiều để hình dung tốn cách cụ thể Trong nhận xét nhật xét dải có chọn hiển nhiên (và nhiều thí sinh phát biểu nhận xét này), nhận xét ngồi khối u hợp Ru cịn có khối chọn tinh tế Khi có hai nhận xét việc áp dụng kỹ thuật đếm hai cách để đánh giá tự nhiên

Việc xây dựng cấu hình cách chọn thách thức

Ở đây, trường hợp n2 đóng vai trị quan trọng, viên gạch để ta xây dựng Rõ ràng cấu trúc hình vng Latin giải giúp giải tốn Các mơ hình, cấu trúc, cách xếp ln đóng vai trị quan trọng tốn xây dựng ví dụ, phản ví dụ Nếu có làm quen nhiều với tốn cực trị rời rạc xử lí này, việc xây dựng cách chủ động

(82)

38 (Việt Nam TST 2000) Trong mặt phẳng cho 2000 đường tròn bán kính cho khơng có hai đường tròn tiếp xúc hợp đường trịn tạo thành tập hợp liên thơng Chứng minh số điểm thuộc hai đường trịn đường trịn cho khơng nhỏ 2000

(Việt Nam TST 2010) Gọi hình chữ nhật có kích thước 1 2 hình chữ nhật đơn một hình chữ nhật có kích thước2 3 , bỏ góc chéo (tức có có vng con) hình chữ nhật kép Người ta ghép khít hình chữ nhật đơn hình chữ nhật kép lại với bảng hình chữ nhật có kích thước là2008 2010 Tìm số nhỏ hình chữ nhật đơn dùng để ghép

Một lần tốn cực trị kì thi chọn đội tuyển, dấu xảy xây dựng khơng q khó để giá trị tốt điều hồn tồn khơng dễ dàng, chí khó!

Một vấn đề thú vị đặt kích thước hình lập phương lẻ ta có kết nào? Áp dụng cách đánh giá chặn trường hợp n chẵn, ta

2

nk số hình vng cần đánh dấu phải thỏa mãn bất đẳng thức

3(2 1) k

T   Từ suy T 6k2 6k2 Tuy nhiên, việc xây dựng cách chọn

6k 6k2 hình lập phương đơn vị thỏa mãn yêu cầu toán câu hỏi mở Với n3, tức k1, chưa tìm cách chọn 14 hình lập phương thỏa mãn yêu cầu toán chưa chứng minh cần nhiều (Ta 15 hình lập phương thỏa mãn yêu cầu)

Ta thử thay giả thiết nêu đề chút để có tốn mới: “Bình chọn bất kì hình lập phương hỏi An có màu sau tiếp tục thế” kiện khác giữ tương tự

Hoặc toán trường hợp hai chiều: Thay khối lập phương bảng ô vuông thay dải ô vuông chọn hàng, cột bảng

(83)

Trần Nam Dũng (chủ biên)

Võ Quốc Bá Cẩn – Trần Quang Hùng – Lê Phúc Lữ Hoàng Đỗ Kiên – Nguyễn Huy Tùng

LỜI GIẢI VÀ BÌNH LUẬN

ĐỀ CHỌN ĐỘI TUYỂN QUỐC GIA DỰ THI IMO 2014

(84)(85)

Việt Nam Team Selection Test 2014

Đề thi thức

Ngày thi thứ - 25/03/2014

Bài 1.Tìm tất hàm số f :Z→Zthỏa mãn

f 2m+ f(m) + f(m)f(n)

=n· f(m) +m, m,n∈Z.

Bài 2.Trong mặt phẳng tọa độ vng góc O x y,xét điểm nguyên có tọa độ thuộc tập hợp sau

T =

(x; y):−20¶x, y¶20,(x; y)6= (0; 0) .

Tô màu điểm thuộcT cho với điểm có tọa độ(x,y)∈T có hai điểm (x;y) (−x;−y) tô màu Với cách tô thế, gọi N số

(x1;y1),(x2;y2)mà hai điểm tô màu

x1≡2x2,y1≡2y2(mod 41).

Tìm tất giá trị có củaN.

Bài 3.Cho tam giác ABC nội tiếp đường trịn(O).Trên cung BC khơng chứa Acủa

(O) lấy điểm D Giả sửC D cắt AB E BD cắt AC F Gọi(K)đường tròn nằm tam giácE BD, tiếp xúc vớiE B,E D tiếp xúc với đường tròn (O). Gọi(L)là tâm đường tròn nằm tam giácF C D, tiếp xúc với F C,F Dvà tiếp xúc với đường tròn(O).

a) Gọi M tiếp điểm (K) vớiBE N tiếp điểm (L) vớiC F Chứng minh đường trịn đường kính M N ln qua điểm cố định khiD di chuyển b) Đường thẳng qua M song song với C E cắt AC P, đường thẳng qua N song

song vớiBF cắtABQ Chứng minh đường tròn ngoại tiếp tam giácAM P,ANQ tiếp xúc với đường tròn cố định Ddi chuyển

Ngày thi thứ hai - 26/03/2014

Bài 4.Cho tam giác ABC nhọn, không cân có đường cao AD P thuộc AD Các đường thẳngP B,PC cắtC A,ABtại E,F.

a) Giả sử tứ giácAE D F nội tiếp Chứng minh PA

P D = (tanB+tanC)cot A 2.

b) GọiC Pcắt đường thẳng quaBvng gócABtạiM.BPcắt đường thẳng quaC vng góc AC N K hình chiếu A lên M N Chứng minh B KC +∠M AN không đổi P di chuyển trênAD.

Bài 5.Tìm tất đa thức P(x),Q(x) có hệ số ngun thỏa mãn điều kiện: Với dãy số

(xn)xác định

x0=2014,x2n+1=P(x2n),x2n=Q(x2n−1)vớin¾1.

(86)

Việt Nam Team Selection Test 2014

Bài 6.Chom,n,plà số tự nhiên không đồng thời bằng0.Không gian tọa độ chia thành mặt phẳng song song cách Một cách điền vào khối lập phương đơn vị số từ1đến60được gọi cách điềnĐiện Biênnếu thỏa mãn: hình hộp chữ nhật với mặt hệ mặt cho tập hợp kích thước ba cạnh (số hình lập phương trên cạnh) xuất phát từ đỉnh là2m+1, 2n+1, 2p+1, khối lập phương đơn vị có tâm trùng với tâm hình hộp chữ nhật điền số trung bình cộng số điền tâm của8hình lập phương góc hình hộp Hỏi có tất cách điềnĐiện Biên?

(87)

Việt Nam Team Selection Test 2014

Nhận xét chung

Xét cấu trúc, đề thi chọn đội tuyển thiIMO năm gồm6bài tốn, có 2bài hình học phẳng,2bài đại số và2bài tổ hợp Cụ thể sau:

• Bài 1: Đại số (phương trình hàm tập số nguyên) • Bài 2: Tổ hợp (bài tốn đếm)

• Bài 3: Hình học phẳng (đường điểm cố định) • Bài 4: Hình học phẳng (hình học tính tốn)

• Bài 5: Đại số (phương trình hàm đa thức tập số ngun) • Bài 6: Tổ hợp (bài tốn đếm)

Ngồi hai hình tính chất số học xuất bốn cịn lại đóng vai trò làm để xây dựng vấn đề không thực số học Trong ngày, toán xếp cách tương đối theo độ khó tăng dần Bài14có thể coi hai dễ Tiếp theo bài2,bài5 mức độ trung bình khó bài3 6.Theo ý kiến chúng tôi, để lọt vào đội tuyển, thí sinh chắn phải giải bài1, 4và làm hồn chỉnh1bài bài2, 5(cũng định), lại khó có thí sinh giải trọn vẹn bài36.

Hai hình học năm nay, xét tính phân loại ổn, có dễ khó, câu3bsẽ thách thức đáng kể Bài4khá dễ xét lại vị trí kỳ thi năm trước Tuy nhiên, đặt vào chung với bài56trong ngày thi lại hợp lý tạo thử thách nhỏ cho thí sinh trước bước vào hai thử thách lớn Về bốn lại, kiến thức sử dụng nói chung quen thuộc, địi hỏi tư túy trình bày cẩn thận khơng dùng định lý khó Bài1là phương trình hàm tập số tự nhiên tính vào đại số, nắm vững cách xử lý phương trình hàm vượt qua khơng khó khăn Bài5về đa thức, đại số có liên quan đến tính chất số học, giải tích Bài26đều tổ hợp đặt số học, xu hướng thường thấy đềVMO, TSTnhững năm gần

Ta thấy điểm trùng hợp ba bài2, 56 có khai thác yếu tốchiềuvà mà tốn hình dung dễ đưa trường hợp1chiều Ba có nhiều điểm khó, cách tiếp cận, lập luận chặt chẽ trình bày lời giải lẫn đáp số cuối toán So với đề thi năm trước thấy đềTST 2014có phần nhẹ hơn, khơng có khó hai dễ kỳ thi dễ Tuy vậy, đề thi đảm bảo tính phân loại cao, hình địi hỏi nắm kỹ thuật định để xử lý khác lại địi hỏi tư chính, hạn chế sử dụng định lý, cơng cụ mạnh đặc điểm bật ưu điểm đề thi

(88)

Việt Nam Team Selection Test 2014

Lời giải chi tiết bình luận

Bài 1.

Tìm tất hàm f :Z→Zthỏa mãn

f 2m+ f(m) + f(m)f(n)

=n f(m) +m, m,n∈Z.(∗)

Lời giải Đặt a= f(0).Rõ ràng f ≡0không phải nghiệm phương trình, tồn m0 Zsao cho f(m0) =6 0. Từ đó, cách thay m = m0 vào đẳng thức(∗) ta dễ

dàng suy f đơn ánh Mặt khác, thayn=0vào(∗),ta f 2m+ (a+1)· f(m)

=m, m∈Z.(∗∗)

Kết chứng tỏ f tồn ánh Do đó,b∈Zsao cho f(b) =−1.Thay m =n = b

vào(1),ta f(2b) =0.Mặt khác, thaym=n=0vào(∗),ta có f(a2+a) =0.

Từ đây, kết hợp với tính đơn ánh f,ta suy b= a

2+a

2 . Bây giờ, thayn =bvào(∗),ta

f(2m) = a

2+a

2 ·f(m) +m, m ∈Z.(∗ ∗ ∗) Tiếp tục, thaym=0vào(∗),ta

f a·f(n) +a

=an, n∈Z.

Đến đây, cách thaym=an vào(∗∗)rồi so sánh với đẳng thức trên, ta suy

(a+1)· f(an) +2an=a· f(n) +a, n∈Z.

Còn thayn=b,ta a(a

2+a)

2 = f(0) =a,từ suy raa∈ {0, 1,−2}.Ta xét trường hợp sau:

• Nếu a=1 dễ dàng suy f(n) =1−2n,∀n ∈Z. Tuy nhiên, thử lại ta thấy

hàm không thỏa mãn yêu cầu đề

• Nếua=0thì từ(∗ ∗ ∗),ta suy f(2m) =m,m ∈Z.So sánh kết với(∗∗),

ta f(m) =0,∀m ∈Z,vơ lý f phải hàm khác

• Nếua=−2, ta có

(89)

Việt Nam Team Selection Test 2014

Mặt khác, theo(∗ ∗ ∗),ta lại có f(−2n) = f(−n)−n.Do đó, ta f(−n) +3n=2·f(n) +2, n ∈Z.

Thay n bởin vào đẳng thức trên, ta có f(n)−3n =2· f(−n) +2, n∈Z.

Kết hợp hai điều lại, ta suy f(n) =n −2,∀n Z. Thử lại, ta thấy hàm

thỏa mãn yêu cầu đề

Vậy hàm số cần tìm f(n) =n−2,∀n∈Z.

Nhận xét.

Ở tốn trên, tính đơn ánh – toàn ánh nhận thấy dễ dàng cịn với số thực khơng tập số nguyên Đây điều tiên cần có để việc thay giá trị thực dễ dàng

Ngoài cách giải trên, ta dự đốn biểu thức hàm số cách chọn số đặc biệt, ta xử lý nhanh chóng “nhu cầu” Việc phát hàm số f(n) =n−2nói chung khơng khó hàm số cần tìm đa thức dễ thấy bậc từ thay vào đồng hệ số Bằng cách thay giá trị thích hợp, ta tính

f(4) =2,f(3) =1,f(2) =0, f(1) =−1,f(0) =−2. Thaym=3vào công thức cho, ta có

f 7+f(n)

=n+3với mọin.

Từ quy nạp để có kết Một cách khác nhanh sau:

Đặt a = f(0) u,v,w số nguyên thỏa f(u) = 0,f(v) = −1, f(w) = 1 Ta thực thay giá trị thích vào biểu thức hàm số cho sau: Nếum=w,n=uw f 2w +1+f(uw)

=u

Nếum =u,n=0thì f(2u) =u Suy

2w +1+ f(uw) =2u⇒ f(uw) =2(uw)−1.

Tiếp tục thaym=x ∈Z,n=uw f 2(x+ (uw)f(x))=x+ (uw)f(x)

Cuối cùng, thaym=x+ (uw)f(x),n =v

x+ (uw)f(x) =u hay f(x) = ux uw.

Đây biểu thức tuyến tính theox, ta dễ dàng chọn giá trị thích hợp

Ngồi ra, ta cịn tiếp cận theo cách sau: Ta chứng minh f(2m−f(m)) =m f song ánh nên f(m)+f−1(m) =2m Đây xuất kỳ thiAPMO 1989:

Xác định tất hàm số f :R→Rthỏa mãn

f đơn điệu tăng

(90)

Việt Nam Team Selection Test 2014

Bài 2.

Trong mặt phẳng tọa độ vng góc O x y, xét điểm nguyên có tọa độ thuộc tập hợp sau

T =

(x; y):−20¶x, y¶20,(x; y)6= (0; 0) .

Tô màu điểm thuộc T cho với điểm có tọa độ(x,y)∈T có hai điểm(x;y)(−x;−y)được tơ màu Với cách tô thế, gọiN số bộ(x1;y1),(x2;y2)mà hai điểm tô màu

x1≡2x2,y1≡2y2(mod 41).

Tìm tất giá trị có củaN.

Lời giải Trước hết, ta chuyển tốn từ2chiều thành1chiều

Ta có210 ≡ −1(mod 41)nên40số ngun khác0 có giá trị tuyệt đối khơng vượt q20

có thể chia thành2dãy, dãy có độ dài20sao cho số hạng đầu chia41x số hạng sau chia412x.

Tô màu số dãy cho trong2số đối có đúng1 số tơ màu Ta quan tâm đến số lượng cặp số liên tiếp tô màu dãy

Dễ thấy 220 1(mod 41) nên thêm số hạng đầu dãy vào cuối dãy mới

gồm 21 số thỏa mãn tính chất nên ta chuyển thành vịng trịn phát biểu lại toán sau: Cho đa giác đều20đỉnh nội tiếp đường tròn cho 2điểm đối xứng qua tâm có 1 đỉnh tơ màu Tính số cặp đỉnh liên tiếp tơ màu có

Vớin số chẵn, gọiSn tập hợp số cặp kề tơ màu có đa giác cón đỉnh Ta chứng minh quy nạp

S4n ={2k+1|0¶kn−1}S4n−2={2k|0¶kn−1}.

Điều chứng minh quy nạp sau: Vớin =2thì dễ thấy nhận xét

Giả sử nhận xét đếnn ¾2.Xét đa giác có4n+2đỉnh Đa giác tạo thành cách thêm đỉnhAvào hai đỉnh thứ2n, 2n+1và thêm đỉnhBvào hai đỉnh thứ4n, 1 Ta xét trường hợp:

• Nếu đỉnh2n, 2n+1đều tơ vàAkhơng tơ tương ứng: đỉnh4n, 1không tô vàBđược tô Số cặp kề tơ giảm

• Nếu đỉnh 2n, 2n+1đều tơ vàAcũng tơ tương ứng: đỉnh4n, 1không tô vàBkhông tô Số cặp kề tăng lên1.

• Nếu hai đỉnh 2n, 2n+11 đỉnh tơ vàA tơ tương ứng: hai đỉnh4n, 11đỉnh tô vàBkhông tô Số cặp kề tăng lên1. • Nếu hai đỉnh 2n, 2n+1, có1đỉnh tơ vàAkhơng tơ tương ứng:

(91)

Việt Nam Team Selection Test 2014

Do đó,S4n+2={x±1|x S4n}hayS4n+2={2k|0¶kn}

Tương tự, ta cóS4n+4={x±1|x S4n+2} Tất nhiên khơng xảy trường hợp S4n+4

có chứa số−1vì để có trường hợp0cặp số ở4n+2, đỉnh phải tô xen kẽ trường hợp giảm số khơng xảy ra, suy raS4n+4={2k+1|0¶kn}.

Rõ ràng cách chứng minh trên, ta cách xây dựng trường hợp để tơ màu thỏa mãn tất giá trị tập hợp tương ứng Nhận xét chứng minh

Từ suy raS20 ={1, 3, 5, 7, 9}.

Do đó, kết cho tốn phụ là2S20 với định nghĩa2S=S+S={a+b|a,bS}

Quay trở lại toán ban đầu, để chuyển từ 1 thành phần x thành 2 thành phần

(x,y), ta thực sau:

Ứng với vòng tròn chứa số thuộc dãyA,ta lấy vòng tròn gồm số thuộc dãy A đặt lên cho số thuộc đường tròn cũ khớp với số thuộc đường tròn Viết cặp số khớp thành dãy, dãy dãy tọa độ điểm mà liền sau của(x1,y1)(2x1, 2y1)theo mod 41

Dễ thấy có tất 20 cách ghép (cố định vòng tròn cũ xoay vòng tròn mới) Tương tự với việc ghép dãyAB,BA,BBnên có tổng cộng là80 cách ghép tạo thành80dãy Tuy nhiên, ta xét thêm4dãy đặc biệt, tương ứng với điểm nằm trục tung trục hoành

Cụ thể xét thêm dãyC gồm20số0 xét4cách ghép: AC,CA,BC,C B.Do đó, tổng cộng có84dãy tọa độ

Theo chứng minh dãy, số cặp có làS20 nên đáp số

tốn là84S20, số chẵn từ1·84=84đến9·84=756 Bài tốn giải

quyết hồn tồn Nhận xét.

Để giải bài2, ta phải “cảm nhận” tốn Một cách tự nhiên lấy x,y nghĩ đến việc tìm2x, 2y (thay ngược lại xét điểm tơ màu) Từ đó, biết số xoay thành vịng trịn nhận thấy miền giá trị số cạnh việc đưa thành mơ nêu dễ trình bày nhất, thứ sáng sủa nhiều Sẽ có ích ta phát biểu tốn tổng qt sau làm việc với tham số nhỏ để tìm quy luật Bài chất khơng khó, dễ sai kết luận vội vàng thông qua số nhận xét nhỏ

Theo ý kiến chúng tôi, việc chấm khó khăn việc đánh giá giải theo hướng7− hay0+. Điểm mấu chốt việc tách riêng hoành độ, tung độ, mơ hình hóa dạng đa giác ý đến đồng dư thức210 = 1024≡ −1(mod 41).

Bài tốn có phong cách bài4VMO2012,bài3VMO2013,đều mẻ, dạng tổ hợp đếm có kết hợp số học, phát biểu có phần gượng ép việc đặt vào trục tọa độ lại giúp học sinh dễ tưởng tượng

(92)

Việt Nam Team Selection Test 2014

Trong không gianO x y z, xét điểm nguyên có tọa độ thuộc tập hợp

T =

(x;y;z):−20¶x,y,z¶20,(x;y;z)6= (0; 0; 0) .

Tơ màu điểm thuộc T cho với điểm có tọa độ (x;y;z)∈ T có hai điểm(x;y;z)(−x;−y;z)được tô màu Với cách tô thế, gọi N số bộ(x1;y1;z1),(x2;y2;z2) mà hai điểm tô màu tọa độ

chúng thỏa mãn điều kiện x1≡2x2,y1≡2y2,z1≡2z1(mod 41).Tìm tất giá trị có

thể có củaN.

Lập luận tương tự đây, ta lại đặt thêm vòng tròn hai vòng tròn cũ Ở ta có ord41(2) =20nên hợp thay 2041bởi số khác Ta thấy

số41, 20quyết định độ dài chu kỳ dãy trường hợp số lượng số chia hết cho chu kỳ này, nghĩa ta chia dãy số cho thànhcác vịng trịn tốn giải tương tự Như thế, ta đến toán tổng quát:

Vớik,n,p,r số nguyên dương mà ordp(r)(2k), xét tập hợp T số nguyên không đồng thời là(x1,x2,· · ·,xn)sao cho |xi k,i =1,n Tơ màu số thuộc T cho có hai (x1,x2,· · ·,xn) (−x1,−x2,· · ·,−xn) tô màu Với cách tô thế, gọiN số bộ(x1,x2,· · ·,xn)(x01,x02,· · ·,x0n)thỏa mãn:

• Cả hai tơ màu

xi =r x0i(mod p).vớii =1, 2, 3,· · ·,n.

Khi đó, tất giá trị có củaNRSordp(r) vớiR= 1 k

‚‚ 2k2

ordp(r)

+1 Œn

−1 Œ

và sốS2k định nghĩa

Tất nhiên, có trường hợp dãy khơng lặp lại phải tính tốn theo cách khác rõ ràng ta khơng thể xây dựng thành mơ hình khép kín đa giác Bài tốn cịn phát triển theo nhiều hướng thú vị!

Bài 3.

Cho tam giác ABC nội tiếp đường trịn (O).Trên cung BC khơng chứa Acủa

(O)lấy điểm D Giả sửC D cắt AB E BD cắt AC F Gọi (K) đường tròn nằm tam giác E BD, tiếp xúc với E B,E D tiếp xúc với đường tròn (O).Gọi(L)là tâm đường tròn nằm tam giác F C D, tiếp xúc vớiF C,F Dvà tiếp xúc với đường tròn(O).

a) Gọi M tiếp điểm (K) với BE N tiếp điểm của(L) vớiC F Chứng minh đường trịn đường kính M N ln qua điểm cố định khiD di chuyển trên(O).

(93)

Việt Nam Team Selection Test 2014 10

Lời giải a) Trước hết, ta chứng minh bổ đề sau:

Bổ đề (định lý Sawayama Thébault mở rộng).Cho tam giác ABC nội tiếp đường tròn

(O).D điểm thuộc tia đối tiaBA Đường tròn(K)tiếp xúcDA,DC tạiM,N tiếp xúc ngoài(O) Chứng minh rằngM N qua tâm bàng tiếp ứng với đỉnhAcủa tam giácABC

Chứng minh. Giả sử đường phân giác đỉnh C tam giác ABC cắt đường tròn

(O)tại E khácC.Đường thẳngC E cắt M N J Đường tròn (K)tiếp xúc với đường tròn

(O) F Dễ dàng thấy E,N,F thẳng hàng Thật vậy, E trung điểm cung AB chứaC (O)nên OE AB Ngồi ra, ta có M N AB nên M N kOE Mặt khác, K,O,F thẳng hàng nênE,N,F thẳng hàng

Từ đây, ta có tam giácE AFE N Ađồng dạng, suy raE A2=E B2=E N·E F.

O

B

C A

D

J N

K

M F

E

Ta lại có

F M N =

1

2∠F K N = 1

2∠F OE =∠F AE =∠F C J suy tứ giácC F M J nội tiếp Do đó,

E F J =180◦−∠N F J=180◦−(∠N F M+∠M F J) =180◦−(∠J MC+∠MC J) =∠M JC.

Từ đó, ta có4E F J ∼ 4E J N,suy E J2=E N·E F =E A2=E B2.

(94)

Việt Nam Team Selection Test 2014 11

Trở lại toán,

Giả sử đường tròn(K)tiếp xúcE D tạiG đường tròn(L)tiếp xúcF D tạiH Theo bổ đề thìMG,N H qua tâm bàng tiếp J ứng với đỉnh Acủa tam giác ABC.

Ta chứng minh rằngM J N=90◦ Thật vậy,

K E F+∠LF E =

1

2∠DE B+ 1

2∠D F C+∠DE F+∠D F E =90

◦−∠BAC+180◦−∠BDC =90◦

do E K LF Ta có MG E K LF N H nên MG vng góc N H J Vậy đường trịn đường kínhM N ln qua điểm J cố định Ta có đpcm

O

B C

D A

E

F J

M K N

L G

H P Q

Y

Z

R

b) Dễ thấy tam giác E MG cân M P kC E nênE MG =∠EG M =∠G M P Do đó, M J

chính phân giác ngồi tam giácAM PAJ phân giác gócBAC nên suy J tâm bàng tiếp gócAcủa tam giácAM P.

(95)

Việt Nam Team Selection Test 2014 12

Chứng minh tương tự, ta có (R) tiếp xúc với đường tròn ngoại tiếp tam giác ANQ Vậy đường trịn ngoại tiếp tam giácAM P,ANQln tiếp xúc đường trịn(R)cố định Ta có đpcm

Nhận xét.Với toán này, quen thuộc với định lý Sawayama Thébault mở rộng giải khơng q khó khăn Nhưng nói chung với hầu hết thí sinh, mặt phải xác định mơ hình nêu đề dạng mở rộng; mặt khác, bạn phải chứng minh lại thành công định lý thời gian tương đối ngắn, toán xứng đáng vị trí số3. Đây lần đề thi chọn đội tuyển Việt Nam có hình vị trí (một luật bất thành văn bài3chính khó đề thi)

Trước hết, ta nhắc lại định lýSawayama Thébaultdạng gốc: Cho tam giácABC nội tiếp đường tròn(O). D điểm thuộc đoạnBC.Đường tròn(K)tiếp xúcDA,DC M,N tiếp xúc với(O).Chứng minh M Nđi qua tâm nội tiếp tam giácABC. Trước hết, ta thử tập trung vào khai thác ý a Rõ ràng thay đường tròn ngoại tiếp thành đường tròn quaB,C, ta thu toán thú vị Chú ý câu a) nói lên MG,N H qua điểm Jcố định Ta có tốn sau:

Bài 3.1.Cho tam giác ABC đường tròn (O)bất kỳ cố định qua B,C. D điểm di chuyển trên(O) choA,D khác phía với B,C.Giả sửC D cắt ABE BDcắt AC F.Gọi (K)là đường tròn tiếp xúc với E B,E D M,Nvà tiếp xúc với(O). Gọi(L)là đường tròn tiếp xúc vớiF C,F Dlần lượt tạiP,Qvà tiếp xúc với(O).Chứng minh giao điểmM N,PQln nằm đường trịn cố định khiD di chuyển Bài 3.2.Cho tam giác ABC nội tiếp đường tròn (O). Gọi P điểm di chuyển cung BC chứaAcủa(O).P B,PC cắtC A,ABlần lượt E,F.Đường tròn(K)tiếp xúc với đoạn E A,E Bvà tiếp xúc với(O).Đường tròn(L)tiếp xúc với đoạnF B,F C tiếp xúc với cungBC khơng chứaAcủa(O).Chứng minh đường trịn đường kính M Nluôn qua điểm cố định khiP di chuyển

Bài 3.3.Cho tam giácABC(O)cố định quaB,C.GọiD điểm di chuyển cung BC (O) cho D,A phía với BC. DB,DC cắt C A,AB E,F. Đường tròn(K)tiếp xúc với đoạnE A,E Btại M,N tiếp xúc với(O).Đường tròn (L)tiếp xúc với đoạn F B,F C P,Qvà tiếp xúc (O) điểm khơng phía A so với BC. Chứng minh giao điểm củaM N PQluôn thuộc đường tròn cố định

Lời giải chi tiết tốn tham khảo trong[2]

Ý thứ hai toán xây dựng thú vị đòi hỏi phải xây dựng thêm đường trịn Mixtilinearbàng tiếp gócAcủa tam giácABC Tính chất sử dụng lời giải quen thuộc chứng minh định lýSawayama Thébaultnên không cần thực lại chứng minh tính chất Một tốn có liên quan xuất cách lâu đề chọn đội tuyển năm1999của Việt Nam:

Bài VN TST 1999.Cho tam giác A1A2A3nội tiếp đường tròn (O).Một đường tròn

(K1) nằm góc A2A1A3 tam giác A1A2A3, tiếp xúc với cạnh A1A2,A1A3

tiếp xúc với đường tròn(O)lần lượt điểm M1,N1,P1 Các điểmM2,N2,P2

M3,N3,P3xác định cách tương tự Chứng minh đoạn thẳngM1N1,M2N2,M3N3

(96)

Việt Nam Team Selection Test 2014 13

Bài 4.

Cho tam giác ABC nhọn, khơng cân có đường cao AD P thuộc AD Các đường thẳngP B,PC cắtC A,AB tạiE,F.

a) Giả sử tứ giácAE D F nội tiếp Chứng minh PA

P D = (tanB+tanC)cot A 2. b) GọiC P cắt đường thẳng quaB vng gócAB M.BP cắt đường thẳng qua

C vng góc AC N K hình chiếu A lên M N Chứng minh

B KC+∠M AN không đổi P di chuyển trênAD.

Lời giải. a) GọiQlà giao điểm E F BC hàng điểm(Q,D,B,C)điều hòa Từ suy chùm(AQ,AD,AB,AC)(DE,D F,DA,DQ)cũng điều hòa

Hơn nữa, doDA DQ nên DA phân giác E D F Do tứ giác AE D F nội tiếp đường tròn nên dễ thấyAE =AF

A

B D C

P F

E

G Q

Giả sử đường tròn ngoại tiếp tứ giác AE D F cắt BC G khác D Ta có DG phân giác ngồi đỉnhD nênGE =G F 4AGE =4AG F nên AG phân giác BAC Theo định lýMenelaus cho cát tuyến BP E tam giác ADC cát tuyến C P F tam giác ABD

E A

EC =

PA P D ·

BD BC,

F A F B =

PA P D ·

C D BC Từ suy

F A F B +

E A EC = PA P D BD BC + C D BC = PA

P D. Do đó, ta

PA P D =

F A G F.

G F F B +

E A GE.

GE EC =cot

A

2tanB+cot A

(97)

Việt Nam Team Selection Test 2014 14

b) Gọi L giao điểm BM C N rõ ràng điểm L cố định Tứ giác AC K N

AC N=∠AK N=90◦nên nội tiếp Tương tự,AB K M nội tiếp Do đó, ta có M AN+∠B KC =∠M AK+∠N AK+∠B KC =∠M B K+∠NC K+∠B KC =∠BLC

không đổi Ta có đpcm

A

B

C D

P

N

M

K L

Nhận xét.Đây tốn mức độ dễ thí sinh xử lý cách nhẹ nhàng với kiến thức quen thuộc hình phẳng Bài tốn mang màu sắc đại số nhiều nói chung địi hỏi thí sinh tính tốn, biến đổi (tương tự đềIMO 2013) tiếp cận theo hướng hình học túy Hai câu a) b) tốn khơng liên quan đến Ta tách riêng thành hai tốn phân tích toán Câu a) phát biểu điều kiện dạng biểu thức lượng giác chưa đẹp, ta hồn tồn có hệ thức lượng túy hình học câu a) Ta xét toán sau:

Bài toán 4.1.Giả thiết tương tự toán Chứng minh tứ giácAE F Dnội tiếp AD

P D =

AB·AC+AD2

DB.DC .

Đây dạng tổng quát câu a tốn gốc xử lý dễ dàng định lýVan-Obel: Cho tam giác ABCD thuộcBC P thuộc đoạnAD TiaBP cắt AC N, tiaC P cắt AB M Khi đó, ta có AD

AP =

AM

AB +

AN

(98)

Việt Nam Team Selection Test 2014 15

Bài toán 4.2.Cho tam giácABC có đường phân giác AD.GọiE,F hình chiếu củaD lên cạnhC A,C B.Giả sửBE giaoC F P.GọiH hình chiếu củaP lên BC.Chứng minh rằngH P phân giácE H F.

Cuối cùng, ta nội dung này, ta phát biểu tốn tổng quát sau:

Bài toán 4.3. Cho tam giác ABC với P điểm tam giác ABC. Các đường thẳng PA,P B,PC cắt AB,BC,C Alần lượt D,E,F. Gọi H,K hình chiếu F,E lên AD L,N hình chiếu F,E lênBC.Giả sử tứ giácAE D F nội tiếp Chứng minh

PA P D =

F H F L ·

DA

DB +

E K E N ·

DA DC.

Nói phần b, ta thấy toán đẳng giác quen thuộc Ta thêm số ý vào toán để vấn đề phong phú hơn:

Bài toán 4.4.Cho tam giácABC đường caoAD.ĐiểmP di chuyển trênAD.Giả sửP B,PC cắt đường thẳng quaC vng góc vớiC Avà quaBvng góc vớiABtại N,M. Gọi K hình chiếu củaAlên M N.

a) Chứng minh rằngM AN+∠B KC không đổi khiP di chuyển

b) Chứng minh rằngM AC =∠N AB.

c) Chứng minh K Alà phân giácB KC.

Dưới hai mở rộng theo đường đẳng giác cho toán trên:

Bài toán 4.5.Cho tam giác ABC nội tiếp (O). Các điểm E,F cố định thuộc (O)sao cho E F kBC.ĐiểmP di chuyển trênAE.Giả sử P B,PC cắt F C,F B tạiN,M.và trung trựcBC cắt M N K.Chứng minh rằngM AN+∠B KC không đổi khiP di chuyển

Bài toán 4.6.Cho tam giácABC nội tiếp đường tròn(O).E,F cố định thuộc(O)sao cho E F kBC.Các điểmP,Qlần lượt thuộcAE,AF.Giả sửP B,PC cắtQB,QC N,M. Chứng minh rằngM AB=∠N AC.

Lời giải chi tiết tốn tham khảo trong[3]

Bài 5.

Tìm tất đa thức P(x),Q(x) có hệ số nguyên thỏa mãn điều kiện: Với dãy số

(xn)xác định

x0=2014,x2n+1=P(x2n),x2n=Q(x2n−1)vớin¾1.

thì số nguyên dương mlà ước số hạng khác0nào dãy(xn)

Lời giải Ta chứng minh đa thức P(x),Q(x) thỏa mãn đề chúng phải có bậc là1.Thật vậy,

Xét trường hợp hai đa thứcP(x),Q(x)là đa thức hằng.(1)NếuP(x)≡a,x Zthì(xn)có dạng:

(99)

Việt Nam Team Selection Test 2014 16

và dễ thấy số hạng dãy nhận1trong3giá trị(2014,a,Q(a))

(2)NếuQ(x)≡a,x ∈Zthì(xn)có dạng

x0=2014,x1=P(2014),x2=a,x3=P(a),· · ·

và dễ thấy số hạng dãy nhận ba giá trị(2014,P(2014),a,P(a)) Cả hai điều không thỏa mãn điều kiện đề số nguyên dươngm phải ước số hạng khác0nào dãy số xn

Tiếp theo, hai đa thức P(x),Q(x) có bậc lớn 1, khơng tính tổng qt, ta giả sử làQ(x) Rõ ràng đóQ(P(x))cũng có bậc lớn hơn1.

Ta thấy nếuR(x)là đa thức có bậc lớn hơn1thì với mọik>0lớn tùy ý, tồn x có giá trị tuyệt đối đủ lớn sau cho|R(x)|>k|x|.Điều dễ thấy khix →+∞ ta có giới hạn

lim x→+∞

|R(x)|

|x| = +∞.

Ta chứng minh tồn N đủ lớn cho |Q(P(x))|>|P(x)|+|x|với x >N. Ta cần xét trường hợp: Nếu P(x) bậc dễ thấy tồn k đủ lớn cho k|P(x)|>|x|với x ∈Z.

Suy ra, tồn N đủ lớn cho

|Q(P(x))|>(k+1)|P(x)|=|P(x)|+k|P(x)|>|P(x)|+|x| với mọix >N. Nhận xét chứng minh

Theo giả thiết dãy số cho, phải tồn số hạng|xi|lớn tùy ý rõ ràng, ta phải có j >0sao cho x2j

>N +1 x2j có giá trị tuyệt đối lớn trong2j số hạng dãy(xn).

Thật vậy, ta thấy tồn vô số số hạng x2j thỏa mãn điều kiện x2j

= max

k¶2j {|xk|}, gọiT tập hợp số thỏa mãn Nếu số hạng thế, khơng có số hạng thỏa mãnx2j

>N+1thì với t T, ta có|xi|¶|xtN+1với mọii t T Tuy nhiên, do|T|vơ hạn nên điều giả sử vô lý nhận xét chứng minh Với x2j số hạng thỏa mãn điều kiện trên, chọn m=

x2j+2−x2j

thì ta thấy m=Q(P(x2j))−x2j

¾

Q(P(x2j))

x2j

>

P(x2j)

>

x2j

x2n+2 =

Q(x2n+1) >

x2n+1

.

Do đó, trong2j +1số hạng dãy, khơng có số hạng chia hết chom Mặt khác,x2k+2−x2k = Q(P(x2k))−Q(P(x2k−2))

P(x2k)−P(x2k−2)

x2kx2k−2

=

m tương tự x2k+3−x2k+1= P(x2k+2)−P(x2k)

.€x2j x2j−2

Š

=m Từ suy vớik¾ j x2k+2−x2k x2k+3−x2k+1đều chia hết chom, nhiênx2j+1x2j+2đều

(100)

Việt Nam Team Selection Test 2014 17

Điều mâu thuẫn cho ta thấy nhận xét ban đầu vàdegP(x) = degQ(x) =1 ĐặtP(x) =ax+bvi|a|>1vQ(x) =c x +d via,b,c,d Zvab6=0thỡ ta cú

ă

x2n+1=ax2n+b x2n+2=c x2n+1+d

với mọin ¾0.

Suy rax2n+2=cax2n+bc+d x2n+3=cax2n+1+ad+bvới mọin¾0

Cả hai dãy có cơng thức truy hồi dạng yn+1 =k yn+h vớik =ac,h ∈Z Giả sử k 6=1 cơng thức tổng quát dãy yn = kny0+h

kn−1

k−1 với n. Rõ ràng

k=−1thì dãy số tương ứng không thỏa mãn, ta xétk6=−1.

• Nếuh=0thì ta có yn =kny0, rõ ràng khơng thỏa mãn điều kiện

• Nếu h 6= 0thì gcd ‚

k,k n−1 k−1

Œ

=1 với n nên giả sử giả sử t số mũ lớn mà kt|h thì số nguyên dương có dạng ks với s > t đều khơng ước của số hạng dãy, không thỏa mãn Từ đây, suy k=1hay ac=1 Cuối cùng, ta cần xét2trường hợp:

• NếuP(x) =x+aQ(x) =x+bvớia,b∈Zthì quy nạp, ta chứng minh

x2k =2014+k(a+b)x2k+1=2014+a+k(a+b) Dễ thấy nếua+b=0

thì dãy khơng thỏa mãn Nếu a+b6=0thì gọiS,T,Rlần lượt tập hợp ước nguyên a+b,20142014+a Ta xét trường hợp sau:

+ Với m/Sthì giả sửa+bchiamt vớit 6=0; đó, kchạy qua hệ thặng dư đầy đủ modulo m tồn số hạng dãy chia hết cho m số lượng số hạng vô hạn Rõ ràng số số hạng 0của dãy hữu hạn (không quá2) nên tồn số hạng khác0của dãy chia hết chom.

+ Vớim Sm/ T,Rthì dãy số tương ứng không thỏa mãn

+ Với m S m T m R tương ứng, số hạng có số chẵn số hạng có số lẻ dãy chia hết cho m dễ thấy, tồn số hạng khác 0 dãy chia hết cho m. Do đó, số a,b phải thỏa mãn

(S\T)∩(S\R) =∅hay ước củaa+bphải ước của2014hoặc ước

2014+a.

• Nếu P(x) =−x +aQ(x) =−x +bthì có lập luận tương số hạng dãy tương ứng làx2k =2014−k(ab)x2k+1=−2014+a+k(ab)

Điều kiện a,bab =6 0 ước abphải ước 2014hoặc ước a−2014.

Vậy tất đa thứcP(x),Q(x)cần tìm

P(x) = x+a,Q(x) =x +btrong a,b∈Zthỏa mãna+b6=0 ước

a+bphải ước của2014hoặc ước của2014+a.

P(x) =−x +a,Q(x) =−x+btrong a,b∈Zthỏa mãn ab6=0 ước

(101)

Việt Nam Team Selection Test 2014 18

Nhận xét.

Đây tốn liên quan đến tính chất số học đa thức Trong tốn này, có tính chất sau hữu dụng: Nếu P(x) đa thức có hệ số ngun

(ab)|(P(a)−P(b)) với a 6= b Học sinh phải vận dụng tốt tính đơn điệu đa thức biến đủ lớn chút khéo léo việc xử lí kĩ thuật vấn đề nhỏ Bài toán yêu cầu cẩn thận, chi tiết; suy luận cách logic, khoa học, có thứ tự hợp lý Có thể nói tốn hay, lạ có nhiều “bẫy”

Thoạt nhìn vào tốn này, ta thấy khơng dễ hình dung mơ hình cách tiếp cận có tới đa thức không liên quan đến xuất Một suy nghĩ tự nhiên nên giải thử với trường hợpP,Qtrùng

Khi ta có tốn với đa thức cách tiếp cận dễ nhìn hơn, chí học sinh xử lý tốt có biết toán sau:

(Arthur Angel, Problem Solving Strategies)Cho đa thứcP(x)với hệ số nguyên thỏa mãn điều kiệnP(n)>n với số nguyên dươngn Xét

x1=1,x2=P(x1),x3=P(x2),· · ·

Biết với số nguyên dươngN,tồn số hạng dãy số chia hết choN Chứng minh rằngP(n) =n+1

Ta có xn+2−xn+1chia hết cho xn+1−xn với n, ta suy phần tử dãy số xn+1,xn+2,xn+3 khi chia cho xn+1−xn có số dư

Cùng với tính tăng đa thức, ta thấy xn+1 “quá lớn” so với xn đa thức tương ứng thỏa mãn điều kiện toán Cách tiếp cận hẳn mang lại nhìn vừa bao quát, vừa đơn giản cho số5.

Lời giải cố gắng làm sáng tỏ tất vấn đề đặt toán nên tương đối dài Ta thấy tư tưởng loại trường hợp bậc0của đa thức để đưa xét trường hợp quan trọng:

• Ít hai đa thức có bậc lớn 1. • Bậc hai đa thức bằng1.

Tuy nhiên, ta cần hiểu nguyên nhân cần phải chia thế, tức lập luận trường hợp (1) lại không áp dụng cho trường hợp (2)? Câu trả lời sau: cách lập luận ở(1)đòi hỏi cần có hai bất đẳng thức sau

|Q(P(x))−x|>|x| |Q(P(x))−x|>|P(x)|

đều phải (*) Khi đó, dùng tính chất đơn điệu đa thức để số nguyên dươngm không ước số hạng khác0nào dãy dẫn đến mâu thuẫn

(102)

Việt Nam Team Selection Test 2014 19

Áp dụng lập luận tương tự trường hợp(1),ta giải trường hợp(2)như sau: Nếu hệ số cao hai đa thứcP(x),Q(x)có giá trị tuyệt đối lớn bằng3thì tính chất (*)

Nếu hệ số cao hai đa thức P(x),Q(x) khơng vượt q ta xét hiệu xn+4−xn lập luận tương tự

Các bạn thử từ ý tưởng để tìm cách lập luận ngắn gọn cho toán trên! Về hướng mở rộng, thử xem xét toán trường hợp thay2đa thức 3đa thức sau:

Tìm tất đa thức P(x),Q(x),R(x) có hệ số ngun thỏa mãn điều kiện: Với dãy số

(xn) xác định x0 =2014,x3n+1= P(x3n),x3n+2=Q(x3n+1),x3n+3 =R(x3n+2),n ¾0,

thì số nguyên dươngm ước số hạng khác dãy(xn).

Các bạn thử xem xét đặc điểm chung khó tốn tốn tổng qt thay2, 3bởi k¾4đa thức so với tốn gốc nhé!

Bài 6.

Cho m,n,p số tự nhiên không đồng thời 0. Không gian tọa độ chia thành mặt phẳng song song cách Một cách điền vào khối lập phương đơn vị số từ 1 đến 60 gọi cách điền Điện Biên thỏa mãn: hình hộp chữ nhật với mặt hệ mặt cho tập hợp kích thước ba cạnh (số hình lập phương trên cạnh) xuất phát từ đỉnh 2m+1, 2n+1, 2p+1, khối lập phương đơn vị có tâm trùng với tâm hình hộp chữ nhật điền số trung bình cộng số điền tâm 8 hình lập phương góc hình hộp Hỏi có tất cách điềnĐiện Biên? Những cách điền giống cách, ta chọn hình lập phương làm tâm số điền vào khối lập phương đơn vị có vị trí tương tâm hai cách điền

Lời giải Khơng tính tổng quát, giả sử mặt phẳng song song hệ mặt cách khoảng đơn vị Ta chọn hình lập phương lấy tâm O làm tâm khơng gian vng góc, trục O x,O y,O z song song với đường thẳng hệ cho chiều dương trục tọa độ chọn tùy ý Khi đó, ta quy ước tọa độ hình lập phương hệ mặt cho khoảng cách từ hình chiếu lên trục O x,O y,O z đếnO(chọn dấu tùy vào chiều âm hay chiều dương trục tọa độ) Xét cách điền có tính chấtĐiện Biên ta gọi hai hình lập phương phải điền số cách điền cóquan hệ với Để đếm số cách điền Điện Biên,ta đếm số lớn hình lập phương đơn vị chọn cho hình đơi khơng có quan hệ với nhau, đặtSlà số lượng hình

Trước hết, ta chứng minh với hình hộp chữ nhật có kích thước cạnh 2m+1, 2n+1, 2p+1thì số điền góc số điền tâm

(103)

Việt Nam Team Selection Test 2014 20

kích thước2m+1, 2n+1, 2p+1và hình cóAlà tâm có8số điền ở8ơ đỉnh khơng nhỏ hơnamà lại có trung bình cộng a Suy ra8số điền đỉnh số điền tâm hình hộp chữ nhật giống Lập luận tương tự, ta mở rộng khơng gian tập hợpT hình lập phương điền số a

Với hình lập phương khơng thuộcT, giả sử b>alà số nhỏ điền cho hình lập phương cịn lại tương tự trên, ta tập hợpT0 gồm hình lập phương điền

sốb Rõ ràng thế, ta thấy nhận xét chứng minh điều kiện cần để cách điền có tính chất Điện Biên

Như thế, hình nằm tọa độ(x,y,z)sẽ có quan hệ với hình nằm tọa độ

(x+ (−1)rm,y+ (−1)sn,z+ (−1)tp).

Các số r,s,t ∈ {0; 1} chọn cách độc lập với (là đỉnh hình hộp chữ nhật có tâm (x,y,z) cạnh 2m +1, 2n+1, 2p+1) Hơn nữa, ta xét hình hộp chữ nhật có cạnh2m+1, 2n+1, 2p+1tương ứng song song với mặt phẳng O y z,O z x,O x y ta hồn tồn hốn đổi thứ tự

Từ suy ra, hình lập phương tọa độ(x,y,z)sẽ điền số giống với hình lập phương tọa độ x+a1m+a2n+a3p,y+b1m+b2n+b3p,z+c1m+c2n+c3p

Tuy nhiên, với hai hình có quan hệ với nhau, xét thay đổi thành phần x,y,z tương ứng đại lượng m,n,p tăng, giảm không giữ ngun lần tính chẵn lẻ củaa1+a2+a3,b1+b2+b3,c1+c2+c3đồng thời thay đổi Ban đầu,

xét vị trí(x,y,z) ba tổng tính chẵn lẻ (cùng 0) nên tính chẵn lẻ a1+a2+a3,b1+b2+b3,c1+c2+c3là ln giống

Một hệ nhận xét là: hình lập phương (x,y,z) có quan hệ với hình lập phương tọa độ (x +x1,y + y1,z +z1) với (x1,y1,z1) hoán vị số

(2k m, 2kn, 2k p)k ∈Z,d m,n,p (∗)

Hơn nữa, theo định lý Bezout tổ hợp tuyến tính m,n,p, ta có d = gcd(m,n,p) > 0 tổ hợp tuyến tính có giá trị tuyệt đối nhỏ Từ suy hình lập phương đơn vị hình lập phương lớn có kích thướcd×d×d đơi khơng có quan hệ với (vì hai hình lập phương đơn vị hình có chênh lệch tọa độ nhỏ hơnd) Từ suy raSlà bội củad3.

(104)

Việt Nam Team Selection Test 2014 21

Đặtm1=

m d ,n1=

n d,p1=

p

d, ta xét trường hợp sau:

• Nếu ba số m1,n1,p1 lẻ, đặtm1=2m2+1,n1=2n2+1,p1=2p2+1thì rõ

ràng m=2d m2+d,n =2dn2+d,p=2d p2+d

Do đó, theo(∗),hình lập phương tọa độ(m,n,p)sẽ có quan hệ với hình tọa độ (m±d,n±d,p±d)(các hình kề đỉnh với A) Trong mơ hình trên, ta chọn hình A 3 hình kề mặt với nó, chiều hình 4 hình đơi khơng có quan hệ với khơng thể chọn số lượng lớn Từ đó, ta có S=4d3.

• Nếu ba số m1,n1,p12 số lẻ 1 số chẵn, ta giả sửm1,n1 lẻ p1

chẵn Ta lại đặtm1=2m2+1,n1=2n2+1,p1=2p2thì rõ ràngm=2d m2+d,n=

2dn2+d,p=2d p2

Do đó, theo(∗),hình lập phương tọa độ(m,n,p)sẽ có quan hệ với hình tọa độ (m±d,n±d,p) (các hình kề cạnh vớiA) Ta lại hốn đổi vị trí m,n,p, tức thay đổi chiều (x,y,z)thì suy thêm(m,n±d,p±d)(m±d,n,p±d) có quan hệ vớiA.Trong mơ hình trên, ta chọn hình A1hình kề mặt với hai hình khơng có quan hệ với chọn số lượng lớn 2 Từ đó, ta cóS=2d3.

• Nếu ba số m1,n1,p11 số lẻ 2 số chẵn, ta giả sửm1 lẻ n1,p1

chẵn Ta lại đặt m1 =2m2+1,n1=2n2,p1 =2p2 rõ ràngm =2d m2+d,n =

2dn2,p=2d p2

Do đó, theo(∗),hình lập phương tọa độ(m,n,p)sẽ có quan hệ với hình tọa độ(m±d,n,p)(các hình kề mặt vớiA) Ta lại hốn đổi vị trí củam,n,p, tức thay đổi chiều của(x,y,z) suy thêm(m,n±d,p)(m,n,p±d)có quan hệ với A.Trong mơ hình trên, ta chọn hìnhAchứ khơng chọn thêm hình khơng có quan hệ với Từ đó, ta cóS=d3.

Cuối cùng, ta thấy độc lập với có 60 cách điền số (từ 1 đến 60) cho nên số cách điền Điện Biên tổng cộng 60S với S xác định sau: Trong ba số

m gcd(m,n,p),

n gcd(m,n,p),

p

gcd(m,n,p),nếu có • 3số lẻ thìS=4d3.

2số lẻ thìS=2d3.

1số lẻ thìS=d3.

Nhận xét.

(105)

Việt Nam Team Selection Test 2014 22

thí sinh nhanh chóng nhận số60khơng có nhiều ảnh hưởng việc tìm lời giải

Bài tốn lạ khó Có thể nói xử lý tốn đếm tổ hợp cơng việc đốn đáp số đốn thành cơng nhiều thứ dễ dàng Điểm bất ngờ lại đáp số mà thực tế, có nhiều đáp số chưa hồn tồn xác đưa Điều có lẽ xuất phát từ việc xử lí hai điều kiện chuyển dạng số học cuối khơng cẩn thận Nếu thực bình tĩnh, có thời gian, ta xét thử vài ví dụ nhỏ vớim,n,p để thấy có kết “tương đối khác nhau” không dạng

Tuy nhiên, phòng thi, với việc bài5làm khơng thời gian, thực thử thách không dễ

Đây tốn hay, có nhiều đất cho bạn học sinh thể khả Bài tốn địi hỏi tư đơn giản, tinh tế phần suy luận tổ hợp, cẩn thận, khoa học phần xử lí điều kiện số học cuối Nếu chưa có hình dung tốt khơng gian 3chiều, ta hồn tồn chọn cách tiếp cận từ việc, đưa tốn về2( xét mặt phẳng hình chữ nhật 2 cạnh lẻ) hay chí 1 chiều ( đường thẳng đoạn thẳng chứa lẻ điểm) Cụ thể đây:

Chok,a số nguyên dương Trên trục số nguyên, người ta đánh số điểm nguyên số từ1 đến k cho: Mỗi đoạn thẳng có hai đầu mút nguyên có chứa 2a+1điểm ngun trung điểm đánh số trung bình cộng số đánh hai đầu mút Hỏi có tất cách đánh số khác nhau?

Bằng lập luận theo nguyên lý cực hạn tương tự lời giải toán gốc, ta thấy rằng: • Với đoạn thẳng có chứa 2a+1 điểm nguyên số đánh cho 2 đầu

mút với số đánh cho trung điểm

• Các điểm cách hơnađơn vị độ dài khơng có ràng buộc với Như ađiểm có tọa độ từ 0, 1, 2, ,a−1 đơi đánh số độc lập với kết toán làka.

Tiếp tục phát triển lên, ta xét toán sau:

Chok,a,blà số nguyên dương Trên trục số nguyên, người ta đánh số điểm nguyên số từ1 đến k cho: Mỗi đoạn thẳng có hai đầu mút nguyên có chứa 2a+1hoặc2b+1điểm ngun trung điểm đánh số trung bình cộng số đánh hai đầu mút Hỏi có tất cách đánh số khác nhau?

Tiếp tục lập luận trên, ta thấy vị trí0,a, 2a,b, 2bđược đánh số hay nói cách khác, vị trí cách nhauahoặc bđơn vị đánh số Từ suy tất điểm vị trí x a+y bvớix,y∈Zx,y khơng đồng thời bằng0sẽ đánh

cùng số với

(106)

Việt Nam Team Selection Test 2014 23

Bằng cách tương tự, việc thay2sốa,bthành nhiều số giải tương tự tốn ln có kết theo dạng lũy thừa Tuy nhiên, thay tốn2chiều, ta lại có đến2kết quả: Chok,a,blà số nguyên dương Trong mặt phẳng tọa độ, người ta đánh số điểm nguyên số từ đếnksao cho: Mỗi hình chữ nhật có cạnh song song với2trục tọa độ cạnh có2a+1, 2b+1điểm nguyên trung bình cộng4số điền đỉnh số điền tâm Hỏi có tất cách đánh số khác nhau?

Kết toán là: Đặtc=gcd(a,b), hai số a c,

b c có: • 2số lẻ kết làk2c2

.1số lẻ kết làkc2

(107)

Việt Nam Team Selection Test 2014 24

Tài liệu tham khảo

1 Topic đề thi bình luận đềVN TST 2014của diễn đànmathscope.org

http://forum.mathscope.org/showthread.php?t=46988

2 Trần Quang Hùng, Xung quanh hình học kỳ thi chọn đội tuyển VN ngày http://analgeomatica.blogspot.com/2014/03/xung-quanh-mot-bai-toan-hinh-hoc-trong.html

3 Trần Quang Hùng, Xung quanh hình học kỳ thi chọn đội tuyển VN ngày

http://analgeomatica.blogspot.com/2014/03/tom-tat.html

4 Nguyễn Thị Hường, Lương Ánh Nguyệt, Lương Thị Thanh Mai, Đào Thị Quỳnh Nga, Định lý Sawayama Thébault

http://analgeomatica.blogspot.com/2014/02/inh-ly-sawayama-va-thebault.html

5 Functional equationAPMO 1989

http://www.artofproblemsolving.com/Forum/viewtopic.php?p=450331

6 Bình luận củaGS Nguyễn Tiến Dũngtrên trangSputnik Education

https://www.facebook.com/sputnikedu/posts/720502957971422

7 Bài toán IMO Shortlist 1994, G4

http://www.artofproblemsolving.com/Forum/viewtopic.php?p=352892

8 Mẫu Latex thầyChâu Ngọc Hùng

https://www.writelatex.com/read/htndbgqrjqzp

(108)(109)(110)(111)(112)(113)(114)(115)(116)(117)(118)(119)(120)(121)(122)(123)(124)(125)(126)(127)(128)(129)(130)(131)(132)(133)(134)(135)(136)(137)(138)(139)(140)(141)

Epsilon

staff

LỜI GIẢI VÀ BÌNH LUẬN ĐỀ THI CHỌN ĐỘI TUYỂN IMO 2017

Trần Nam Dũng – Võ Quốc Bá Cẩn – Trần Quang Hùng Lê Phúc Lữ - Nguyễn Tất Thu

1 Lời nói đầu

Dù Epsilon nói lời tạm biệt với bạn đọc từ ngày 13/2/2017 tinh thần Epsilon đội ngũ Epsilon cịn Và có nghĩa sản phẩm mang tinh thần Epsilon đời Tinh thần ngắn gọn là:Chuyên nghiệp – Từ cộng đồng – Vì cộng đồng.

Minh chứng cho tinh thần tài liệu mà bạn đọc“Giải bình luận đề thi chọn đội tuyển Việt Nam dự thi Tốn Quốc tế 2017”, đóng góp đội ngũ Epsilon dành cho cộng đồng Khi viết đội ngũ Epsilon, không muốn nhắc đến người lính ngự lâm thuộc Ban biên tập (Epsilon staff) mà cịn người ln sát cánh suốt năm qua trình xây dựng Epsilon thành niềm yêu mến chờ đợi cộng đồng

Giải bình luận đề thi, không muốn đem lại cho độc giả lời giải, đáp án để so khớp sai mà phân tích hướng tiếp cận, nguồn gốc, lớp tốn tương tự Chúng tơi mạn phép đưa bình luận chủ quan hay, dở, độ khó dễ, tính phù hợp, độ cũ toán giúp cho thầy ban đề có thêm ý kiến phản biện, để công tác đề thi ngày tốt hơn, chất lượng Hy vọng tập tài liệu nhận đón nhận cộng đồng Chúng tơi ln lắng nghe ý kiến đóng góp, trao đổi thẳng thắn bạn đọc nội dung tài liệu vấn đề liên quan Chúng ta cộng đồng

If you want to go far, go together.

2 Thông tin quyền

Bản quyền thuộc tất thành viên nhóm biên soạn (Trần Nam Dũng, Võ Quốc Bá Cẩn, Trần Quang Hùng, Lê Phúc Lữ, Nguyễn Tất Thu)

Đây thành trình lao động miệt mài nhóm để chia sẻ đến cộng đồng Mọi người xem tài liệu MIỄN PHÍ Tuy nhiên, vui lòng ghi rõ nguồn chia sẻ

Tất hoạt động mua bán, kinh doanh liên quan đến tài liệu mà không chấp thuận nhóm trái pháp luật Chúng ta lên án hành vi vi phạm quyền để bảo vệ quyền lợi tác giả, sản phẩm trí tuệ Xin cảm ơn

(142)

Epsilon

staff

2 Lời giải bình luận đề thi chọn đội tuyển IMO 2017

3 Đề thi

3.1 Ngày thi thứ (25/03/2017)

Bài (7.0 điểm) Cho44cái lỗ phân biệt rãnh đường thẳng và2017con kiến Mỗi kiến chui lên từ lỗ bò đến lỗ khác với vận tốc không đổi chui xuống GọiT tập thời điểm mà kiến chui lên chui xuống lỗ Biết vận tốc kiến đôi khác vàjTj 45:Chứng minh tồn hai kiến khơng gặp (Quy ước hai kiến gặp tồn thời điểm mà hai vi trí rãnh kể lúc chui lên xuống.)

Bài (7.0 điểm) Với số nguyên dươngn, đặtxn DC2nn :

a) Chứng minh 72 k < n < 7k với k số nguyên dương xn

là bội của2 7:

b) Tìm tất số nguyên dương h > 1để tồn số nguyên dươngN ; T cho với

n > N thìxn dãy số tuần hoàn theo modulohvới chu kỳT :

Bài (7.0 điểm) Cho tam giácA B C ngoại tiếp đường tròn I /và I /tiếp xúc với cạnh

B C ; C A ; A B D ; E ; F : GọiIb; Ic tâm đường trịn bàng tiếp góc

B ; C tam giác A B C :GọiP ; Qlần lượt trung điểm IbE ; IcF : Giả sử P A C /

cắtA B điểm thứ haiR QA B / cắtA C điểm thứ hai S :

a) Chứng minh rằngP R ; QS ; A I đồng quy

b) Giả sử DE ; DF cắt IbIc K ; J : E J cắt F K M P E ; QF cắt

P A C / ; QA B /lần lượt điểm thứ hai X ; Y :Chứng minh ba đường thẳng

B Y ; C X ; A M đồng quy

3.2 Ngày thi thứ hai (26/03/2017)

Bài (7.0 điểm) Cho tam giácA B C nội tiếp đường tròn O / : Điểm Adi động O /

sao cho A B > B C M trung điểm A C :Đường trịn đường kính B M cắt O /

R :Giả sử R M cắt O / điểm thứ hai Q ;cắt B C P :Đường trịn đường kínhBP cắt

A B ; B O điểm thứ hai K ; S :

a) Chứng minh rằngSRđi qua trung điểm củaKP :

b) Gọi N trung điểm B C : Trục đẳng phương hai đường trịn đường kính

A N ; B M cắtSR tạiE :Chứng minh rằngM E qua điểm cố định

Bài (7.0 điểm) Cho số thực dươnga1; a2; : : : ; a2 7: Với mỗin > 7; ta đặt

an D max˚ai1ai2ai3 ji1 C i2 C i3 D n ; i1 i2 i3 n :

Chứng minh tồn số nguyên dương m không vượt số nguyên dương

(143)

Epsilon

staff

Lời giải bình luận đề thi chọn đội tuyển IMO 2017 3

Bài (7.0 điểm) Với số nguyên dương n, xét a1; a2; : : : ; a2 n hoán vị nsố

nguyên dương Một hoán vị gọi “đẹp” với mọi1 i < j n

thìai C anCi D nC1 vàai aiC1không đồng dư vớiaj ajC1 theo modulo nC1

(Quy ước a2 nC1 D a1:)

a) Với n D 6, hoán vị đẹp

b) Chứng minh với mỗin ngun dương ln tồn hốn vị đẹp

4 Bình luận chung

Trong hai ngày 25, 26/3/2017 diễn kỳ thi chọn đội tuyển Việt Nam dự thi Toán Quốc tế năm 2017 với tham dự 49 học sinh xuất sắc đến từ tỉnh thành trường chuyên thuộc trường đại học lớn nước Mỗi ngày thi, thí sinh phải giải tốn thời gian 240 phút

Đề thi năm đánh giá khó có phong cách gần với đề thi IMO năm trước Cấu trúc đề thi bao gồm hình học, đại số, tổ hợp, số học tổ hợp số học Như đề chọn đội tuyển năm theo xu hướng IMO thiên tổ hợp nhiều Ngay (bài đại số) có cách phát biểu (và cách giải) có nhiều nét tổ hợp Các toán theo đánh giá chung coi dễ ngày Bài hình câu phần a) cần kiến thức chương trình cấp 2, phần b) vận dụng kiến thức trục đẳng phương, ý điểm cố định có đặc biệt so với thơng thường trước Bài số toán số học tuý, khai thác chủ đề quen thuộc học sinh Việt Nam số dư phép chia hệ sốCnk cho số nguyên tốp với định lý Lucas, Wolstenhome, Babbage hay công thức Legendre, vậy, dù mức độ khác nhau, thí sinh tiếp cận

Đáng ý số 1, ban đề thi đánh giá dễ ngày thứ lại gây nhiều khó khăn cho thí sinh Các em phát biểu lại tốn cách rành mạch ngơn ngữ Tốn học nên xử lý được, cịn để tốn nhiều thời gian vào tốn Nếu biết cách chuyển đổi tốn (mơ hình hố) thành đồ thị (theo nghĩa đồ thị) đồ thị (theo nghĩa đường biểu diễn đường kiến) tốn giải cách đơn giản, chẳng hạn quy nạp

Các tốn cịn lại gồm số 3, đánh giá khó Bài hình số phần a) sử dụng kiến thức tam giác đồng dạng tứ giác nội tiếp chương trình cấp 2, ý khó tập trung câu b) địi hỏi vận dụng nhiều kiến thức tổng hợp có liên quan nhiều đến hình G7 IMO shortlist 2002 có sử dụng kiến thức hàng điều hịa Cấu trúc ghép nối toán riêng lẻ thành tổng hợp Bài số tốn có cách phát biểu tổ hợp, dạng dãy số truy hồi với số hạng Để tiếp cận này, học sinh phải tỉnh táo nhận số 2017 đề thay số nguyên dương bắt đầu làm thử với trường hợp tham số nhỏ để dự đoán quy luật dãy số Cũng thấy cách logarith hoá, phép nhân toán biến thành phép cộng tốn đưa biến thể toán số đề thi IMO 2010 Điều lần nhấn mạnh độ khó số 5, đồng thời điểm chưa hay đề thi lần

Bài toán số toán số học-tổ hợp với yêu cầu xây dựng hoán vị củaf1 ; ; : : : ; ng

(144)

Epsilon

staff

4 Lời giải bình luận đề thi chọn đội tuyển IMO 2017

bằng phép thử sai, xây dựng ví dụ tổng quát cho số trường hợp Chẳng hạn nếu2 nC1

có ngun thuỷ đặtai D ˛i mod nC1 /thì rõ ràng hiệuaiC1 ˛i ˛ /

phân biệt modulo2 nC vàai C anCi ˛i C ˛n/ mod nC / :Trong trường

hợp tổng quát, ý tưởng xây dựng n số hạng dãy số, đoạn sau lấy phần bù Tuy nhiên, cần có điều kiện bổ sung anvà a1; vìanC1 D nC a1:Và

cũng điểm mà thí sinh phạm sai lầm ngộ nhận

Với nhận định tham khảo số thông tin làm thí sinh, chúng tơi cho bên cạnh toán thuộc dạng “cần phải làm muốn đậu đội tuyển” vé dự IMO định lại 1, 3, 5, với tình trạng khó đốn Khác với số năm trước có tốn q khó khơng làm được, đề thi năm độ khó chia đều, khó dễ thuỳ theo sở trường em Thực tế có học sinh giải tạo thành “cài lược”, ví dụ có học sinh làm 1, lại khơng làm 3, 5, có học sinh làm lại không làm Ngoài 1, 5, có lỗi trình bày, chí có sai sót lớn ngộ nhận Với đề thi này, làm chắn gần có vé, cịn điểm chuẩn vùng tranh chấp 3.5++

5 Lời giải bình luận tốn

Bài (7.0 điểm) Cho 4cái lỗ phân biệt rãnh đường thẳng và2 kiến Mỗi kiến chui lên từ lỗ bò đến lỗ khác với vận tốc khơng đổi chui xuống Gọi T tập thời điểm mà kiến chui lên chui xuống lỗ Biết vận tốc kiến đôi khác vàjTj :Chứng minh tồn hai kiến khơng gặp (Quy ước hai kiến gặp tồn thời điểm mà hai vi trí rãnh kể lúc chui lên xuống.)

Lời giải Ta xét trục toạ độO x y vớiO x trục đánh dấu vị trí lỗ rãnh thẳng

O y trục thời gian Các lỗ có hồnh độ x1; x2; : : : ; x4 thời điểm mà

kiến chui lên chui xuống lày1; y2; : : : ; y4 5:Một kiến từ điểm xa; yb/đến

điểm xc; yd/tương ứng với tình chui lên từ lỗ xa vào thời điểm yb đến lỗxc vào

thời điểmyd: Do kiến bị với vận tốc khơng đổi nên đồ thị biểu diễn đường theo thời gian

chính đoạn thẳng nối hai điểm Có tất 7đoạn thẳng Vì vận tốc kiến đôi khác nên đoạn thẳng có phương khác Ta cần chứng minh có hai đoạn thẳng số khơng cắt

Chú ý số điểm đầu mút đoạn thẳng là4 4 D < 7nên tốn giải hồn tồn ta chứng minh mệnh đề sau:Nếu cón điểm mặt phẳng thì khơng tạo qnđoạn thẳng với đỉnh điểm cho đơi có điểm chung và khơng có hai đoạn song song hay đè lên nhau.

Ta chứng minh quy nạp theo n :Với n D ; 3mệnh đề hiển nhiên Giả sử mệnh đề vớin 1điểm Xétn điểm:

(145)

Epsilon

staff

Lời giải bình luận đề thi chọn đội tuyển IMO 2017 5

Giả sử đỉnh có hai đoạn xuất phát Nếu có đỉnh Acó ba đoạn xuất phát “có vấn đề”: chẳng hạn làA B ; A C ; A D:Nếu B C Dchứa Abên đoạn thứ hai từB cắtA C vàA D: NếuB C D khơng chứaA bên ta có chẳng hạn tiaA C nằm hai tia A B vàA D: Khi đoạn thứ hai từ đỉnhC

khơng thể cắt A B A D: Như từ đỉnh có hai đoạn, suy số đoạn số đỉnh trường hợp

Bài tốn giải hồn tồn

Bình luận

Đây tốn có cách phát biểu thú vị có lời giải khơng phức tạp thực rối khơng nhìn thấy chất chuyển đổi mơ hình điểm, đoạn thẳng trên, với hai ý quan trọng: kiến bị với vận tốc khơng đổi (các đường biểu diễn quỹ đạo kiến đoạn thẳng) vận tốc đôi khác (các đoạn thẳng không phương) Trên thực tế, nhiều bạn tốn nhiều thời gian cho tốn khơng tiếp cận lời giải Và lời giải, hai điều kiện quan trọng khơng khai thác chắn lời giải khơng hồn chỉnh

Lời giải trình bày GS Nguyễn Tiến Dũng, HCV IMO 1985 chúng tơi trình bày lại cho giống lời giải Còn phần dẫn nhập cách diễn đạt lời giải khác GS Nguyễn Tiến Dũng:

Có liên quan số4 ; 52 7? Dễ thấy 4 D < 7;nên có thể đốn mấu chốt tốn?

Vậy lại dùng tích? Bởi có nhiều điểm vị trí, thời gian chạm lỗ lên xuống lỗ/trên mặt phẳng toạ độ Ta vẽ mặt phẳng toạ độ đồ thị đường kiến, đồ thị đoạn thẳng nối hai số điểm trên. Tất đoạn thẳng có hướng khác nhau.vì vận tốc kiến khác nhau/ : Hướng hiểu góc so với đường nằm ngang modulo :

Câu hỏi đặt hai đoạn có điểm chung (hai kiến có gặp nhau) có nhiều đoạn thẳng?

Ta xếp thứ tự đoạn thẳng theo thứ tự vịng trịn theo góc chúng. Cố định đoạn đầu tiên, từ điểm A1 đến điểm A2: Đoạn thứ hai có hướng quay về “bên phải” so với đoạn thứ nhất, nên phải có thêm điểm mới A3 tức hoặc A1A3 với A3 nằm bên phảiA1A2; hoặc A3A2 với A3 nằm bên trái A1A2; hoặc A3A4 hai điểm mới/nằm hai bên của A1A2/ : Thêm đoạn thứ ba phải thêm nhất một điểm mới, trừ trường hợp tạo thành ba đoạn A1A2; A1A3; A2A3: Cứ thế: thêm đoạn cần thêm điểm mới, trừ thêm đoạn cuối dùng được hai điểm cũ Như để cónđoạn đơi có điểm chung cần nhất nđiểm Với n D 4lần4 5thì có nhiều kiến đơi có gặp Vì2 lớn hơn4 4lần 5nên có hai kiến khơng gặp nhau.

(146)

Epsilon

staff

6 Lời giải bình luận đề thi chọn đội tuyển IMO 2017

các toán đầu cuối, tức toán phát biểu rành mạch ngơn ngữ Tốn học mà lại trọng đến tốn đòi hỏi đọc hiểu, phát chất vấn đề phát biểu lại mơ hình Tốn học

Chúng cung cấp cho bạn đọc số tốn mà lời giải địi hỏi bước đọc hiểu, phát biểu lại mơ hình Tốn học để tham khảo:

1 (IMO Shortlist, 2010)Trong buổi biểu diễn ca nhạc, có2 0ca sĩ trình diễn Với ca sĩ có danh sách (có thể rỗng) ca sĩ khác mà anh cô muốn diễn sau tất ca sĩ thuộc danh sách Có thể xảy có đúng2

cách xếp để tất mong muốn ca sĩ thoả mãn?

2 (Tournament of the towns, 1992-1993)Trong từ điển thực vật loài đặc trưng bởi1 0 dấu hiệu (mỗi dấu hiệu có, khơng có) Hai lồi coi không giống chúng khác nhất5 1dấu hiệu

a) Chứng minh từ điển có khơng q5 0lồi đơi khơng giống

b) Có thể có 0lồi đôi không giống không?

3 (Moscow MO, 2010)Trên mặt phẳng ta đánh dấu4 nđiểm, sau ta nối tất cặp điểm có khoảng cách bằng1cm Người ta thấy trongn C 1điểm ln có hai điểm nối Chứng minh có nhất7 nđược kẻ

4 Ban đầu có số lẻ viên sỏi Ở bước tiếp theo, ta chia số sỏi làm hai, nửa để xuống phía dưới, bên trái, nửa để xuống phía dưới, bên phải, dư viên để xuống vị trí đống sỏi ban đầu Ở lượt tiếp theo, ta lại làm với đống sỏi Như số vị trí bước nhận sỏi từ ba đống sỏi nó: phía trên, phía bên trái phía bên phải

Ví dụ, bắt đầu đống sỏi2 1viên lượt chia ta có ba đống sỏi

1 1 :

Ở lượt chia thứ hai ta có

5 1 :

Và lượt chia thứ ba

2 7 :

Chứng minh tiếp tục chia đống sỏi có số lẻ viên, đến lúc ta đến trạng thái có đống sỏi viên liền

Bài (7.0 điểm) Với số nguyên dương n, đặtxn D C2 nn :

a) Chứng minh 72 k < n < 7k với k số nguyên dương

xn bội 7:

(147)

Epsilon

staff

Lời giải bình luận đề thi chọn đội tuyển IMO 2017 7

Lời giải a)Ta chứng minh kết luận toán cho số nguyên tốp lẻ thay cho số2 7:

Giả sử tồn số nguyên dươngk cho p2k < n < pk:Ta có

vp xn/ D vp C2 nn

D vp n / Š

2 vp n Š / :

Do p2k < n < pk nênpk < n < pk < pkC1;từ suy

vp n Š /

D n p C n p2

C C

2 n pk

:

Mặt khác, với x R;ta có b2 xc 2bxc;đẳng thức xảy fxg < 12:Kết hợp với

giả thiết p2k < n < pk;ta có

vp n / Š

> n p C n p2

C C

n

pk D vp n Š / ;

hay vp xn/ > 0nên ta cóxn::: p :

b)Ở ý này, ta có hai cách tiếp cận sau

Cách (Dựa theo lời giải Hà Duy Hưng)Giả sửh > 1là số thoả mãn yêu cầu toán Với mỗip nguyên tố lẻ màp j h, ta có dãy số dư xn theo modulop tuần hoàn Sử dụng

kết câu a) với p2k < n < pk

xn mod p / :

Chọnk đủ lớn để p2k > T C 1, ta suy tất số dư củaxn cho p 0với

n n0, đón0 ZC đủ lớn Tuy nhiên chọnt ZC đủ lớn để pt > n0 đặt

n D pt2 ta có ngayvp xn/ D 0, đóxnkhơng chia hết cho p ; vơ lý

Vậy h có ước nguyên tố hay h D 2k với k nguyên dương Nếu k > ; ta chọn

r D k xét sốn có dạngn D 2a1 C C2ar;trong đóa

1 > maxfT ; Ng

T ; N số giả thiết củah :Khi

v2 xn/ D S2 n / S2 n / D r;

trong đóS2 x /là tổng chữ số biểu diễn nhị phân Thành thửxn 2k mod h /

Tuy nhiên, với i ZC mà i < 2a1 số chữ số biểu diễn nhị phân n C i

tăng thêm đơn vị, mà xnCi mod h / Do a1 > maxfT ; Ng nên

xn xnCT mod h /, mâu thuẫn

Vậy k D h D Đây đáp số tốn, dễ thấy xn số chẵn với số

nguyên dươngn ; sau: nếun D 2a1 C C 2ar với0 a

1 < < ar

2 n D 2a1C1 C C 2arC1

nên

v2 xn/ D Sn n / S2 n / D r :

(148)

Epsilon

staff

8 Lời giải bình luận đề thi chọn đội tuyển IMO 2017

Cách 2.(Theo lời giải Nguyễn Song Minh)Doh > ; nên giả sử p ước nguyên tố lớn củah : Giả sử T D pmT0 chu kỳ nhỏ ứng với n đủ lớn, với T0; p / D :

Theo định lý Lucas, ta có

C2 nn C2 ppmmnn C

pmnCT

2 pmnC2 T C

nCT0/ pm

nC2 T0/ pm C

nCT0

2 nC2 T0 mod p / :

Suy m D :Nếup lẻ giả sửT D a1a2 ar/p với R > r đủ lớn, ta có

C21 C2 ppRR C

pRCT

2 pRC2 T C

1 2: C

T

2 T mod p / :

Suy CT

2 T mod p /, từ ta có vớiK đủ lớn

C2 K TK T mod p / :

Mặt khác T ; p / D 1nên tồn số nguyên dương K đủ lớn chop j K T C / Kết hợp với K T C / j C2 K TK T ;ta thu modp / điều vơ lý, đóh D 2k:

VìT lẻ theo lý luận trên, ta chọn K choh j nC K T C1 /

C2 nn C2 nnCCK T2 K T mod h / ;

với nđủ lớn Ta chọnn D 2N, ta có số số lẻ dãyCnk k n /bằng2 nên

C2 nn mod / ;

do h D : Hơn C2 nn số chẵn với số nguyên dươngn :Vậyh D :

Bình luận

Một lần chủ đề tính chất số học hệ số nhị thứcCnk lại khai thác Đó

điều dự đốn năm nguyên tố (như 2003 hay trước nhiều năm 1979) không ngờ năm điều lại ưu đến hai lần (VMO số số kỳ TST này) Vì vậy, với công cụ trang bị đầy đủ công thức Legendre, định lý Lucas, Lagrange, Babbage hay Wolstenhome tốn này, đặc biệt câu a) khơng gây khó khăn cho bạn học sinh

Nhắc lại định lý Babbage đồng thức Cbpap Cba mod p2/ ; cho số nguyên tố p ; định lý Wolstenhome dạng hệ số nhị thức đồng thức

Cbpap Cba mod p3/chỉ cho số nguyên tốp > :

Sự kiệnC2 nn chẵn tính chất quen thuộc có nhiều cách chứng minh Hơn ta có số mũ củantrong C2 nn số số1trong biểu diễn nhị phân n ;cho nên với

h D 2s có h D 2là thoả mãn điều kiện

Chúng dẫn số toán thi Olympic năm gần có sử dụng nhóm định lý

1 (Vietnam TST, 2010)Gọi Sn tổng bình phương hệ số sau khai triển

(149)

Epsilon

staff

Lời giải bình luận đề thi chọn đội tuyển IMO 2017 9

2 (Saudi Arabia TST, 2015)Với số nguyên dươngn ; đặt

S n / D

n

X

kD0

rk;

trong đórk số dư phép chiaCnk cho :Tìm tất số nguyên dương

nsao cho S n / n :

3 (Trường Xuân Toán học miền Nam, 2017)Cho dãy số xn/xác định công thức:

x0 D ; xn D

x0 C x1 C Cxn

n

với n 7:

Hỏi có sối mà0 i chojxijchia hết cho 3?

Bài (7.0 điểm) Cho tam giácA B C ngoại tiếp đường tròn I /và I / tiếp xúc với cạnhB C ; C A ; A B tạiD ; E ; F :GọiIb; Ic tâm đường trịn bàng

tiếp gócB ; C tam giácA B C :Gọi P ; Qlần lượt trung điểm củaIbE ; IcF : Giả

sử P A C /cắt A B điểm thứ hai R QA B /cắt A C điểm thứ haiS : a) Chứng minh rằngP R ; QS ; A I đồng quy

b) Giả sử DE ; DF cắt IbIc K ; J : E J cắt F K M vàP E ; QF cắt P A C / ; QA B / điểm thứ hai X ; Y : Chứng minh ba đường thẳngB Y ; C X ; A M đồng quy

Lời giải Ta dùng bổ đề sau để giải toán

Bổ đề Cho tam giácA B C có đường trịn nội tiếp I /tiếp xúc với cạnhB C ; C A ; A B lần lượt tại D ; E ; F :Gọi M ; N lần lượt trung điểm củaDE ; DF :Gọi P là giao điểm của đường trung trực củaM C N B :

a) Chứng minh đường tròn P B C / tiếp xúc với đường tròn I /.

b) GọiJ là điểm đối xứng vớiD qua P :Chứng minh rằng J là tâm đường trịn bàng tiếp gócA của tam giácA B C.

c) Chứng minh giao điểm của P M A C nằm đường tròn P B C /.

Chứng minh a)Gọi G giao điểm thứ hai củaP D I / : Dễ thấy trung trực C M trục đẳng phương đường tròn điểmC đường tròn I / ;và trung trựcB N trục đẳng phương đường tròn điểm B đường trịn I / : Do

P C2 D P D P G D P B2:

Từ suy ∠P G C D ∠P C B ∠P GB D ∠P B C nên tứ giácP B G C nội tiếp đường tròn K / Lại có I D k KP KPI D D K GI G nên ba điểm G ; I ; K thẳng hàng Do đó, K /

(150)

Epsilon

staff

10 Lời giải bình luận đề thi chọn đội tuyển IMO 2017

A

B C

I

K F

E

D R

P N

Q

M G

J

b)Do GB đường đối trung tam giácGF D nên

∠F G N D ∠B GD D ∠P B C

∠GF N D ∠GD C D ∠P DB :

Từ suy hai tam giác GF N BDP đồng dạng g-g Lại có N ; P trung điểm

DF ; D J nên hai tam giácGF D BD J đồng dạng c-g-c Từ suy

∠DB J D ∠D GF D ∠BDF

nênB J k DF : VậyB J phân giác ngồi gócB :

Tương tự, ta có C J phân giác ngồi gócC nênJ tâm đường trịn bàng tiếp gócA

c)Dễ thấy tứ giácB C M N nội tiếp tâm nội tiếp giao điểmP trung trực C M ; B N :Từ suy

P M2 D P C2 D P D P G :

Suy hai tam giác P DM P M G đồng dạng Từ

∠DM P D ∠D GM D ∠E G C :

Lại có

∠GE C D 0ı ∠GE A D 0ı ∠GDE D ∠P DM D ∠P M G :

(151)

Epsilon

staff

Lời giải bình luận đề thi chọn đội tuyển IMO 2017 11

Bổ đề Cho tam giácA B C :Đường tròn nội tiếp I ; r /tiếp xúc với cạnhB C ; C A ; A B lần lượt tạiD ; E ; F : Một đường tròn qua B ; C và tiếp xúc I /tạiX :Khi đó, ta có

a) XE XF XD2 D

r2 I B I C : b) XE

XF D

I B DE2 I C DF2:

Chứng minh Gọi giao điểm tiếp tuyến chung hai đường tròn tiếp xúc tạiX vàB C

M GọiG điểm đối xứng D quaM M G2 D M D2 D M X2 D M B M C nên

B C ; D G / D ;do đóEF quaG :Gọi A Llà đường cao tam giácA B C theo tốn quen thuộc I G ? A D nên hai tam giácI GD DA Lđồng dạng

A B C I F E D K G X M L Z H

Lại có DX ? I M M trung điểm GDnên DX qua trung điểmA L : GọiDK đường kính I / chùm D A L ; X K / D 1, chiếu lên I /dẫn tới tứ giác DK Z X

điều hòa vớiZ giao điểm củaA D I / Từ suy tiếp tuyến tạiZ ; D vàX K đồng quy tạiG Nói cách khác, X K qua G Gọi H giao điểm A D EF :Ta có

XE XF

K E K F D

GE GF D

H E H F D

Œ A ED  Œ A F D  D

DE DF

DB I C

I B D C :

Lại có K E I C D r2 D K F I B nên

XE XF D

I B DE2 I C DF2:

Mặt khác, ta có

F X F K

E X E K D

GX GK D

GD2 GK2 D

(152)

Epsilon

staff

12 Lời giải bình luận đề thi chọn đội tuyển IMO 2017

và K E I C D r2 D K F I B nên

XE XF XD2 D

r2 I B I C :

Bổ đề chứng minh

Từ bổ đề trên, ta suy hai hệ

XE2 XD2 D

r2 DE2 I B2 DF2

XF2 XD2 D

r2 DF2 I C2 DE2:

Bổ đề Cho tam giác A B C ; đường tròn nội tiếp I / tiếp xúc vớiB C tại D Dựng một đường tròn qua A ; B tiếp xúc với I /tạiM và đường tròn quaA ; C tiếp xúc với I /tại N : B M cắtC N tại P :Chứng minh rằng∠P A B D ∠DA C :

Chứng minh Gọi tiếp điểm I /vớiC A ; A B E ; F vàX Y Z tam giác Ceva củaP GọiK giao điểm củaB Y vàDF : Sử dụng bổ đề hệ với ý rằngM K

là đường đối trung tam giácM DF ;ta có

Y C Y A D

Œ Y B C  Œ Y BA  D

Œ Y B C  Œ K BD 

Œ K BD  Œ K BF 

Œ K BF  Œ Y BA  D B C B Y

BD BK K D K F

BF BK B Y BA D B C

BA

M D2 M F2 D

B C BA

r2 DF2 EF2 I A2:

Tương tự, ta có

ZB Z A D

B C C A

r2 DE2 EF2 I A2:

(153)

Epsilon

staff

Lời giải bình luận đề thi chọn đội tuyển IMO 2017 13

Sử dụng định lý Ceva, ta có

XB X C D

Y A Y C

ZB Z A D

A B : DE2 A C : DF2:

Do

DB D C

XB X C D

p b

p c

A B DE2 A C DF2 D

DF I B DE I C

A B DE2 A C DF2 D

A B2 A C2:

Từ suy raA D A X đẳng giác trong∠BA C :

Trở lại toán:

a) Theo bổ đề phần a), b), ta thấy đường tròn P A C /và QBA / tiếp xúc với I / :

Từ đó, tiếp tục theo bổ đề phần c) thìP R ; QS qua trung điểmN EF hiển nhiên M nằm A I :

A B C Ib Ic I F E P Q S R D K J Y X Z M N

b)Cũng theo bổ đề phần a), ta thấyX ; Y tiếp điểm đường tròn P C A / ; QA B /với I / GọiZ giao điểm củaB Y C X :Theo bổ đề 3, ta thấy

∠DA B D ∠Z A C :

Ta chứng minh∠DA B D ∠M A C từ suy A ; M ; Z thẳng hàng Thật vậy, ta dễ thấy ∠E A J D 0ı

2∠BA C D ∠EDF nên tứ giácK A ED nội tiếp

Chứng minh tương tự, tứ giácJA F D nội tiếp Từ dễ thấy ∠E K A D ∠J DA D ∠J F A

nên A J E M nội tiếp Tương tự, tứ giácA K F M nội tiếp nên theo định lý Miquel,M nằm

(154)

Epsilon

staff

14 Lời giải bình luận đề thi chọn đội tuyển IMO 2017

Bình luận

Hai ý a), b) toán không liên quan tới Ý a) tốn giúp ta tìm trung điểmN củaEF Ý a) hệ trực tiếp bổ đề Nội dung bổ đề có [1] thực chất bổ đề có liên quan chặt chẽ tới toán IMO ShortList 2002, G7 xem [2], [3] Phần khó ý b) cần chứng minh∠Z A C D ∠DA B, thực chất ý chứng minh có [4] từ lâu chứng minh chặt chẽ trong bổ đề Để chứng minh ý dùng phép nghịch đảo [4] Bạn đọc tham khảo mở rộng toán ý b) sau, tham khảo [4]

Bài toán mở rộng.Cho tam giác A B C nội tiếp đường tròn O / K điểm

nằm phân giác ∠BA C K / đường tròn tiếp xúc với O / L

E ; F điểm thuộc K / cho C E ; DF tiếp tuyến K / ;

E ; F vàAkhác phía vớiB C Đường trịn ngoại tiếp tam giác A C E vàA BF cắt P khácA Chứng minh rằng∠P A C D ∠L A B :

Tài liệu tham khảo:

[1] Two circles are tangent,

https://artofproblemsolving.com/community/c6h1299041 [2] IMO Shortlist 2002, G7,

https://artofproblemsolving.com/community/c6h17323p118682

[3] http://analgeomatica.blogspot.com/2016/01/ve-bai-toan-g7-trong-imo-shortlist-2002.html

[4] Passes through tangent point,

https://artofproblemsolving.com/community/q1h1180223

Bài (7.0 điểm) Cho tam giácABC nội tiếp đường tròn.O/:ĐiểmAdi động trên.O/

sao choAB > BC vàM trung điểm củaAC:Đường tròn đường kínhBM cắt.O/tại

R:Giả sửRM cắt.O/tại điểm thứ haiQ;cắtBC tạiP:Đường trịn đường kínhBP cắt

AB; BO điểm thứ haiK; S:

a) Chứng minh rằngSRđi qua trung điểm củaKP:

b) Gọi N trung điểm BC: Trục đẳng phương hai đường trịn đường kính

AN; BM cắtSRtạiE:Chứng minh rằngME ln qua điểm cố định

Lời giải a)Dễ thấyBQlà đường kính của.O/ GọiI giao điểm củaSRvàPK:Ta thấy

∠SP I D∠SBK D∠QCA

∠PSI D∠PBR D∠CQR:

(155)

Epsilon

staff

Lời giải bình luận đề thi chọn đội tuyển IMO 2017 15

O

A

B C

M

N

R P

Q

S K

I E

X T

D

L H

b) Định nghĩa lại điểm E chân đường cao hạ từ C tam giác ABC: Gọi H giao điểm đường caoAD; BL tam giác ABC: Dễ thấy tứ giácLMND nội tiếp nên

CM CL DCN CDvàHAHD DHB HL;suy raCH trục đẳng phương đường trịn đường kínhAN vàBM

Ta cóE nằm trênCH nên thuộc trục đẳng phương đường trịn đường kínhAN vàBM Do

∠BEH D∠BRP D90ınên tứ giácBHERnội tiếp, ta suy

∠HRE D∠EBH D∠OBC D∠PBS D∠PRS

nên ba điểmS; R; E thẳng hàng Từ suy raE giao điểm củaSRvà trục đẳng phương đường trịn đường kínhAN; BM:

GọiX giao điểm củaEM vàBQ:GọiT trung điểm củaBC dễ thấy tứ giácETML

nội tiếp Từ suy

∠MEC D90ı ∠MET D90ı ∠ALT D90ı ∠BAC D∠BAL D∠BCQ:

Kết chứng tỏ tứ giácBCXE nội tiếp, suy ra∠BXC D BEC D 90ı:VậyX hình chiếu củaC lênBQcố định Tóm lại, ta cóEM quaX cố định

Bình luận

(156)

Epsilon

staff

16 Lời giải bình luận đề thi chọn đội tuyển IMO 2017

Bạn đọc tham khảo phát triển tốn ý b) sau:Cho tam giácABC cố định đường tròn.K/thay đổi quaB; C cắtCA; AB lần lượt tạiE; F Gọi J; Llần lượt tâm ngoại tiếp tam giácABE ACF: EJ cắtFLtạiP: Qlà tâm ngoại tiếp tam giácKEF:Chứng minh đường thẳngPQluôn qua điểm cố định khi.K/thay đổi.

Bài (7.0 điểm) Cho 7số dương a1; a2; : : : ; a2 7:Với mỗin > 7; ta đặt an D max

˚

ai1ai2ai3ji1 C i2 C i3 D n ; i1 i2 i3 n :

Chứng minh tồn số nguyên dươngm không vượt số nguyên dương

N > m choanan m D a2n m với mọin > N :

Lời giải Với mỗin nguyên dương, ta đặtbn D lnan:Khi tốn phát biểu

lại sau:Cho 7số thựcb1; b2; : : : ; b2 7: Với mỗin > 7; ta đặt

bn D max˚bi1 C bi2 C bi3ji1 Ci2 Ci3 D n ; i1 i2 i3 n :

Chứng minh tồn số nguyên dương m không vượt quá và số nguyên dương N > m sao chobn C bn m D bn m với mọin > N :

Gọi` ` /là số cho b`

` D max

nb

i

i

ˇ ˇ

ˇ1 i o

:Ta có nhận xét sau:

Nhận xét Với mọi nnguyên dương thì bn

n b`

` :

Chứng minh Ta chứng minh quy nạp theon :Rõ ràng khẳng định với n

do cách định nghĩa ` : Ta cần xét trường hợpn > 7: Giả sử khẳng định với

k < n :Từ cách xác địnhbn;ta thấy tồn tạij1; j2; j3 Nthỏa j1Cj2Cj3 D n cho

bn D bj1 C bj2 C bj3:

Sử dụng giả thiết quy nạp, ta có

bn j1

b`

` C j2 b`

` C j3 b`

` D n b`

` ;

từ suy bn

n b`

` :Do khẳng định với n :Nhận xét chứng minh

Bây giờ, với nnguyên dương, đặtcn D n b` ` bn từ nhận xét ta cócn với

mọin : Đồng thời, với mọin

cnC2 ` D n C ` / b` ` bnC2 `

nC ` / b` ` bn C b` Cb`/

D n b` ` bn D cn:

Từ suy

(157)

Epsilon

staff

Lời giải bình luận đề thi chọn đội tuyển IMO 2017 17

Gọix số nguyên dương nhỏ cho x ` > 7và đặt

M D maxfci j1 i x ` 1g:

Khi đó, với mọin > x ` ; đặtn D k x `C r (với0 r < x `), ta có

cn crC2 k / x ` crC2 k / x ` crC2 x ` M :

Ta có nhận xét sau:

Nhận xét Với mọi nnguyên dương, tồn số tự nhiên s1; s2; : : : ; s2 sao cho cn D s1c1 Cs2c2 C C s2 7c2 7:

Chứng minh Ta chứng minh quy nạp theo n : Rõ ràng khẳng định với

n (chỉ cần chn sn D 1v si D vii Ô n) Ta cần xét trường hợp n >

là đủ Giả sử khẳng định với k < n :Tư công thức xác định củabn; ta suy

cn D minfci1Cci2Cci3 j1 i1 i2 i3 n ; i1Ci2Ci3 D ng; 8n > 7:

Do tồn j1; j2; j3 N thỏaj1 C j2 C j3 D n cho

cn D cj1 C cj2 C cj3:

Theo giả thiết quy nạp, tồn số tự nhiênu1; u2; : : : ; u2 7; v1; v2; : : : ; v2 7; w1;

w2; : : : ; w2 cho

cj1 D u1c1 C u2c2 C C u2 7c2 7;

cj2 D v1c1 C v2c2 C C v2 7c2 7;

cj2 D w1c1 C w2c2 C Cw2 7c2 7:

Do đó, ta có

cn D s1c1 Cs2c2 C C s2 7c2 7;

trong đósi D ui C vi C wi với1 i 7:Như khẳng định với n :Nhận

xét chứng minh

Nhận xét Dãy cn/chỉ nhận hữu hạn giá trị.

Chứng minh Nhận xét suy trực tiếp từ tính bị chặn củacn kết nhận

xét

Bây giờ, docnchỉ nhận hữu hạn giá trị

cnC2 k x ` cnC2 k / x ` cn

với mọin nên tồn tạiN1 đủ lớn đểcn D cn x ` với mọin > N1:Khi đó, ta có

n b` ` bn D n x ` / b` ` bn x `; 8n > N1;

hay

(158)

Epsilon

staff

18 Lời giải bình luận đề thi chọn đội tuyển IMO 2017

Từ đây, ta suy

bn x ` D b` C bn x `; 8n > N1 C x ` :

Cộng chéo vế hai kết trên, ta

bnC bn x ` D bn x `; 8n > N1 C x ` :

ChọnN D N1 C2 x `và m D x ` ; ta có điều phải chứng minh

Bình luận

Bước logarith hố để đưa dãybnđịnh nghĩa bn D maxfbi Cbj Cbk j i Cj C

k D ng hiển nhiên Nó khơng tạo bước ngoặt đáng kể tạo tiện lợi trình bày tính tốn Trong phần sau ta coi làm việc với toán chuyển đổi sang phép cộng

Đây toán dãy số định nghĩa truy hồi thông qua max;tạo tình tổ hợp thú vị Để hình dung “hành vi” dãy số này, ta cần xây dựng thử dãy số với tham số nhỏ Chẳng hạn, thay2 bằng5 với5số hạng ; ; ; ;

thì số hạng 7; ; ; ; ; ; : : :và nhìn thầy vai trị số hạng thứ5 (số 0) dãy số này, từ dẫn đến ý tưởng xét` cho b`

` lớn

nhất bước

Bài tốn coi anh em song sinh với số đề thi Toán Quốc tế năm 2010, cụ thể sau:

Choa1; a2; a3; : : :là dãy số thực dương và s là số nguyên dương cho an D maxfak Can k j k n 1g

với mọi n > s : Chứng minh tồn số nguyên dương l s N sao cho an D al C an l với mọin N :

Sự khác biệt nằm phép cộng phép nhân, vốn chuyển sang dễ dàng nhờ logarith, độ sâu phép truy hồi Tuy nhiên bước chuyển từ sang gần không ảnh hưởng đến bước lý luận lời giải Ý tưởng xét

t D max˚ai

i ; i D ; : : : ; s xét dãy sốbn D n t an:

Bài (7.0điểm) Với số nguyên dươngn, xét a1; a2; : : : ; a2 n hoán vị của2 nsố

nguyên dương Một hoán vị gọi “đẹp” với mọi1 i < j n

thìai CanCi D nC1vàai aiC1không đồng dư vớiaj ajC1theo modulo2 nC1

(Quy ước a2 nC1 D a1:)

a) Với n D 6, hoán vị đẹp

(159)

Epsilon

staff

Lời giải bình luận đề thi chọn đội tuyển IMO 2017 19

Lời giải a)Ta xét hoán vị sau:

1 ; ; ; ; ; ; ; 1 ; ; ; ; 7:

Dễ thấy rằng1 C1 D C1 D C9 D C5 D 3C1 D C7 Đồng thời, hiệu số liên tiếp lấy theo modulo ; ; ; ; ; ; ; 1 ; ; ; ; phân biệt Do đó, hốn vị thỏa mãn điều kiện đề

b)Gọi A tập hợp hoán vị đẹp B tập hợp cách điền số ; ; : : : ; n

lên đỉnh đa giác X1X2X3: : : X2 n, cho hai số thuộc đầu mút

đường kính có tổng nC 1I đồng thời khơng có dãy liên tiếp số có tổng bội nC :

Ta chứng minh tồn song ánh từ A ! B đó, để chứng minh tồn hốn vị, ta cần rằngjAj D jBj > :

(1)Tồn song ánhA ! B :

Xét hoán vị thuộcAlà a1; a2; : : : ; a2 n/thì vớii D ; n, ta chọnbi f1 ; : : : ; ng

sao cho bi aiC1 mod nC / Theo giả thiết tất sốbi phải đôi

phân biệt Ta quy ước rằnga2 nC1 D a1 b2 nC1 D b1

Ta điền số lên vòng tròn theo thứ tự,bi điền cho đỉnh Xi Dễ thấy

bi C biCn aiC1 C aiC1Cn aiCn D n C / mod nC / :

Do0 < bi C biCn < n C / nênbi C biCn D n C1 :Ngồi ra, ta có

ai D ai 1/C ai 2/C C a2 a1/C a1 D a1 C i

X

kD1

bk

và số đôi phân biệt theo modulo2 nC 1nên không tồn i ; j cho

aj D j

X

kDi

bk mod nC / :

Do đó, cách điền số bi thỏa mãn điều kiện cách điền trongB :Ngược lại, xét

cách điền B bi điền cho đỉnhXi; i D ; n xét dãy số

a1 D b1; a2 D b1 C b2; a3 D b1 Cb2 C b3; : : : ; D i

X

kD1

bk

và tổng lấy theo modulo2 n C1 :Khi đó, ta thấy số dương đồng thời

phân biệt, khơng, giả sử cóai D aj

0 D aj D j

X

kDi

bk mod nC / ;

mâu thuẫn Hơn nữabi C biCn ai C aiCn aiCn mod n C1 / nên

(160)

Epsilon

staff

20 Lời giải bình luận đề thi chọn đội tuyển IMO 2017

Suy ra, tổngai CaiCnvới i D ; nđều có số dư làa f0 ; ; ; : : : ; ngkhi

chia cho2 nC :Chú ý có ntổng chúng đôi rời nên

2 n

X

kD1

k D

2 n

X

iD1

C aiCn/ n a mod n C / ;

mà n ; n C / D nên cần có a D 0hay C aiCn D nC với i D ; n

Do đó, dãy số hốn vị đẹp nên thuộcA : Đến ta có song ánh từA vàoB

/ chứng minh

(2)Chứng minhjBj > :

Trước hết, ta thấy số cách điền chobi CbiCn D nC1với mọii D ; nlàn Š2n

vì số ! nthuộc cặp rời có tổng nC 1và đổi chỗ số điền choXi

vàXiCn:Giả sử cách điền số lên đa giác, có dãy liên tiếp có k số mà tổng chia

hết cho nC k gọiSk số tập có k phần tử đây, ta có

n 2n k n k / Š S

k cách điền Cụ thể cón cách chọn vị trí bắt đầu dãy (vì tính đối

xứng qua tâm nên dãyk số thỏa k số đối xứng thỏa), cịn lại2 n k

số xếp tùy ý thỏa mãn tính đối xứng 2n k n k / Š

Từ suy số cách điền thỏa điều kiện tổngbi C biCn D n C 1nhưng có dãy số

liên tiếp có tổng chia hết cho2 nC 1sẽ không vượt

S D n

n

X

kD3

2n k n k / Š Sk:

Để đánh giá đại lượng này, ta tiếp tục chứng minh nhận xét sau:

a) Sk

1 kC

k n

b) C2 nk 4k 1Cnk 11 với n ; k ; n k :Khi vớin D ; ; ; ta có

thể hoán vị cụ thể thỏa mãn đề

c) x

0

0 Š C x1

1 Š C x2

2 Š C C xn

n Š < e

x

với mọin

Thật vậy:

a) Ta thấy có C2 nk tập tùy ý cók 1phần tử n Với tập

thế có khơng q cách để thêm vào số để đượck số có tổng bội của2 nC1 Ngược lại, tập conk phần tử có tổng chia hết cho2 nC tương ứng với đúngk

tập cók 1phần tử của2 n :Dùng đếm hai cách, ta có k Sk C2 nk hay

(a) chứng minh

b) Có thể chứng minh dễ dàng quy nạp theon với ý

C2 nk C k n D

2 n k C

k C k n n k k C k n

2 n k C

n k

k

< 4k:

c) Có thể chứng minhf x / D ex Pn

iD0 xi

(161)

Epsilon

staff

Lời giải bình luận đề thi chọn đội tuyển IMO 2017 21

Cuối cùng, ta có

S n

n

X

kD3

2n k n k / Š1 kC

k

n n n

X

kD3

2n k n k / Š1 kC

k n 14

k

D 2nn Š1

n

X

kD3

2k k Š <

n

n Še

2 1

C2 C / <

n

n Š :

Do đó,jBj 2nn Š S > Bài tốn giải hồn tồn

Bình luận

Ở này, chúng tơi có tham khảo ý tưởng Nguyễn Huy Tùng, IMO 2014 (ý tưởng cho lời giải chi tiết trên) Lê Hồng Quý, IMO 2006 (ý tưởng lời giải phần bình luận bên dưới)

Lời giải tiếp cận theo hướng xác suất, cụ thể để chứng minh tồn mơ hình thỏa mãn ràng buộc đề bài, thay xây dựng trực tiếp, đếm ước lượng tương đối số mơ hình thỏa mãn chứng minh số dương

Một số toán tương tự:

1 (Bulgaria, 2016)Hỏi có tồn hay khơng số ngun dương n < 09sao cho ncó thể biểu diễn dạng tổng ba số phương dương phân biệt hơn1 0 0cách? Tương tự ý tưởng nêu 6, ta đếm số lượng tổnga2 Cb2 C c2 phân biệt với1 a < b < c N N số nguyên dương Có tất cảCN3

a ; b ; c /nhưng nhận không quá3 N2 giá trị nên phải tồn giá trị tương ứng với C

3 N

3 N2 ChọnN D 0 3thì câu trả lời cho toán khẳng định

2 Chứng minh 21 0 người, không thiết phải có 0 người đơi quen đôi không quen

3 Trong bảng1 01 0 ; người ta viết vào ô số nguyên dương không vượt quá5 0 số xuất hai lần Chứng minh chọn ra1 0 số phân biệt thuộc hàng cột khác từ bảng

Để giải 6, ta tiến hành trực tiếp với ý tưởng xây dựng cho với mọii thìjaiC1 aij i C mod nC /, có điều u cầu

bài toán thỏa mãn Cụ thể là:

Xếp số lên đỉnh đa giác X1X2: : : X2 n choXi D i vớii D ; nvà

Xi D i vớii D n C ; n Ta tiến hành chọn số hoán vị theo quy tắc sau:

Xuất phát từ đỉnhX1, ta chọna1 D

Đi đến đỉnh liền sau củaXnC1là XnC2, chọna2 D

Đi đến đỉnh liền sau củaX2 làX3 chọna3 D :

(162)

Epsilon

staff

22 Lời giải bình luận đề thi chọn đội tuyển IMO 2017

Tuy nhiên, vấn đề làanC1 D nên hiệujanC1 anj fn ; nC 1g,

do ta cần tránh bước thứi mà jaiC1 aij fn ; n C1g (nếu hai

số chẵn giá trị tương ứng ; n C 2)

Để làm điều này, bước thứi, thay ta vào điểm bên cạnh điểm đối xứng qua đường kính ta đến điểm bên cạnh số để chojaiC1 aij D 1,

đoạn lại thực cũ Do việc chọn giá trị theo chiều nên không xảy trường hợp lặp lại số cũ

Cuối cùng, ta cần xác định bước thứi Để ý thực ban đầu, ta có sốa1; a2; : : : ; anchỉ chứa số nguyên dương lẻ số nguyên âm chẵn

Khi đó, nếunchẵn thìan D nvà jan anC1j D n 1, cần tránh giá trị nC2

ra n / nC / mod n C1 / Thay đến điểm nC22, ta sang điểm bên cạnh

Nếun lẻ tương tự, thay đến nC21, ta sang điểm bên cạnh Theo quy tắc giá trịjai aiC1j nhận tất giá trị lẻ từ1 ! n 1và

(163)

Epsilon

staff

LỜI GIẢI VÀ BÌNH LUẬN ĐỀ THI CHỌN ĐỘI TUYỂN IMO 2018

Trần Nam Dũng – Võ Quốc Bá Cẩn – Trần Quang Hùng Lê Phúc Lữ – Nguyễn Lê Phước

1 Lời nói đầu

Kỳ thi chọn đội tuyển Việt Nam dự thi toán quốc tế năm 2018 diễn hai ngày 30 31/3 vừa qua Trong ngày, thí sinh phải làm ba tốn vịng 270 phút

Tiếp nối truyền thống nhiều năm nay, Ban biên tập chúng tơi muốn theo sát thí sinh giáo viên chuyên Toán, giải phân tích, khai thác có đề thi Thơng qua đó, người thấy chất, vẻ đẹp vấn đề, quan trọng có nguồn tư liệu, định hướng rèn luyện cho mùa thi sau Ban biên tập hiểu đề thi TST ln mang tính thử thách, tạo cảm hứng bên cạnh VMO, IMO, nội dung ln chờ đón

Năm 2018, với đời đồng hành chương trình BM2E – Bring Math to Everyone, chúng tơi có nhiều động lực hỗ trợ Với mong muốn đẩy mạnh phong trào chuyên Toán nhiều nơi, hy vọng B2ME ln nhận đón nhận người chung tay phát triển Ngoài với tinh thần hội nhập quốc tế, năm xuất Epsilon 14, phiên tiếng Anh

Cuối cùng, để hoàn thành tài liệu này, xin gửi lời cám ơn đến thầy Nguyễn Chu Gia Vượng (viện Toán học) bạn Hoàng Đỗ Kiên (HCB IMO 2013), Nguyễn Nguyễn (PTNK TPHCM) tài liệu góp ý đáng giá Mọi thắc mắc, góp ý, xin gửi tin nhắn cho trang chủ tạp chí Epsilon

(164)

Epsilon

staff

2 Lời giải bình luận đề thi chọn đội tuyển IMO 2018

2 Đề thi

2.1 Ngày thi thứ (30/03/2018)

Bài (7.0 điểm) Cho tam giácABC nhọn không cân cóD; E; F trung điểm

các cạnhBC; CAvàAB:Gọi.O/; O0/lần lượt tâm ngoại tiếp tâm Euler tam giác

Xét điểmP bên tam giácDEF vàDP; EP; FP cắt lại.O0/lần lượt tạiD0; E0; F0:Gọi

A0là điểm đối xứng vớiAquaD0:Xác định tương tự vớiB0vàC0:

a) Chứng minh nếuPO DPO0thì đường tròn.A0B0C0/đi quaO:

b) LấyX đối xứng vớiA0 qua đường thẳngOD:Xác định tương tự vớiY vàZ:Gọi H

trực tâm tam giácABC vàXH; Y H; ZH cắt BC; CA; AB theo thứ tự tạiM; N; K:

Chứng minh rằngM; N; K thẳng hàng

Bài (7.0 điểm) Vớimlà số nguyên dương, xét bảng ô vuôngm2018gồmmhàng,2018

cột mà có vài trống, cịn vài ô đánh số0hoặc1:Bảng gọi “đầy

đủ” với chuỗi nhị phânS có2018ký tự nào, ta chọn hàng

của bảng điền thêm0; 1vào để2018ký tự hàng tạo thành chuỗiS (nếu chuỗiS có

sẵn hàng coi thỏa mãn) Bảng gọi “tối giản” đầy đủ ta bỏ hàng khơng cịn đầy đủ

a) Với k ; chứng minh tồn bảng tối giản 2k cho có

đúngk cột có đủ 0lẫn1 :

b) Cho bảng tối giảnm có đúngk cột chứa cả0lẫn1 Chứng minh rằngm 2k:

(Một dãy nhị phân độ dài dãy có dạng x1x2: : : x2 xi f0 ; 1g với

mọii f1 ; ; : : : ; 8g:)

Bài (7.0 điểm) Cho số nguyênn 3và Anlà tập hợp tất số nguyên dương nhỏ

n ; nguyên tố vớin :Xét đa thức

Pn x / D

X

k2An

xk 1:

a) Chứng minh rằngP x /chia hết cho đa thức xr C 1với r số nguyên dương

b) Tìm tất số nguyên dương nđểPn x /bất khả quy trênZŒ x  :

2.2 Ngày thi thứ hai (31/03/2018)

Bài (7.0 điểm) Cho a số thực thuộc đoạn 12; 23

: Xét dãy số un/ vn/

n D ; ; : : : /được xác định sau:

un D

3

2nC1 /

b2nC1ac

; D

3

2nC1 /

nCb2nC1ac

:

a) Chứng minh

2

X

iD0

ui

!2 C

2

X

iD0

vi

!2

a2 a C C

42 9:

(165)

Epsilon

staff

Lời giải bình luận đề thi chọn đội tuyển IMO 2018 3

Bài (7.0 điểm) Một bảng vng m n A B C D có đỉnh giao lộ (có tất

mC1 / nC1 / giao lộ) Người ta muốn thiết lập tuyến đường A ;đi theo

các cạnh song song với cạnh hình chữ nhật qua tất giao lộ lần,

sau quay A :

a) Chứng minh xây dựng đường mlẻ nlẻ

b) Vớim ; n thỏa mãn điều kiện câua), hỏi có giao lộ mà có ngã rẽ?

Bài (7.0 điểm) Cho tam giácA B C nhọn nội tiếp O / J /là đường trịn bàng tiếp góc

Acủa tam giác Gọi D ; E ; F tiếp điểm J /vớiB C ; C A A B :

a) Gọi L trung điểm B C : Đường tròn đường kính LJ cắt đường thẳng

DE ; DF tạiK vàH :Chứng minh đường tròn BDK / C DH /

cắt đường tròn J / :

b) Giả sử đường thẳng EF cắt đường thẳng B C G GJ cắt đường thẳng

A B ; A C M ; N : Gọi P vàQ điểm đường thẳng J B ; J C

sao cho ∠P A B D ∠QA C D 0ı: Gọi T là giao điểm hai đường thẳng

P M ; QN S điểm cung lớn B C đường tròn O / : Gọi I

tâm đường tròn nội tiếp tam giác A B C :Chứng minh đường thẳng S I cắt đường

(166)

Epsilon

staff

4 Lời giải bình luận đề thi chọn đội tuyển IMO 2018

3 Bình luận chung

Các tốn năm phân theo phân môn sau:

Bài 6:Hình học

Bài 5:Tổ hợp

Bài 3:Số học

Bài 4:Đại số

Hai hình có cấu hình đẹp, đặc biệt 1, nơi ta gặp lại ý tưởng

kinh điển hình học qua đường trịn Euler đường trịn Hagge: chứng minh đồng viên

thơng qua phép vị tự Tuy nhiên hai 1và 6quá giống dạng bài, công

cụ: định lý Menelaus, đẳng giác, phương tích, phép biến hình Nếu sử dụng hai hình, nên hai khác hướng

Hai tổ hợp khác nhẹ nhàng dù liên quan đến lĩnh vực toán đại: lý thuyết

mã hóa thơng tin lý thuyết đồ thị.Bài 5có vẻ quen thuộc chưa gặp

khơng khó xử lý

Bài 3là phát biểu dạng đa thức (thậm chí có từ bất khả quy) chất

một số học Ý tưởng xây dựng song ánh (dạng k ! n k ; k ! k C n2

tương tự) đơn thức, trường hợp nphi phương (square-free) dùng quy nạp

theo số ước ngun tố Bài có nhiều liên quan đến tổng Ramanujan số học

Bài 4là bất đẳng thức dãy số có hình thức lạ xấu Tuy nhiên, nhìn kỹ có

thể thấy tư tưởng dùng hệ đếm nhị phân Nếu “bắt” ý phần sau phần xử lý kỹ thuật Có lẽ mà ban đề thi đưa vào vị trí dễ ngày thứ hai, dù

theo chúng tơi, khó hơnbài 5nhiều “sát thủ”

Như vậy, thấy rằng, hai 3, 4là hai Số học Đại số song kiếm hợp bích

Nhìn tổng thể, đề thi năm hay tốt, có nhiều đất diễn cho thí sinh, khơng có siêu sát thủ Năm ban chấm thi vất vả Dự kiến có thí sinh làm trọn vẹn

Có điểm chưa tất TST năm có hai ýa)vàb) Trong số

trường hợp có hai ý hay nhiều trường hợp làm tốn trở nên lắt nhắt, hay

như ýb)củabài 3trở nên tầm thường có ýa) Việc có hai hình “đồng dạng”

điều cần khắc phục nên khai thác thêm mảng Hình tổ hợp bất đẳng thức hình học

để đa dạng hóa Hơn hai hình có hai ýa),b)gần độc lập với thành gần

(167)

Epsilon

staff

Lời giải bình luận đề thi chọn đội tuyển IMO 2018 5

4 Lời giải bình luận toán

Bài (7.0điểm) Cho tam giácA B C nhọn khơng cân có D ; E ; F trung điểm cạnhB C ; C A vàA B : Gọi O / ; O0/lần lượt tâm ngoại tiếp tâm Euler tam giác Xét điểmP bên tam giácDEF DP ; EP ; F P cắt lại O0/lần lượt tại D0; E0; F0:GọiA0là điểm đối xứng với AquaD0:Xác định tương tự với B0và C0:

a) Chứng minh nếuP O D P O0 đường tròn A0B0C0/ quaO :

b) LấyX đối xứng vớiA0qua đường thẳng OD:Xác định tương tự vớiY Z :Gọi H trực tâm tam giác A B C XH ; Y H ; ZH cắt B C ; C A ; A B theo thứ tự

M ; N ; K :Chứng minh M ; N ; K thẳng hàng

Lời giải a) Gọi Ilà điểm đối xứng với O qua P : Ta có O0 là trung điểm của OH nên

O0P k I H : Lại có P O D P O0nênI O D I H :

A

B C

H O0 O

D P D0

A0

G

I S

GọiS vàG trung điểm đoạn thẳngAI; AH:Ta có

SP D

2AO D

2R DO

0D

vàSP kAO kO0D nên tứ giácO0SPDlà hình bình hành Từ suy raDP kO0S:

Lại cóSP D 12R DO0D0nên tứ giácSD0PO0là hình thang cân Suy raO0P DSD0và

IH D2O0P D2SD0DIA0:

Từ đó, ta cóIA0 DIH DIO nênA0nằm đường trịn.I; IO/:Chứng minh tương tự, ta

(168)

Epsilon

staff

6 Lời giải bình luận đề thi chọn đội tuyển IMO 2018

b)GọiRlà bán kính đường tròn.O/:Dễ thấyGD DR:Xét phép vị tự tâmAtỉ số 12 biến

B; C; A0; X; H; M trung trựcBC thành thànhF; E; D0; U; G; M0và trung trực

EF:Khi đó, ta có M CMB D MM00FE vàU đối xứng vớiD0qua trung trựcEF:Suy

MB M C D

M0F M0E D

GF GE

UF UE D

p

R2 DF2

p

R2 DE2

D0E D0F:

A

B C

H O0 O

D G

E

F P

D0

A0

X

M

M0 U

Tương tự, ta tính NCNA KAKB:DoDD0; EE0; FF0đồng quy nên

D0F D0E

F0E F0D

E0D E0F D1:

Từ M CMB NCNA KBKA D1;hayM; N; K thẳng hàng

Bình luận Đây tốn hình học hay có nhiều ý Nếu câub)chỉ ứng

dụng định lý Menelaus câua)là tốn thú vị Điểm đáng ý

câua)là ta viết lại thành toán thú vị sau:

Bài toán 1.Cho tam giácABC vớiD; E; F lần lượt trung điểm cạnhBC; CA

AB: P là điểm mặt phẳng Các đường thẳngPD; PE; PF cắt lại đường tròn

.DEF /lần lượt tạiX; Y Z:GọiU; V W lần lượt đối xứng củaA; B; C quaX; Y; Z:

a) Chứng minh đường tròn.U V W /đi qua trực tâm tam giácABC:

b) GọiK là tâm đường tròn.U V W /:Chứng minh đường thẳngPK luôn qua một điểm cố định khiP thay đổi.

(169)

Epsilon

staff

Lời giải bình luận đề thi chọn đội tuyển IMO 2018 7

Sau tổng quát cho toán đồng viên trên:

Bài toán 2.Cho tam giácABC nội tiếp đường tròn.O/và hai điểmP; Qbất kỳ trên mặt phẳng Phép vị tự tâmP tỉ sốkbiến điểmA; B; C thành điểmA0; B0C0:Gọi

A0A1; B0B1 C0C1lần lượt đường kính đường trịn.A0B0C0/:Các đường thẳng

QA1; QB1QC1 cắt lại đường tròn.A1B1C1/lần lượt tạiA2; B2; C2:

a) Phép vị tự tâmA; B; C tỉ số 1 k1 lần lượt biến điểmA2; B2; C2thànhA3; B3 C3: Chứng minh đường tròn.A3B3C3/đi quaP:

b) GọiK là tâm đường tròn.A3B3C3/:Chứng minh phép vị tự tâmK tỉ số k1 biến

P thành tâm ngoại tiếp tam giácABC:

Mặt khác từ cấu hình tốn TST tìm khai thác nhiều điểm thú vị khác, sau khai thác thú vị từ cấu hình này:

Bài tốn 3.Cho tam giác A B C có trực tâm H : Gọi A1; B1; C1 lần lượt trung điểm của cạnh B C ; C A A B : P là điểm đường thẳng Euler tam giác

A B C : N / là đường tròn Euler tam giác A B C :GọiA1A0; B1B0 C1C0 lần lượt

là đường kính đường trịn N / :

a) Chứng minh đường thẳngA A0; BB0 C C0đồng quy tại X :

b) Các đường thẳng A A0; BB0 C C0 lần lượt cắt lại đường tròn N / tạiA2; B2; C2:

Qlà điểm đường thẳng P X :Các đường thẳngQA2; QB2 Q C2lần lượt cắt lại đường tròn N / tạiA3; B3; C3:GọiA4; B4 C4 lần lượt đối xứng của A ;

B ; C lần lượt quaA3; B3; C3:Chứng minh đường tròn K /ngoại tiếp tam giác

A4B4C4 đi quaH :

c) Chứng minh rằng K luôn nằm đường thẳng cố định song song với đường thẳng P X khiQ thay đổi.

Ngồi ta sử dụng kết toán sau làm bổ đề để chứng minh ýb)của TST

bằng phương pháp phương tích trụ đẳng phương:

Bài tốn 4.Cho tam giácA B C và điểmP bất kỳ mặt phẳng Các điểm X ; Y Z nằm trên đường tròn P B C / ; P C A / ; P A B / sao cho đường tròn X Y Z /đi quaP :Gọi

U ; V W lần lượt đối xứng củaX ; Y ; Z qua trung trực củaB C ; C A ; A B : Chứng minh

rằng đường tròn U V W / cũng qua P :

Có thể tham khảo thêm mở rộng liên kết:http://analgeomatica.blogspot

(170)

Epsilon

staff

8 Lời giải bình luận đề thi chọn đội tuyển IMO 2018

Bài (7.0 điểm) Vớim số nguyên dương, xét bảng ô vuôngm gồm mhàng,

2 cột mà có vài trống, cịn vài đánh số0 hoặc1 :Bảng

gọi “đầy đủ” với chuỗi nhị phân S có 8ký tự nào, ta chọn hàng bảng điền thêm0 ; 1vào để 8ký tự hàng tạo thành chuỗi S (nếu chuỗiS có sẵn hàng coi thỏa mãn) Bảng gọi “tối giản” đầy đủ ta bỏ hàng khơng cịn đầy đủ

a) Với0 k ; chứng minh tồn bảng tối giản2k cho có

đúngk cột có đủ 0lẫn1 :

b) Cho bảng tối giảnm2 8có đúngkcột chứa cả0lẫn1 Chứng minh rằngm 2k: (Một dãy nhị phân độ dài dãy có dạng x1x2: : : x2 xi f0 ; 1g

với mọii f1 ; ; : : : ; 8g:)

Lời giải a)Đầu tiên, xét bảng ô vuông trống kích thước2k và2k xâu nhị phân

có độ dàik ;ta tiến hành điền số ; 1của xâu vào phần bên trái

hàng, xâu dùng cho hàng Khi đó, phần cịn lại phía bên phải là2 k cột

để trống Dễ thấy cột trongk cột bên trái bảng có chứa đúng2k số0

2k số1 (tức có đủ cả0lẫn 1như ràng buộc) Ta chứng minh bảng tối giản

0 : : : : : : : : : : : :

0 : : : : : : : : : : : :

: : : : : : : : : : : : : : : : : :

1 : : : : : : : : : : : :

1 : : : : : : : : : : : :

Với xâu nhị phân s D a1a2 : : : a2 tùy ý, ta xét xâu s0 D a1a2: : : ak

nó Rõ ràng s0 xuất phần đầu hàng bảng nên ta điền tiếp

akC1; akC2; : : : ; a2 vào phần trống thu đượcs : Ngồi ra, ta thấy có

đúng hàng chứa số củas0nên bỏ hàng khơng có hàng dùng để

“khơi phục” s Do đó, bảng cho thỏa mãn

b)Giả sử k cột bên trái bảng có chứa đủ 0và1 :Dưới đây, ta chứng minh

nhận xét quan trọng mấu chốt để giải tốn

Nhận xét.Tất trong2 k cột bên phải trống.

Chứng minh.Xét xâu nhị phâns có độ dài k giả sửAs tập hợp hàng màk ô

đầu tiên bên trái sinh s :Ta chứng minh As có hàng mà

tồn từ vị trík C1 đến vị trí2 trống

Xét vị trí thứkC1ở hàng trongAs:Dễ thấy tất thuộc cột thứkC1

của bảng, cột không chứa đồng thời0 :

Nếu toàn cột có chứa số giả sử số0 (tương tự là1) Khi đó, chuỗi độ dài

k C 1có dạng 0 : : : 1gồm k số0bên trái biểu diễn hàng nào,

mâu thuẫn với tính đầy đủ bảng Do đó, phải có tập As có hàng mà vị trí

(171)

Epsilon

staff

Lời giải bình luận đề thi chọn đội tuyển IMO 2018 9

Tiếp tục, lại xét vị trí thứ k C 2thì tương tự trên, tồn cộtk C 2có chứa số khơng

thỏa nên phải có cột trống, ta lại tiếp tục chọn tập A00s A0s mà vị trí thứ

k C ; k C 2đều trống Cứ làm gặp cột cuối ta hàng có

tất vị trí từk C 1đến 8đều trống

Như thế, với xâu nhị phâns có độ dàik, ta ln tìm hàng mà tất ô từ vị trí

k C1đến2 trống Chú ý hàng không thiết phân biệt có

hàng sử dụng cho nhiều xâu Gọi tập hợp hàng làA :

Ta thấy Acó thể biểu diễn cho tất xâu độ dài :Rõ ràng hàng bảng

đều phải thuộc vàoAvì khơng, ta loại bỏ hàng mà bảng cịn đầy đủ, mâu

thuẫn với tính chất tối giản Như vậy,A tập hợp tất hàng bảng Do đó,

tồn cột từk C 1đến 8của bảng trống Nhận xét chứng minh

Tiếp theo, tồn bảng bên phải trống nên biểu diễn xâu nhị phân độ dài

2 k Vì bảng ban đầu tối giản nên buộc bảng bên trái tối giản

Xóa đi2 k cột trống để cịn bảng kích thước m k :Đánh số hàng từ1đến m

và đặt Ai i D ; ; : : : ; m /là tập hợp xâu nhị phân độ dàik sinh từ hàng thứ

i (chú ý hàng cịn trống xen kẽ, khơng thiết phải điền

tồn số 0và 1)

Vì bảng ban đầu tối giản xâu nhị phân độ dài 8nên bảng sinh tối

giản với xâu nhị phân độ dài k :Suy đặtB D A1 [ A2 [ [Amthì jBj D 2k

(nghĩa bảng sinh tất 2k xâu nhị phân độ dài k)

Rõ ràng với i i D ; ; : : : ; m / ; phải có xâu nhị phân độ dài k

sinh hàng thứi ngược lại, tất xâu nhị phân sinh từ hàngi sinh

ra từ hàng khác ta bỏ hàng dẫn đến bảng khơng cịn tối giản Điều có nghĩa

Ai phải đóng góp phần tử vàoAvà phần tử không thuộc tập khác Do m

tập hợp đóng góp nhấtm phần tử vào B kéo theo jBj m :

So sánh đánh giá trên, ta có ngaym 2k Đây điều phải chứng minh

Bình luận Ở câua), với số hàng bảng là2k và với điều kiện đặt đúngk cột có chứa

cả0lẫn ;ta dễ dàng nghĩ đến việc xét tất xâu nhị phân có độ dàik (hiển nhiên có

đúng2k xâu thế)

Cái khó câub)chính tính tùy ý cột trong2 kcột cịn lại, ngồi trường

hợp đặc biệt vừa xét, cịn cột điền0 ;chỉ điền :Ta phải dùng tính

tối giản bảng để loại trường hợp Chú ý câub)trong đề cho sốk tùy ý nên có

thể xuất phát từ trường hợpk D 0để thấy tất ô phải trống Đây

định hướng lời giải

Với quan hệ hàng bảng chuỗi nhị phân, tốn hứa hẹn có cách tiếp cận sử dụng lý thuyết graph, đặc biệt bipartie graph Mong trao đổi thêm với bạn đọc Dạng tốn tập hợp mơ hình hóa bảng xuất lần đề VMO 2015 Dưới đây, ta xét số toán tương tự:

Bài toán (IMO Shortlist, 1998).Cho số nguyên n 2và tập hợp A D f1 ; ; : : : ; ng:

Một họF gồmt tập hợp của A ; đặt làA1; A2; : : : ; At được gọi “rời nhau” nếu như với cặp sốfx ; yg lấy từ Athì tồn tạiAi F

ˇ

ˇAi \ fx ; yg

ˇ

ˇ D :Họ F được

gọi “bao phủ” phần tử củaA đều thuộc vào tập trong F :

(172)

Epsilon

staff

10 Lời giải bình luận đề thi chọn đội tuyển IMO 2018

Bài toán (Bổ đề Kleitman).Một họF gồm tập tập hợp nsố nguyên dương đầu tiên gọi “down closed” mỗiX F thì tất tập củaX cũng thuộcF : Một họ

F gồm tập hợp gọi “up closed” mỗi X F thì tất tập hợp nhận X tập thuộcF : Chứng minh rằng

a) Nếu F1; F2 là hai họ “down closed” thì jF1 \F2j

jF1j jF2j

2n :

b) Nếu F1; F2 lần lượt họ “down closed” “up closed” thì jF1 \F2j j

F1j jF2j

2n :

Bài toán (Iran, 2001).Với số nguyên dương n ; xét bảng ô vuông n n được điền các số 1 ; một “đường chéo suy rộng” tập hợpn phần tử không hàng cùng cột Giả sử bảng có đường chéo suy rộng chứa toàn số : Chứng minh rằng xếp lại hàng cột bảng để bảng mà số vị trí

i ; j / với j < i n đều là0 :

Bài tốn 4.Xét bảng vngm n có chứa số0 hoặc1 :Giả sử tồn bảng có nhất

˛ m nsố1với0 < ˛ < :Chứng minh có hai ràng buộc sau phải đúng:

i) Có hàng có chứa nhấtnp˛ số1 :

(173)

Epsilon

staff

Lời giải bình luận đề thi chọn đội tuyển IMO 2018 11

Bài (7.0 điểm) Cho số nguyênn 3và An tập hợp tất số nguyên dương nhỏ

hơnn ; nguyên tố vớin : Xét đa thức Pn x / D

X

k2An

xk 1:

a) Chứng minh rằngP x /chia hết cho đa thức xr C1 vớir số nguyên dương

b) Tìm tất số nguyên dương nđểPn x /bất khả quy trênZŒ x  :

Lời giải a)Trước hết, ta thấy vớim ; k ZCvà klẻ thìxk m C1chia hết cho xm C1 :

Để thuận lợi lập luận, xét đa thứcQn x / D x Pn x / D Pk2Anx

k và dễ thấy cần

chứng minhQn x /chia hết choxrC1với sốr ZCnào Xét trường hợp sau:

Nếu n số lẻ:Vì gcd n ; k / D gcd n ; n k / với k D ; ; : : : ; n

nên với k An n k An: Suy ra, ta nhóm đơn thức trongPn x /

thành cặp rời có dạng xk; xn k/ : Vì k ; n k khác tính chẵn lẻ nên

n k / k D n k lẻ xk C xn k D xk xn k C1 / chia hết chox C :Từ

đó suy Qn x /chia hết chox C1 ; thỏa mãn

Nếu n chia hết cho 4:Tương tự trường hợp trên, ta ghép cặp.xk; xn k/với

chú ý hiệun 2k số chia4 dư2:Suy raxk Cxn k D xk.xn 2k C1/chia hết cho

x2C1:Do đó,Q

n.x/chia hết chox2C1;cũng thỏa mãn

Nếu n chia dư 2:Ta chứng minh nhận xét sau:

Nhận xét 1.NếuQn.x/chia hết choxrC1 r 2ZC/thì với số nguyên tố lẻppjn; đa thứcQpn.x/cũng thế.

Chứng minh.Giả sử a An sốkp C a Ap n vớik D ; ; : : : ; p :

Ứng với số k ;tổng đơn thức với mũ kp C avới a Antương ứng có

cùng nhân tử kp chia hết cho P

a2Ax a

, tức chia hết cho xr C :Do

đó, Qp n x / chia hết choxr C :

Nhận xét 2.Với n D p1p2 pm; trong đóm ZC p1; p2; : : : ; pm là các số nguyên tố lẻ phân biệt tùy ý thìQn x /sẽ phân tích được.

Chứng minh.Ta chứng minh quy nạp theo mlà số ước nguyên tố lẻ n :

Nếum D 1thì khin D pvớip nguyên tố, dễ thấyAn D f1 ; ; : : : ; pg nfp ; pg

nênQn x /sẽ chia hết choxp C :

Giả sử khẳng định với m : Xétn D p1p2 pm p số nguyên tố lẻ,

gcd p ; n / D ; Qn x / chia hết cho xr C : Ta chứng minh với

N D p n thìQN x /cũng chia hết choxr C1 :

Trước hết, giả sử trongAN có tính ln số ngun tố vớinnhưng lại

chia hết chop thay có QN x / ;ta có đa thức RN x / :Đa thức

sẽ lặp lại p lần của đa thức Qn x / (nghĩa RN x / chia thành

nhóm mà nhóm so với Qn x /chỉ sai khác lũy thừaxkp với1 k p 1) Do

(174)

Epsilon

staff

12 Lời giải bình luận đề thi chọn đội tuyển IMO 2018

Tiếp theo, ta cần bỏ lũy thừa chia hết chop, đơn thức có dạng xap với

a An:Khi đó, dễ thấy tổng chúng đa thứcQn xp/mà Qn xp/chia hết cho

xp r C1, tức chia hết cho xr C1 :Tóm lại Q

N x / D RN x / Qn xp/chia

hết choxr C :Do đó, khẳng định với sốncóm C 1ước nguyên tố lẻ Theo

nguyên lý quy nạp thìnhận xét 2được chứng minh

Kết hợp hai nhận xét lại, ta thấy trường hợp thỏa mãn điều kiện Bài toán giải hoàn toàn

b)Theo câua), ta đưa tìm tất số nguyên dươngn chojAnj D ; điều

tương đương với việc tìmn 3để' n / D : Ta xét trường hợp sau:

Nếun D p nguyên tố thì2 D ' n / D n nênn D 3thỏa mãn

Nếu n D p˛ với ˛ > vàp nguyên tố D ' n / D p˛ p / nên dễ thấy rằngp D ; ˛ D 2và n D 4thỏa mãn

Nếu n có hai ước nguyên tố khác p ; q theo tính chất nhân tính

hàm Euler,2 D ' n /sẽ chia hết cho' p / D p và' q / D q 1nên

p D q D ;tương ứng với n D 6thỏa mãn

Vậy tất số ncần tìm f3 ; ; 6g:

Bình luận Câub)của toán thực nhẹ nhàng hướng dẫn trực tiếp từ câua)

Nếu đề cho ýa)hoặc ýb)thì trọn vẹn nhiều Câu a)thực kết đẹp

và thú vị, trường hợp n số square-free chẵn (n khơng chia hết cho số

phương lớn hơn1 nào) Có ý tưởng để giải trường hợp chứng minh số lượng

các số có dạng4 k C 1và k C An nhau; nhiên, điều

n có ước nguyên tố dạng4 k C Điều chứng minh nhờ hai nhận xét:

TrongAnln chứa số có dạng b D k C 3(nếun D mC 2thì hai

sốb D mC 3và b D mC 5)

Tập hợpA0

n D fb a ja Angcó số lượng số chia 4dư1 dư3 nhưAn Chú ý

rằng a mod4 / thìb a mod4 /và ngược lại

Một kết thú vị khác đa thứcPn x /chia hết cho x2

k

C 1nếu có số nguyên tốp

thỏa mãn pjn v2 p C1 / D k :Bạn đọc tự chứng minh kết

Bài toán giải tổng Ramanujan với ý đặtCn r / D Qn r/mà

D e2 i n thìCm n D CmCn với mọim ; n nguyên dương thỏa gcd m ; n / D :

Dưới số toán tương tự:

Bài toán 1.Với số nguyên dương n ;xét đa thức Pn x / D Pa2Anx

a

vớiAnlà tập hợp ước dương của n :Tìm tất số nguyên dươngn sao choPn x /bất khả quy.

Bài toán 2(China MO, 2013).Với số nguyên dươngn ;xét đa thứcPn x / D PniD0cixi với ci f0 ; 1g ci Cni mod2 / :

a) Với m ; n ; x0 là số nguyên dương và x0 C1 không phải lũy thừa của ; chứng

minh nếuPm x0/chia hết cho Pn x0/thìPm x /chia hết cho Pn x /với mọi số nguyên dương x :

(175)

Epsilon

staff

Lời giải bình luận đề thi chọn đội tuyển IMO 2018 13

Bài (7.0 điểm) Cho a số thực thuộc đoạn 2;

2

: Xét dãy số un/ vn/

n D ; ; : : : /được xác định sau:

un D

3

2nC1 /

b2nC1ac

; D

3

2nC1 /

nCb2nC1ac :

a) Chứng minh

2

X

iD0

ui

!2

C

2

X

iD0

vi

!2

a2 a C 0C

42 9: b) Tìm tất giá trị ađể đẳng thức xảy

Lời giải a)Từ giả thiết, ta cóvi D ui vớii chẵn vàvi D ui vớii lẻ nên bất đẳng thức cần chứng minh viết lại thành

1 0

X

iD0

u2 i C 0

X

iD0

u2 iC1

!2 C

1 0

X

iD0

u2 i

1 0

X

iD0

u2 iC1

!2

a2 aC1 0C

42 9; hay

1 0

X

iD0

u2 i

!2 C

1 0

X

iD0

u2 iC1

!2

a2 a C5 C

42 9: /

Bây giờ, gọi biểu diễn nhị phân alà

a D C1 X

iD1

xi

2i

với xi f0 ; 1g: Do 12 a 23 nên dễ thấy x1 D : Với số tự nhiên i ; ta thấy tính

chẵn lẻ b2iC1ac phụ thuộc vào xiC1; cụ thể: NếuxiC1 D 0thì b2iC1ac chẵn

xiC1 D 1thì b2iC1ac lẻ Suy /b2

iC1ac

D 1nếu xiC1 D /b2

iC1ac

D

nếuxiC1 D :Trong trường hợp, ta có

/b2iC1ac D xiC1: ĐặtA D P1 0

iD0

x2 iC1

22 iC1 B D

P1 0

iD0 x2 iC2

22 iC2;ta có

1 0

X

iD0

u2 i D 0

X

iD0

3 x2 iC1/

22 iC1 D

1

2 41 0 A ; 0

X

iD0

u2 iC1 D 0

X

iD0

3 x2 iC2/

22 iC2 D

1

41 0 B :

Ngồi ra, ta cóa A C B 12 nên

3 a2 a C 5C

42 D a /

C 1C

42

A C 3B /2 C C

(176)

Epsilon

staff

14 Lời giải bình luận đề thi chọn đội tuyển IMO 2018

Ta chứng minh

2

2 41 0 A

2 C

1

41 0 B

2

AC3B /2C1C

42 9: / Bằng biến đổi tương đương, ta viết bất đẳng thức dạng

6

41 0 9A C B

1 C

41 0 A

41 0

1 42 8: DoA 12 nên6 A > C

1

41 0 9:Suy

6

41 0 9A C B

1 C

41 0 A

41 0 9A

41 0 0

X

iD0

1 22 iC1

D

41 0

2

1

41

D

41 0

1 42 8:

Bất đẳng thức (3) chứng minh Bằng cách sử dụng bất đẳng thức (2) (3), ta thu bất đẳng thức (1) Ta có điều phải chứng minh

b)Từ đánh giá trên, ta thấy dấu đẳng thức xảy a D A C B ; B D

A D 23 1 41

; hay a D 23 1 41

:

Bình luận Biểu thức b2nC1acchính gợi ý quan trọng việc xét biểu diễn nhị phân

a :Thực tế khơng có cách xác định tính chẵn lẻ đại lượng khơng dùng

đến biểu diễn nhị phân a : Ngoài ra, tốn trên, chúng tơi sử dụng đẳng thức

/a D a với a f0 ; 1gđể lời giải gọn gàng

Biểu diễn nhị phân (và biểu diễn p-phân nói chung) cơng cụ quan trọng tốn học,

có nhiều ứng dụng tổ hợp, số học, đại số giải tích Theo dõi Shortlist IMO, ta thấy ý tưởng xuất nhiều Dưới xin đưa số toán vậy:

Bài toán (Putnam, 1981).Gọi f n /là số chữ số 1trong biểu diễn nhị phân số nguyên dương n :Tính giá trị tổng:

1 X

nD1

f n / n n C /:

Bài toán (IMO Shortlist, 1983).Cho f W Œ ;  ! R là hàm số liên tục thỏa mãn đồng thời điều kiện:

8 ˆ <

ˆ :

f x / D bf x / ; x

2;

f x / D b C b / f x / ;

2 x ;

(177)

Epsilon

staff

Lời giải bình luận đề thi chọn đội tuyển IMO 2018 15

Bài toán (IMO, 1988).Hàm số f xác định tập hợp số nguyên dương cho

bởi: f / D ; f / D ; f n / D f n / ; f n C / D 2f n C / f n /

f nC3 / D 3f nC1 / 2f n /với số nguyên dươngn :Tìm số tất số nguyên

dương n ;nhỏ hay bằng 8thỏa mãn điều kiệnf n / D n :

Bài toán (IMO Shortlist, 1996).Cho dãy số a n / n D ; ; ; : : : / được xác định bởi

a / D và với mọin > 1thì

a n / D a jn

2 k

C /n n2C1 /:

a) Tìm giá trị lớn của a n /vớin là số nguyên dương 9 và tìm tất các giá trị n 9 6để giá trị lớn đạt được.

(178)

Epsilon

staff

16 Lời giải bình luận đề thi chọn đội tuyển IMO 2018

Bài (7.0 điểm) Một bảng ô vng m n A B C D có đỉnh giao lộ (có tất mC / n C / giao lộ) Người ta muốn thiết lập tuyến đường bắt đầu từA ; theo cạnh song song với cạnh hình chữ nhật qua tất giao lộ lần, sau quay A :

a) Chứng minh xây dựng đường mlẻ nlẻ

b) Vớim ; nthỏa mãn điều kiện câu a), hỏi có giao lộ mà có ngã rẽ?

Lời giải Đánh số hàng cột theo thứ tự từ trái sang phải1 ! m C1 ; xuống

1 ! n C :Khi đó, điểmA góc bên trái với vị trí ; / :

a) Điều kiện cần:Ta thấy đường biểu diễn dãy ký tự L ; R ; U

và D ; ký tự hướng sang trái, sang phải, lên xuống

dưới giao lộ

Do có tổng cộng m C / nC / giao lộ nên có nhiêu số ký tựL ; R ; U ; D:

Vì đường xuất phát từ A quay trở A nên số lần rẻ trái số lần rẻ phải, số lần

xuống số lần lên Điều chứng tỏ số ký tự L ; R nhau, số ký tựU ; D

nhau Suy tổng số ký tự mC / n C /phải chẵn Vì nên phải có m lẻ hoặcn lẻ

Điều kiện đủ:Giả sửm số lẻ, trường hợpn lẻ hoàn toàn tương tự Ta theo quy tắc sau (có mơ tả hình trên):

Lần ngang đầu tiên: xuất phát tạiA ; từ cột1 ! n

Mỗi lần dọc xuống ơ; ngang di chuyển cột $ n :

Lần ngang cuối cùng: từ cộtn ! 1sau ngược từ hàng n ! đến điểmA :

Điều thực dom lẻ nên số tuyến đường nằm ngang chẵn Vậy điều kiện cần

đủ để đề mhoặc nphải lẻ

b)Xét hai giao lộ có ngã rẽ gần (điểmA tạm tính giao lộ đó)

hai giao lộ có đoạn đường, nằm ngang nằm dọc Ta thấy đường dãy đoạn dọc ngang luân phiên (mỗi đoạn qua hai nhiều giao lộ), xuất phát theo chiều ngang theo chiều dọc ngược lại

Gọi số đoạn ngang r, đoạn dọc làc k số ngã rẽ (khơng tính A) Ta chứng minh

các nhận xét sau:

(179)

Epsilon

staff

Lời giải bình luận đề thi chọn đội tuyển IMO 2018 17

Chứng minh.Tại giao lộ chuyển có ngã rẽ đoạn ngang dọc Số ngã rẽ,

tính thêm A ;chính số cặp khơng thứ tự có dạng fđoạn ngang, đoạn dọcgmà đoạn

ngang dọc có chung đầu mút giao lộ Hơn nữa, đoạn ngang có chung

đầu mút với 2đoạn dọc, đoạn dọc có chung đầu mút với 2đoạn ngang nên

k C D r D c :

Để tìm giá trị nhỏ củak, ta đưa tìm giá trị nhỏ r c :

Nhận xét 2.r m C 1hoặcc nC :

Chứng minh.Giả sử r m số đoạn ngang không xuất đủ m C 1hàng,

do có hàng mà n C giao lộ ứng với đoạn dọc, tức c n C :

Tương tự nếuc n thìr m C :

Từ đó, ta đưa xét trường hợp sau đây:

Nếu m lẻ, n chẵn:Giả sửr mthì theo nhận xét trên, có hàng mà cảnC1giao

lộ ứng với đoạn dọc, có lẻ điểm hàng nên với đoạn dọc đó, ta

khơng thể quay đượcA, khơng thỏa Do r mC1vàk D r mC1

Ta dễ dàng xây dựng đường với đúngm C đoạn dọc tương tự câu a) Vì

nên mink D m C1 :

Nếu m chẵn, n lẻ:Lập luận tương tự, ta có minkD2nC1:

Nếu m; n lẻ:Dễ thấy hai đẳng thức đánh giár mC1; c nC1

đều xảy nên mink D2min.m; n/C1:

Nói tóm lại:

Nếum; nđều lẻ mink D2min.m; n/C1:

Nếumchẵn,nlẻ minkD2nC1:

Nếumlẻ,nchẵn minkD2mC1:

Bình luận Đây tốn kinh điển đường Hamilton qua tất đỉnh

grid graph, tham khảo thêm liên kết:http://mathworld.wolfram.com/

GridGraph.html

Kết câua)đã cũ chí, ta cịn đếm số đường theo m ; nnhờ

công thức truy hồi Ngoài cách giải trên, ta tiếp cận theo hướng tơ màu sau:

(180)

Epsilon

staff

18 Lời giải bình luận đề thi chọn đội tuyển IMO 2018

Khi đó, lần từ giao lộ sang giao lộ kia, màu thay đổi Để qua hết giao

lộ, giao lộ lần quay A rõ ràng, màu phải đổi chẵn lần Suy

m C1 / n C / chẵn hay hai sốm ; nlẻ

Ở câub), ta cần phải đánh giá cẩn thận quan hệ số lần ngang dọc

vai trị tính chẵn lẻ sốm ; n :

Một số toán tương tự:

Bài tốn (Tạp chí Crux) Một chuột ăn miếng phơ mai hình lập phương kích thước 3 3gồm miếng nhỏ1 1 nó ăn phần ruột bên miếng/ :

Hỏi xuất phát từ miếng góc qua được miếng kết thúc tại miếng không, biết di chuyển hai miếng có chung mặt với nhau và không quay miếng qua

(181)

Epsilon

staff

Lời giải bình luận đề thi chọn đội tuyển IMO 2018 19

Bài (7.0 điểm) Cho tam giác A B C nhọn nội tiếp O / J /là đường trịn bàng tiếp góc Acủa tam giác Gọi D ; E ; F tiếp điểm J /với B C ; C A A B :

a) Gọi L trung điểm B C : Đường trịn đường kính LJ cắt đường thẳng

DE ; DF K H : Chứng minh đường tròn BDK /

C DH / cắt đường tròn J / :

b) Giả sử đường thẳng EF cắt đường thẳng B C G GJ cắt đường thẳng

A B ; A C M ; N :GọiP vàQlà điểm đường thẳngJ B ; J C

sao cho ∠P A B D ∠QA C D 0ı: Gọi T giao điểm hai đường thẳng

P M ; QN S điểm cung lớnB C đường trịn O / : GọiI tâm

đường tròn nội tiếp tam giác A B C : Chứng minh đường thẳng S I cắt đường thẳngA T điểm thuộc đường tròn O / :

Lời giải a)Gọi D0và F0 tiếp điểm đường tròn I /nội tiếp tam giác A B C

với B C ; BA :Dựng đường kínhD W đường trịn J / : Ta có

A I A J D

I F0

J F D I D0

J W

nên ba điểmA ; D0và W thẳng hàng.

A

B C

J F

E O

D L I

H D0

F0

D1

W

X U

U0

Z V

(182)

Epsilon

staff

20 Lời giải bình luận đề thi chọn đội tuyển IMO 2018

GọiD1là giao điểm đường thẳngID0vàAD:Ta cóJD DJ W nênID0DID1:Mà

BD0 DCDnênLD DLD0;từ suy raILkDD1:

GọiXlà trung điểm củaD0W tứ giácDLXJ hình chữ nhật Suy ra∠XHD D90ı;hay

XH kJB:Từ đóXH ?BI:

Dựng hình chữ nhật C D J U : Ta có J U k C D J U D C D nên J U k BD0

J U D BD0; suy tứ giác BD0U J hình bình hành Từ đây, ta thu D0U k XH :

Gọi Y ; V vàZ giao điểm đường thẳngXH với đường thẳng D J ; U W ; U C :Khi đó, ta cóV trung điểm củaU W ; màU Z k Y W nênV trung điểm Y Z :

Xét tứ giác C E U J nội tiếp có C E D C D D U J nên U E k C J hay U E ? DE : Mà

E W ? ED nên ba điểmW ; U E thẳng hàng

Xét tam giácB I C Y V W ;ta có

∠Y W V D ∠C ED D ∠I C B

∠W Y V D ∠X Y J D ∠B J D D ∠I B C

nên4B I C 4Y V W (g-g) MàV trung điểm củaY ZvàLlà trung điểm củaB C nên ta có4B I L 4Y Z W (c-g-c) Do ∠Y W Z D ∠B LI D ∠BDA D 0ı ∠W D U0:

Từ suy ∠D U0W D 9 0ı với U0 là giao điểm đường thẳng W Z và A D: Mặt

khác, ta lại có ∠D U0W D ∠D U0Z D 0ınên U0 thuộc đường tròn C DH /và J / :

Chứng minh tương tự, đường tròn BDK / quaU0:Từ đó, ta có điều phải chứng minh

b)(Hình vẽ xem trang sau) Gọi S0 R giao điểm thứ hai đường thẳng

A I ; S I đường tròn O / : Ta cóS0I2 D S0L S0S nên ∠S0I L D ∠S0S I D ∠SAR :

MàI L k A D nên∠S0I L D ∠S0A D ;từ suy ∠S0AR D ∠S0A D hay

∠BAR D ∠C A D: /

KẻDH0 ? J G H0 J G / : Ta cóGH0 GJ D GD2 D GE GF nên tứ giác EF J H0

nội tiếp Lại có tứ giácA E J F nội tiếp nên điểmA ; E ; F ; J H0đồng viên, từ suy

ra∠A H0J D 9 0ı hay ba điểmA ; D và H0 thẳng hàng.

Mặt khác, ta có ∠P A J D ∠QA J D 0ı C 21∠BA C D ∠B I C ∠I B C D ∠I J C ;

∠I C B D ∠I J B nên4I B C 4AP J 4A J Q(g-g)

Gọi P0; Q0 trung điểm J P J Q 4I B L 4AP P0 4A J Q0

4I C L 4AQ Q0:Suy tứ giác AP0J Q0nội tiếp

Từ ∠B I L D ∠JAQ0 D ∠J P0Q0 D ∠BP Q ; ta suy tứ giác P B I L0 nội tiếp (L0 là

giao điểm đường thẳng LI P Q) Mà ∠P B I D 0ı nên I L ? P Q : Lại có

I L k A D A D ? M N nên M N k P Q :

QuaM kẻ đường thẳng song song với đường thẳngAP cắt đường thẳngA T tạiA0:Ta có

T A0 T A D

T M T P D

T N T Q;

suy NA0 k AQ ∠A M A0 D ∠A NA0 D 0ı: Từ

∠BA T D 0ı ∠A A0M D 0ı ∠A N M D ∠C A D: /

(183)

Epsilon

staff

Lời giải bình luận đề thi chọn đội tuyển IMO 2018 21

A

B

C

J F

E O

D L I

G

M

N P

Q

T S

S0

R L0

H0

P0

Q0

A0

Bình luận Đây tốn khó Hai ý tốn khơng liên quan tới Ýa)có thể tổng quát (và đơn giản hơn) thành toán sau:

(184)

Epsilon

staff

22 Lời giải bình luận đề thi chọn đội tuyển IMO 2018

Tính chất cịn mở rộng cách sử dụng điểm Miquel:

Bài toán 2.Cho tam giácA B C có điểmD ; E ; F bất kỳ nằm cạnhB C ; C A ; A B :

Các đường tròn A EF / ; C F D / ; C DE / có điểm chung làM : Một đường trịn K /

đi qua P D cắt lại đường thẳngDE ; DF lần lượt tại Q R :Chứng minh các đường tròn DB Q / ; D C R / DEF / có điểm chung khác D:

Lời giải hai toán tổng qt ngồi phương pháp cộng góc thơng thường ta có

thể sử dụng phép nghịch đảo cực D:

Ýb)của TST tốn thách thức địi hỏi học sinh phải có nhiều kỹ biết

một số bổ đề giải trọn vẹn Ý tổng quát cho tam giác pedal sau:

Bài toán 3.Cho tam giácA B C và điểm P bất kỳ mặt phẳng Gọi DEF là tam giác pedal điểmP : Đường thẳngEF cắt đường thẳng B C tại G : Đường thẳng GP cắt các đường thẳngC A A B lần lượt tạiQ ; R :Các đường thẳng quaAvng gócA B A C cắt các đường thẳngP B ; P C tạiM ; N : Đường thẳng N Qcắt đường thẳngM Rtại L :Đường

tròn DEF /cắt lại đường thẳng B C tại K :Chứng minh rằng ∠K A B D ∠L A C :

Khi P trùng với tâm đường tròn bàng tiếp góc Acủa tam giác A B C ta thu ýb)của

bài TST Đây mở rộng theo kiểu đẳng giác có ý nghĩa phức tạp Sơ lược bước chứng minh cho toán tổng quát sau:

Chứng minh M N song song vớiGP cách sử dụng tỉ số kép, tức chứng minh

P B C ; D G / D P M N ; D1/ :

Chứng minh hai tam giác K QRvà A M N thấu xạ tâm T định lý Desargues,

với định lý dùng để biến đổi tỉ số Thales, Menelaus

Cuối hai đường thẳngA LvàA T đẳng giác với góc∠M A N (cũng đẳng

giác góc∠BA C) thơng qua bổ đề:Cho hình thangA B C D có hai đáy làA B

C D: Đường thẳngA D cắt đường thẳngB C tại E : Các đường chéo A C BD cắt nhau tại F : Nếu điểmM thỏa mãn M A ; M B đẳng giác với góc ∠C M D thì M A ; M B cũng đẳng giác với góc ∠E M F :

Có thể tham khảo thêm mở rộng liên kết:http://analgeomatica.blogspot

http://forum.mathscope.org/showthread.php?t=46988 http://analgeomatica.blogspot.com/2014/03/xung-quanh-mot-bai-toan-hinh-hoc-trong.html http://analgeomatica.blogspot.com/2014/03/tom-tat.html http://analgeomatica.blogspot.com/2014/02/inh-ly-sawayama-va-thebault.html http://www.artofproblemsolving.com/Forum/viewtopic.php?p=450331 https://www.facebook.com/sputnikedu/posts/720502957971422 http://www.artofproblemsolving.com/Forum/viewtopic.php?p=352892 https://www.writelatex.com/read/htndbgqrjqzp https://artofproblemsolving.com/community/c6h1299041. https://artofproblemsolving.com/community/c6h17323p118682. http://analgeomatica.blogspot.com/2016/01/ve-bai-toan-g7-trong-imo-shortlist-2002.html . https://artofproblemsolving.com/community/q1h1180223. http://analgeomatica.blogspot.com/2018/04/cac-bai-toan-hinh-hoc-hang-tuan-tuan-1.html http://mathworld.wolfram.com/GridGraph.html

Ngày đăng: 08/02/2021, 01:57

Từ khóa liên quan

Tài liệu cùng người dùng

Tài liệu liên quan